Transforms and Pde

You might also like

Download as pdf or txt
Download as pdf or txt
You are on page 1of 143

UNIT-I

FOURIER SERIES

Periodic function :

A function 𝑓 𝑥 is said to be periodic, if and only if 𝑓 𝑥 + 𝑇 = 𝑓 𝑥 where T is called period


for the function 𝑓 𝑥 .

Eg: sin 𝑥 𝑎𝑛𝑑 cos 𝑥 are periodic functions with period 2𝜋. 𝑖. 𝑒 sin 2𝜋 + 𝑥 = sin 𝑥

cos 2𝜋 + 𝑥 = cos 𝑥

Fourier series:

Fourier series is an infinite trigonometric series defined by



𝑎0 𝑛𝜋𝑥 𝑛𝜋𝑥
+ 𝑎𝑛 cos + 𝑏𝑛 sin , 𝑐 ≤ 𝑥 ≤ 𝑐 + 2𝑙
2 𝑙 𝑙
𝑛=1

Where 𝑎0 , 𝑎𝑛 𝑎𝑛𝑑 𝑏𝑛 are called Euler’s constants or Fourier coefficients.

Dirichlet’s conditions:

A function defined in 𝑐 ≤ 𝑥 ≤ 𝑐 + 2𝑙 can be expanded as an infinite trigonometric series of


the form

𝑎0 𝑛𝜋𝑥 𝑛𝜋𝑥
+ 𝑎𝑛 cos + 𝑏𝑛 sin , provided
2 𝑙 𝑙
𝑛 =1

i) 𝑓 𝑥 is single valued and periodic in 𝑐, 𝑐 + 2𝑙

ii) 𝑓 𝑥 is continuous or piecewise continuous with finite number of finite discontinuous in


𝑐, 𝑐 + 2𝑙

iii) 𝑓 𝑥 has no or finite number of maxima or minima in 𝑐, 𝑐 + 2𝑙 .

Fourier coefficients for the function f(x) in the interval 𝒄 ≤ 𝒙 ≤ 𝒄 + 𝟐𝒍 is given by

𝑐+2𝑙 𝑐+2𝑙
1 1 𝑛𝜋𝑥
𝑎0 = 𝑓 𝑥 𝑑𝑥 , 𝑎𝑛 = 𝑓 𝑥 𝑐𝑜𝑠 𝑑𝑥 ,
𝑙 𝑙 𝑙
𝑐 𝑐

𝑐+2𝑙
1 𝑛𝜋𝑥
𝑏𝑛 = 𝑓 𝑥 𝑠𝑖𝑛 𝑑𝑥
𝑙 𝑙
𝑐

Fourier coefficients for the function f(x) in the interval 𝟎 ≤ 𝒙 ≤ 𝟐𝒍 is given by


2𝑙 2𝑙 2𝑙
1 1 𝑛𝜋𝑥 1 𝑛𝜋𝑥
𝑎0 = 𝑓 𝑥 𝑑𝑥 , 𝑎𝑛 = 𝑓 𝑥 𝑐𝑜𝑠 𝑑𝑥 , 𝑏𝑛 = 𝑓 𝑥 𝑠𝑖𝑛 𝑑𝑥
𝑙 𝑙 𝑙 𝑙 𝑙
0 0 0

Fourier coefficients for the function f(x) in the interval −𝒍 ≤ 𝒙 ≤ 𝒍 is given by

𝑙 𝑙 𝑙
1 1 𝑛𝜋𝑥 1 𝑛𝜋𝑥
𝑎0 = 𝑓 𝑥 𝑑𝑥 , 𝑎𝑛 = 𝑓 𝑥 𝑐𝑜𝑠 𝑑𝑥 , 𝑏𝑛 = 𝑓 𝑥 𝑠𝑖𝑛 𝑑𝑥
𝑙 𝑙 𝑙 𝑙 𝑙
−𝑙 −𝑙 −𝑙

Fourier series for the function f(x) in the interval 𝟎 ≤ 𝒙 ≤ 𝟐𝝅.



𝑎0
𝑓 𝑥 = + 𝑎𝑛 cos 𝑛𝑥 + 𝑏𝑛 sin 𝑛𝑥
2
𝑛 =1

Where
2𝜋 2𝜋 2𝜋
1 1 1
𝑎0 = 𝑓 𝑥 𝑑𝑥, 𝑎𝑛 = 𝑓 𝑥 𝑐𝑜𝑠 𝑛𝑥𝑑𝑥, 𝑏𝑛 = 𝑓 𝑥 𝑠𝑖𝑛 𝑛𝑥𝑑𝑥
𝜋 𝜋 𝜋
0 0 0

Fourier series for the function f(x) in the interval −𝝅 ≤ 𝒙 ≤ 𝝅.



𝑎0
𝑓 𝑥 = + 𝑎𝑛 cos 𝑛𝑥 + 𝑏𝑛 sin 𝑛𝑥
2
𝑛 =1

Where
𝜋 𝜋 𝜋
1 1 1
𝑎0 = 𝑓 𝑥 𝑑𝑥 , 𝑎𝑛 = 𝑓 𝑥 𝑐𝑜𝑠 𝑛𝑥𝑑𝑥 , 𝑏𝑛 = 𝑓 𝑥 𝑠𝑖𝑛 𝑛𝑥𝑑𝑥
𝜋 𝜋 𝜋
−𝜋 −𝜋 −𝜋

Fourier series for the even function f(x) in the interval −𝝅 ≤ 𝒙 ≤ 𝝅. (or)

Half range cosine series in the interval 𝟎 ≤ 𝒙 ≤ 𝝅.


𝜋 𝜋
2 2
𝑎0 = 𝑓 𝑥 𝑑𝑥 , 𝑎𝑛 = 𝑓 𝑥 cos 𝑛𝑥𝑑𝑥 , 𝑏𝑛 = 0
𝜋 𝜋
0 0

then the Fourier series is reduced to



𝑎0
𝑓 𝑥 = + 𝑎𝑛 cos 𝑛𝑥
2
𝑛=1

Fourier series for the odd function f(x) in the interval−𝝅 ≤ 𝒙 ≤ 𝝅. (or)
Half range sine series in the interval 𝟎 ≤ 𝒙 ≤ 𝝅.
𝜋
2
𝑎0 = 0, 𝑎𝑛 = 0, 𝑏𝑛 = 𝑓 𝑥 𝑠𝑖𝑛 𝑛𝑥𝑑𝑥 ,
𝜋
0

then the Fourier series is reduced to


𝑓 𝑥 = 𝑏𝑛 sin 𝑛𝑥
𝑛=1

Fourier series for the even function f(x) in the interval −𝒍 ≤ 𝒙 ≤ 𝒍. (or)

Half range Fourier cosine series in the interval 𝟎 ≤ 𝒙 ≤ 𝒍

𝑙 𝑙
2 2 𝑛𝜋𝑥
𝑎0 = 𝑓 𝑥 𝑑𝑥 , 𝑎𝑛 = 𝑓 𝑥 𝑐𝑜𝑠 𝑑𝑥 , 𝑏𝑛 = 0
𝑙 𝑙 𝑙
0 0

then the Fourier series is reduced to



𝑎0 𝑛𝜋𝑥
𝑓 𝑥 = + 𝑎𝑛 cos
2 𝑙
𝑛=1

Fourier series for the odd function f(x) in the interval −𝒍 ≤ 𝒙 ≤ 𝒍. (or)

Half range Fourier sine series in the interval 𝟎 ≤ 𝒙 ≤ 𝒍

𝑙
2 𝑛𝜋𝑥
𝑎0 = 0, 𝑎𝑛 = 0, 𝑏𝑛 = 𝑓 𝑥 𝑠𝑖𝑛 𝑑𝑥
𝑙 𝑙
0
then the Fourier series is reduced to

𝑛𝜋𝑥
𝑓 𝑥 = 𝑏𝑛 sin
𝑙
𝑛=1

Fourier series for the complex function f(x) in the interval −𝝅 ≤ 𝒙 ≤ 𝝅


𝑓(𝑥) = 𝑐𝑛 𝑒 𝑖𝑛𝑥
𝑛=−∞

where
𝜋
1
𝑐𝑛 = 𝑓(𝑥)𝑒 −𝑖𝑛𝑥 𝑑𝑥
2𝜋
−𝜋
Parseval’s identity for the Fourier series in the interval 𝟎, 𝟐𝝅

2𝜋 ∞
1 2
𝑎02
𝑓 𝑥 𝑑𝑥 = + 𝑎𝑛2 + 𝑏𝑛2
𝜋 2
0 𝑛=1

Parseval’s identity for the Fourier series in the interval 𝟎, 𝟐𝒍

2𝑙 ∞
1 2
𝑎02
𝑓 𝑥 𝑑𝑥 = + 𝑎𝑛2 + 𝑏𝑛2
𝑙 2
0 𝑛=1

Parseval’s identity for the Fourier series in the interval −𝝅, 𝝅


𝜋 ∞
1 𝑎02
𝑓 𝑥 𝑑𝑥 =
2
+ 𝑎𝑛2 + 𝑏𝑛2
𝜋 2
−𝜋 𝑛=1

Parseval’s identity for the Fourier series in the interval −𝒍, 𝒍

𝑙 ∞
1 2
𝑎02
𝑓 𝑥 𝑑𝑥 = + 𝑎𝑛2 + 𝑏𝑛2
𝜋 2
−𝑙 𝑛=1

Parseval’s identity for the half range Fourier cosine series in the interval 𝟎, 𝝅
𝜋 ∞
2 2
𝑎02
𝑓 𝑥 𝑑𝑥 = + 𝑎𝑛2
𝜋 2
0 𝑛=1

Parseval’s identity for the half range Fourier sine series in the interval 𝟎, 𝝅
𝜋 ∞
2 2
𝑓 𝑥 𝑑𝑥 = 𝑏𝑛2
𝜋
0 𝑛=1

Parseval’s identity for the half range Fourier cosine series in the interval 𝟎, 𝒍

𝑙 ∞
2 2
𝑎02
𝑓 𝑥 𝑑𝑥 = + 𝑎𝑛2
𝑙 2
0 𝑛=1

Parseval’s identity for the half range Fourier sine series in the interval 𝟎, 𝟐𝒍
𝑙 ∞
2
𝑓 𝑥 2
𝑑𝑥 = 𝑏𝑛2
𝑙
0 𝑛=1

Root-Mean Square value of a function 𝒇 𝒙

Solution:

RMS value of a function 𝑓 𝑥

𝑏 2 𝑑𝑥
𝑎
𝑓 𝑥
=
𝑏−𝑎

Problems:

1.Find the Fourier series for 𝑓(𝑥) = 𝑥 𝑠𝑖𝑛𝑥, 0 < 𝑥 < 2𝜋.

Solution: We know that



𝑎0
𝑓 𝑥 = + 𝑎𝑛 cos 𝑛𝑥 + 𝑏𝑛 sin 𝑛𝑥
2
𝑛 =1

Where
2𝜋 2𝜋
1 1
𝑎0 = 𝑓 𝑥 𝑑𝑥 = 𝑥 sin 𝑥 𝑑𝑥
𝜋 𝜋
0 0

1 2𝜋
= 𝑥(− cos 𝑥) − 1(− sin 𝑥) 0
𝜋
1
= 2𝜋(− cos 2𝜋) − 1(− sin 2𝜋) − 0 = −2
𝜋
2𝜋
1
𝑎𝑛 = 𝑓 𝑥 𝑐𝑜𝑠 𝑛𝑥𝑑𝑥
𝜋
0

2𝜋
1
= 𝑥 sin 𝑥 𝑐𝑜𝑠 𝑛𝑥𝑑𝑥
𝜋
0

2𝜋
1
= 𝑥 sin 1 + 𝑛 𝑥 + sin 1 − 𝑛 𝑥 𝑑𝑥
2𝜋
0

2𝜋
1 − cos 1 + 𝑛 𝑥 − sin 1 + 𝑛 𝑥
= 𝑥 − 1.
2𝜋 1+𝑛 1+𝑛 2 0
2𝜋
1 − cos 1 − 𝑛 𝑥 − sin 1 − 𝑛 𝑥
+ 𝑥 − 1.
2𝜋 1−𝑛 1−𝑛 2 0

1 1
=− +
1+𝑛 1−𝑛
2
=− ,𝑛 ≠ 1
1 − 𝑛2
When 𝑛 = 1
2𝜋
1
𝑎1 = 𝑓 𝑥 𝑐𝑜𝑠 𝑥𝑑𝑥
𝜋
0

2𝜋
1
= 𝑥 sin 𝑥 𝑐𝑜𝑠 𝑥𝑑𝑥
𝜋
0

2𝜋
1
= 𝑥 sin 2𝑥 𝑑𝑥
2𝜋
0

2𝜋
1 − cos 2𝑥 − sin 2𝑥 1
= 𝑥 − 1. =−
2𝜋 2 22 0 2
2𝜋
1
𝑏𝑛 = 𝑓 𝑥 𝑠𝑖𝑛 𝑛𝑥𝑑𝑥
𝜋
0

2𝜋
1
= 𝑥 sin 𝑥 𝑠𝑖𝑛 𝑛𝑥𝑑𝑥
𝜋
0

2𝜋
1
= 𝑥 cos 1 − 𝑛 𝑥 − cos 1 + 𝑛 𝑥 𝑑𝑥
2𝜋
0

2𝜋
1 sin 1 − 𝑛 𝑥 − cos 1 − 𝑛 𝑥
= 𝑥 − 1.
2𝜋 1−𝑛 1−𝑛 2 0

2𝜋
1 sin 1 + 𝑛 𝑥 − cos 1 + 𝑛 𝑥
− 𝑥 − 1. = 0, 𝑛 ≠ 1
2𝜋 1+𝑛 1+𝑛 2 0

When 𝑛 = 1
2𝜋
1
𝑏1 = 𝑓 𝑥 𝑠𝑖𝑛 𝑥𝑑𝑥
𝜋
0

2𝜋
1
= 𝑥 sin 𝑥 𝑠𝑖𝑛 𝑥𝑑𝑥
𝜋
0

2𝜋
1
= 𝑥 1 − cos 2𝑥 𝑑𝑥
2𝜋
0

2𝜋
1 sin 2𝑥 𝑥 2 cos 2𝑥
= 𝑥 𝑥− − 1. −
2𝜋 2 2 22 0

1 4𝜋 2
= 4𝜋 2 − =𝜋
2𝜋 2

1 2
𝑓 𝑥 = −1 − 𝑐𝑜𝑠𝑥 + cos 𝑛𝑥 + 𝜋 sin 𝑥
2 𝑛2 −1
𝑛=2

2. Show that for

2𝑙 𝜋𝑥 1 2𝜋𝑥 1 3𝜋𝑥
0 < 𝑥 < 𝑙, 𝑥 = 𝑠𝑖𝑛 − 𝑠𝑖𝑛 + 𝑠𝑖𝑛 −⋯
𝜋 𝑙 2 𝑙 3 𝑙

Using root mean square value of 𝑥, deduce the value of

1 1 1
2
+ 2 + 2 +⋯
1 2 3

Solution: We know that



𝑛𝜋
𝑓 𝑥 = 𝑏𝑛 sin 𝑥
𝑙
𝑛=1

Where

𝑙
2 𝑛𝜋
𝑏𝑛 = 𝑓 𝑥 sin 𝑥 𝑑𝑥
𝑙 𝑙
0

𝑙
2 𝑛𝜋
= 𝑥 sin 𝑥 𝑑𝑥
𝑙 𝑙
0
𝑙
𝑛𝜋 𝑛𝜋
cos 𝑙 𝑥 sin
2
= 𝑥 − −1 − 𝑙 𝑥
𝑙 𝑛𝜋 𝑛𝜋 2
𝑙 𝑙 0

2𝑙 𝑛+1
= −1
𝑛𝜋

2𝑙 𝑛+1
𝑛𝜋
𝑥= −1 sin 𝑥
𝑛𝜋 𝑙
𝑛=1

2𝑙 𝜋𝑥 1 2𝜋𝑥 1 3𝜋𝑥
𝑥= 𝑠𝑖𝑛 − 𝑠𝑖𝑛 + 𝑠𝑖𝑛 −⋯
𝜋 𝑙 2 𝑙 3 𝑙

Using RMS value

𝑙 ∞
2
𝑓 𝑥 2
𝑑𝑥 = 𝑏𝑛2
𝑙
0 𝑛=1

𝑙 ∞ 2
2 2𝑙
𝑥 𝑑𝑥 =
2
−1 𝑛+1
𝑙 𝑛𝜋
0 𝑛=1

𝑙 ∞
2 𝑥3 4𝑙2 1 𝑛+1
2
= −1
𝑙 3 0
𝜋2 𝑛
𝑛=1

2 𝑙3 4𝑙2 1 1 1
= 2 + + +⋯
𝑙 3 𝜋 12 22 32

1 1 1 𝜋2
+ + +⋯=
12 22 32 6

3. Obtain Fourier series for 𝑓(𝑥) = 𝑥 2 in 0 ≤ 𝑥 ≤ 2𝜋.

Solution: We know that



𝑎0
𝑓 𝑥 = + 𝑎𝑛 cos 𝑛𝑥 + 𝑏𝑛 sin 𝑛𝑥
2
𝑛 =1

Where

2𝜋 2𝜋
1 1
𝑎0 = 𝑓 𝑥 𝑑𝑥 = 𝑥 2 𝑑𝑥
𝜋 𝜋
0 0
2𝜋
1 𝑥3 8𝜋 2
= =
𝜋 3 0
3

2𝜋
1
𝑎𝑛 = 𝑓 𝑥 𝑐𝑜𝑠 𝑛𝑥𝑑𝑥
𝜋
0

2𝜋
1
= 𝑥 2 𝑐𝑜𝑠 𝑛𝑥𝑑𝑥
𝜋
0

2𝜋
1 sin 𝑛𝑥 cos 𝑛𝑥 sin 𝑛𝑥 4
= 𝑥2 −2𝑥 − 2
+2 − =
𝜋 𝑛 𝑛 𝑛3 0 𝑛2
2𝜋
1
𝑏𝑛 = 𝑓 𝑥 𝑠𝑖𝑛 𝑛𝑥𝑑𝑥
𝜋
0

2𝜋
1
= 𝑥 2 𝑠𝑖𝑛 𝑛𝑥𝑑𝑥
𝜋
0

2𝜋
1 cos 𝑛𝑥 sin 𝑛𝑥 cos 𝑛𝑥 −4
= 𝑥2 − −2𝑥 − + 2 = 𝜋
𝜋 𝑛 𝑛2 𝑛3 0 𝑛

4𝜋 2 1 𝜋
𝑓 𝑥 = +4 2
cos 𝑛𝑥 − sin 𝑛𝑥
3 𝑛 𝑛
𝑛 =1

4. Prove that in the interval

𝑙 4𝑙 𝜋𝑥 1 3𝜋𝑥
0 < 𝑥 < 𝑙, 𝑥 = − 2 𝑐𝑜𝑠 + 2 𝑐𝑜𝑠 +⋯
2 𝜋 𝑙 3 𝑙

and deduce that

1 1 1 𝜋4
+ + + ⋯ =
14 34 54 96

Solution: We know that



𝑎0 𝑛𝜋𝑥
𝑓 𝑥 = + 𝑎𝑛 cos
2 𝑙
𝑛=1

where
𝑙 𝑙
2 2
𝑎0 = 𝑓 𝑥 𝑑𝑥 = 𝑥 𝑑𝑥
𝑙 𝑙
0 0

𝑙
2 𝑥2
= =𝑙
𝑙 2 0

𝑙
2 𝑛𝜋𝑥
𝑎𝑛 = 𝑓 𝑥 cos 𝑑𝑥
𝑙 𝑙
0

𝑙
2 𝑛𝜋𝑥
= 𝑥 cos 𝑑𝑥
𝑙 𝑙
0

𝑙
𝑛𝜋𝑥 𝑛𝜋𝑥
2 sin cos
= 𝑥 𝑙 −1 − 𝑙
𝑙 𝑛𝜋 𝑛𝜋 2
𝑙 𝑙 0

2 2
2 𝑙 𝑙
= (−1)n+1 −
𝑙 𝑛𝜋 𝑛𝜋

2𝑙
= (−1)n − 1
𝑛2 𝜋 2

When 𝑛 is odd

4𝑙
𝑎𝑛 = −
𝑛2 𝜋 2
When 𝑛 is even

𝑎𝑛 = 0

𝑙 4𝑙 1
𝑓 𝑥 = − 2 cos 𝑛𝑥
2 𝜋 𝑛2
𝑛 =1

𝑙 4𝑙 𝜋𝑥 1 3𝜋𝑥
𝑥= − 2 𝑐𝑜𝑠 + 2 𝑐𝑜𝑠 +⋯
2 𝜋 𝑙 3 𝑙

Using Parseval’s identity

𝑙 ∞
2 𝑎02
𝑓 𝑥 𝑑𝑥 =
2
+ 𝑎𝑛2
𝑙 2
0 𝑛=1
𝑙 ∞
2 𝑙2 4𝑙 2
𝑥 𝑑𝑥 =
2
+ −
𝑙 2 𝑛2 𝜋2
0 𝑛=1,3,5,…

𝑙 ∞
2 𝑥3 𝑙2 16𝑙2 1 2
= +
𝑙 3 0
2 𝜋4 𝑛2
𝑛=1,3,5,…

2𝑙2 𝑙2 16𝑙2 1 1 1
− = 4 + + +⋯
3 2 𝜋 14 34 54

16𝑙 2 1 1 1 𝑙2
+ + +⋯ =
𝜋 4 14 34 54 6

1 1 1 𝜋4
+ + + ⋯ =
14 34 54 96

5. Find the complex form of Fourier series of the function 𝑓(𝑥) = 𝑒 𝑎𝑥 , −𝜋 ≤ 𝑥 ≤ 𝜋

In the form


𝑎𝑥
sinh 𝑎𝜋 𝑛
𝑎 + 𝑖𝑛 𝑖𝑛𝑥
𝑒 = −1 𝑒
𝜋 𝑎2 + 𝑛2
−∞

And hence prove that


𝜋 −1 𝑛
=
a sinh 𝑎𝜋 𝑎2 + 𝑛2
−∞

Solution: We know that


𝑓(𝑥) = 𝑐𝑛 𝑒 𝑖𝑛𝑥
𝑛=−∞

where
𝜋
1
𝑐𝑛 = 𝑒 𝑎𝑥 𝑒 −𝑖𝑛𝑥 𝑑𝑥
2𝜋
−𝜋

𝜋
1 𝑎−𝑖𝑛 𝑥
= 𝑒 𝑑𝑥
2𝜋
−𝜋
𝜋
1 𝑒 𝑎−𝑖𝑛 𝑥 1 𝑎−𝑖𝑛 𝜋 𝑎−𝑖𝑛 −𝜋
= = 𝑒 −𝑒
2𝜋 𝑎 − 𝑖𝑛 −𝜋
2𝜋 𝑎 − 𝑖𝑛

𝑎 + 𝑖𝑛
= 𝑒 𝑎𝜋 𝑒 −𝑖𝑛 𝜋 − 𝑒 −𝑎𝜋 𝑒 𝑖𝑛 𝜋
2𝜋 𝑎 − 𝑖𝑛 𝑎 + 𝑖𝑛

𝑎 + 𝑖𝑛
= 𝑒 𝑎𝜋 cos 𝑛𝜋 − 𝑖𝑠𝑖𝑛 𝑛𝜋 − 𝑒 −𝑎𝜋 cos 𝑛𝜋 + 𝑖𝑠𝑖𝑛 𝑛𝜋
2𝜋 𝑎2 + 𝑛2
𝑛
𝑎 + 𝑖𝑛 −1 𝑒 𝑎𝜋 − 𝑒 −𝑎𝜋
=
𝜋 𝑎2 + 𝑛2 2
𝑛
𝑎 + 𝑖𝑛 −1
𝑐𝑛 = sinh 𝑎𝜋
𝜋 𝑎2 + 𝑛2


sinh 𝑎𝜋 𝑎 + 𝑖𝑛 𝑖𝑛𝑥
𝑒 𝑎𝑥 = −1 𝑛
𝑒
𝜋 𝑎2 + 𝑛2
−∞

Put 𝑥 = 0in the above series, We get



sinh 𝑎𝜋 𝑛
𝑎 + 𝑖𝑛
1= −1
𝜋 𝑎2 + 𝑛2
−∞

Equating real parts on both sides, we get



sinh 𝑎𝜋 𝑛
𝑎
1= −1
𝜋 𝑎2 + 𝑛2
−∞


𝜋 −1 𝑛
=
a sinh 𝑎𝜋 𝑎2 + 𝑛2
−∞

𝑙 − 𝑥, 0 < 𝑥 < 𝑙
6. Find the Fourier series for 𝑓 𝑥 =
0, 𝑙 < 𝑥 < 2𝑙

Hence deduce the sum to infinity of the series


1
2𝑛 + 1 2
𝑛=0

Solution: We know that



𝑎0 𝑛𝜋𝑥 𝑛𝜋𝑥
𝑓 𝑥 = + 𝑎𝑛 cos + 𝑏𝑛 sin
2 𝑙 𝑙
𝑛=1
Where

2𝑙 𝑙
1 1
𝑎0 = 𝑓 𝑥 𝑑𝑥 = (𝑙 − 𝑥)𝑑𝑥
𝑙 𝑙
0 0

𝑙
1 𝑥2 1 2 𝑙2
= 𝑙𝑥 − = 𝑙 −
𝑙 2 0
𝑙 2

𝑙
𝑎0 =
2
2𝑙
1 𝑛𝜋𝑥
𝑎𝑛 = 𝑓 𝑥 𝑐𝑜𝑠 𝑑𝑥
𝑙 𝑙
0

𝑙
1 𝑛𝜋𝑥
= 𝑙 − 𝑥 𝑐𝑜𝑠 𝑑𝑥
𝑙 𝑙
0

𝑙
𝑛𝜋𝑥 𝑛𝜋𝑥
1 𝑠𝑖𝑛 𝑙 𝑐𝑜𝑠
= 𝑙−𝑥 − (−1) − 𝑙
𝑙 𝑛𝜋 𝑛𝜋 2
𝑙 𝑙 0

1 −𝑐𝑜𝑠 𝑛𝜋 1 𝑙 𝑛+1
= 2 + 2 = −1 +1
𝑙 𝑛𝜋 𝑛𝜋 𝑛2 𝜋 2
𝑙 𝑙

𝑎𝑛 = 0, 𝑤𝑕𝑒𝑛 𝑛 𝑖𝑠 𝑒𝑣𝑒𝑛

2𝑙
𝑎𝑛 = , 𝑤𝑕𝑒𝑛 𝑛 𝑖𝑠 𝑜𝑑𝑑
𝑛2 𝜋 2
2𝑙
1 𝑛𝜋𝑥
𝑏𝑛 = 𝑓 𝑥 𝑠𝑖𝑛 𝑑𝑥
𝑙 𝑙
0

𝑙
1 𝑛𝜋𝑥
= 𝑙 − 𝑥 𝑠𝑖𝑛 𝑑𝑥
𝑙 𝑙
0

𝑙
𝑛𝜋𝑥 𝑛𝜋𝑥
1 𝑐𝑜𝑠 𝑙 𝑠𝑖𝑛
= 𝑙−𝑥 − 𝑛𝜋 − (−1) − 𝑙
𝑙 𝑛𝜋 2
𝑙 𝑙 0
1 𝑙 2 −1 𝑛
𝑙 −1 𝑛
𝑏𝑛 = =
𝑙 𝑛𝜋 𝑛𝜋

𝑙 2𝑙 1 𝜋𝑥 1 3𝜋𝑥 1 5𝜋𝑥
𝑓 𝑥 = − 2 2 𝑐𝑜𝑠 + 2 𝑐𝑜𝑠 + 2 𝑐𝑜𝑠 +⋯
4 𝜋 1 𝑙 3 𝑙 5 𝑙

𝑛
𝑙 −1 𝑛𝜋𝑥
+ 𝑠𝑖𝑛
𝜋 𝑛 𝑙
𝑛=1

𝑙 2𝑙 1 𝜋𝑥 1 3𝜋𝑥 1 5𝜋𝑥
𝑙−𝑥 = + 2 2 𝑐𝑜𝑠 + 2 𝑐𝑜𝑠 + 2 𝑐𝑜𝑠 +⋯
4 𝜋 1 𝑙 3 𝑙 5 𝑙

𝑛
𝑙 −1 𝑛𝜋𝑥
+ 𝑠𝑖𝑛
𝜋 𝑛 𝑙
𝑛=1

Here ‘0’ is a point of discontinuity which is an end point of the given interval. Therefore the
value of the Fourier series at 𝑥 = 0 is the average value of 𝑓 𝑥 at 𝑥 = 0 and 𝑥 = 2𝑙

Putting 𝑥 = 0, we get

𝑓 0 + 𝑓(2𝑙) 𝑙 2𝑙 1 1 1
= + 2 2+ 2+ 2+⋯
2 4 𝜋 1 3 5
2𝑙 1 1 1 𝑙−0 𝑙 𝑙
+ + + ⋯ = − =
𝜋 2 1 2 32 52 2 4 4

1 1 1 𝜋2
+ + + ⋯ =
12 32 52 8
7. Find the Fourier series for 𝑓 𝑥 = cos 𝑥 in − 𝜋, 𝜋

Solution:

𝑓 −𝑥 = |cos⁡
(−𝑥)| = | cos 𝑥| = 𝑓(𝑥)

∴ 𝑓 𝑥 is an even function.
𝜋 𝜋
2 2
𝑎0 = 𝑓 𝑥 𝑑𝑥 = | cos 𝑥| 𝑑𝑥
𝜋 𝜋
0 0
𝑦 𝑦
𝑦 = |cos 𝑥|

1 1
𝑦 = cos 𝑥

0 𝑥 0 𝑥
𝜋 𝜋 3𝜋 𝜋 𝜋 3𝜋
2 2 2 2

-1 -1

𝜋
2 𝜋 𝜋
2 cos 𝑥 𝑖𝑓 0 < 𝑥 <
= cos 𝑥 𝑑𝑥 + − cos 𝑥 𝑑𝑥 ∵ | cos 𝑥| = 2
𝜋 𝜋
0 𝜋 − cos 𝑥 𝑖𝑓 < 𝑥 < 𝜋
2 2
𝜋
2 𝜋 4
= sin 𝑥 02 − sin 𝑥 𝜋 =
𝜋 2 𝜋
𝜋
2
𝑎𝑛 = 𝑓 𝑥 𝑐𝑜𝑠 𝑛𝑥𝑑𝑥
𝜋
0

𝜋
2
= | cos 𝑥| 𝑐𝑜𝑠 𝑛𝑥𝑑𝑥
𝜋
0

𝜋
2 𝜋
2
= cos 𝑥 𝑐𝑜𝑠 𝑛𝑥 𝑑𝑥 + − cos 𝑥 cos 𝑛𝑥 𝑑𝑥
𝜋
0 𝜋
2

𝜋
cos 𝑥 𝑖𝑓 0 < 𝑥 <
∵ | cos 𝑥| = 2
𝜋
− cos 𝑥 𝑖𝑓 < 𝑥 < 𝜋
2
𝜋
2 𝜋
2 1 1
= cos(n + 1) 𝑥 + 𝑐𝑜𝑠 (𝑛 − 1)𝑥 𝑑𝑥 − cos(n + 1)𝑥 cos(n − 1)𝑥] 𝑑𝑥
𝜋 2 2
0 𝜋
2

𝜋
𝜋
1 sin(n + 1) 𝑥 sin(n − 1) 𝑥 2 1 sin(n + 1) 𝑥 sin(n − 1) 𝑥
= + − +
𝜋 𝑛+1 𝑛−1 0 𝜋 𝑛+1 𝑛−1 𝜋
2
𝜋 𝜋
2 sin(n + 1) 2 sin(n − 1) 2
= +
𝜋 𝑛+1 𝑛−1

𝜋 𝜋 𝜋 𝜋 𝜋 𝜋 𝜋 𝜋
2 sin n 2 cos 2 + cos 𝑛 2 sin 2 sin n 2 cos 2 − cos 𝑛 2 sin 2
= +
𝜋 𝑛+1 𝑛−1

𝜋
2 cos 𝑛 2 𝑛 − 1 − 𝑛 − 1
=
𝜋 𝑛2 − 1

4 𝜋
𝑎𝑛 = − cos 𝑛 , 𝑛 ≠ 1.
𝜋 𝑛2 − 1 2

when 𝑛 = 1, we get
𝜋
2
𝑎1 = 𝑓 𝑥 𝑐𝑜𝑠 𝑥𝑑𝑥
𝜋
0

𝜋
2
= | cos 𝑥| 𝑐𝑜𝑠 𝑥𝑑𝑥
𝜋
0

𝜋
2 𝜋
2
= cos 𝑥 𝑐𝑜𝑠 𝑥 𝑑𝑥 + − cos 𝑥 cos 𝑥 𝑑𝑥
𝜋
0 𝜋
2

𝜋
2 𝜋
2 1 1
= cos2 𝑥 + 1 𝑑𝑥 − cos2 𝑥 + 1]𝑑𝑥
𝜋 2 2
0 𝜋
2

𝜋
𝜋
1 sin 2𝑥 2 sin 2𝑥
= +𝑥 − +𝑥
𝜋 2 0 2 𝜋
2

1 𝜋 𝜋
𝑎1 = −𝜋+ = 0
𝜋 2 2

𝑏𝑛 = 0,

Then the Fourier series is reduced to



𝑎0
𝑓 𝑥 = + 𝑎𝑛 cos 𝑛𝑥
2
𝑛=1

∞ 𝜋
2 4 cos 𝑛 2
𝑓 𝑥 = − cos 𝑛𝑥
𝜋 𝜋 𝑛2 − 1
𝑛 =2

8. Find Half Range Cosine Series 𝑓(𝑥) = 𝑥 in 0 < 𝑥 < 

Solution: We know that



𝑎0
𝑓 𝑥 = + 𝑎𝑛 cos 𝑛𝑥
2
𝑛 =1

where
𝜋 𝜋
2 2
𝑎0 = 𝑓 𝑥 𝑑𝑥 = 𝑥 𝑑𝑥
𝜋 𝜋
0 0

𝜋
2 𝑥2
= =𝜋
𝜋 2 0

𝜋
2
𝑎𝑛 = 𝑓 𝑥 cos 𝑛𝑥𝑑𝑥
𝜋
0

𝜋
2
= 𝑥 cos 𝑛𝑥𝑑𝑥
𝜋
0

𝜋
2 sin 𝑛𝑥 cos 𝑛𝑥
= 𝑥 −1 −
𝜋 n n2 0

2 2
2 1 n+1
1
= (−1) −
𝜋 𝑛 𝑛

2
= (−1)n+1 − 1
𝜋𝑛2
When 𝑛 is odd

4
𝑎𝑛 = −
𝜋𝑛2

When 𝑛 is even

𝑎𝑛 = 0

𝜋 4 1
𝑓 𝑥 = − cos 𝑛𝑥
2 𝜋 𝑛2
𝑛=1

𝜋 4 1
𝑥= − cos 𝑥 + 2 cos 3𝑥 + ⋯
2 𝜋 3

9. Find the Fourier series of periodicity 2 𝜋 for (𝑥) = 𝑥 2 , −𝜋 < 𝑥 < 𝜋 and deduce

1 1 1 𝜋4
+ + … =
14 24 34 90

Solution:

𝑓 −𝑥 = (−𝑥)2 = 𝑥 2 = 𝑓(𝑥)

∴ 𝑓 𝑥 is an even function.
𝜋 𝜋
2 2
𝑎0 = 𝑓 𝑥 𝑑𝑥 = 𝑥 2 𝑑𝑥
𝜋 𝜋
0 0

𝜋
2 𝑥3 2𝜋 2
= =
𝜋 3 0
3
𝜋
2
𝑎𝑛 = 𝑓 𝑥 𝑐𝑜𝑠 𝑛𝑥𝑑𝑥
𝜋
0

𝜋
2
= 𝑥 2 𝑐𝑜𝑠 𝑛𝑥𝑑𝑥
𝜋
0

𝜋
2 2 sin 𝑛𝑥 cos 𝑛𝑥 sin 𝑛𝑥
= 𝑥 − 2𝑥 − 2
+2 −
𝜋 𝑛 𝑛 𝑛3 0

𝑛
2 2 𝜋cos 𝑛𝜋 4 −1
= − = −
𝜋 𝑛2 𝑛2

𝑏𝑛 = 0

𝑎0
𝑓 𝑥 = + 𝑎𝑛 cos 𝑛𝑥
2
𝑛=1


𝜋2 −1 𝑛
𝑓 𝑥 = −4 cos 𝑛𝑥
3 𝑛2
𝑛 =1

By Parseval’s identity
𝜋 ∞
2 2
𝑎02
𝑓 𝑥 𝑑𝑥 = + 𝑎𝑛2
𝜋 2
0 𝑛=1

2
𝜋 2𝜋2 ∞ 𝑛 2
2 3 4 −1
𝑥2 2 𝑑𝑥 = + −
𝜋 2 𝑛2
0 𝑛=1

𝜋 ∞
2 𝑥5 2𝜋4 1 2
= + 16
𝜋 5 0
9 𝑛2
𝑛=1

2𝜋4 2𝜋4 1
= + 16
5 9 𝑛4
𝑛=1

1 2𝜋 4 2𝜋 4
16 = −
𝑛4 5 9
𝑛=1

1 8𝜋 4
16 =
𝑛4 45
𝑛=1

1 𝜋4
=
𝑛4 90
𝑛 =1

𝑙 + 𝑥 , −𝑙 < 𝑥 < 0
10. Find the Fourier series for 𝑓 𝑥 = and deduce that
𝑙 − 𝑥, 0 < 𝑥 < 𝑙

1 1 𝜋2
+ + …=
12 32 8

Solution:

𝑙 − 𝑥 , −𝑙 < −𝑥 < 0 𝑙 −𝑥 ,𝑙 > 𝑥 > 0


𝑓 −𝑥 = = = 𝑓(𝑥)
𝑙 + 𝑥, 0 < −𝑥 < 𝑙 𝑙 + 𝑥, 0 > 𝑥 > −𝑙

∴ 𝑓 𝑥 is an even function.
𝑙 𝑙
2 2
𝑎0 = 𝑓 𝑥 𝑑𝑥 = 𝑙 − 𝑥 𝑑𝑥
𝑙 𝑙
0 0

𝑙
2 𝑥2 2 2 𝑙2
= 𝑙𝑥 − = 𝑙 −
𝑙 2 0
𝑙 2
𝑎0 = 𝑙
𝑙
2 𝑛𝜋𝑥
𝑎𝑛 = 𝑓 𝑥 𝑐𝑜𝑠 𝑑𝑥
𝜋 𝑙
0

𝑙
2 𝑛𝜋𝑥
= 𝑙 − 𝑥 𝑐𝑜𝑠 𝑑𝑥
𝑙 𝑙
0

𝑙
𝑛𝜋𝑥 𝑛𝜋𝑥
2 sin 𝑙 cos 𝑙
= 𝑙−𝑥 𝑛𝜋 − −1 −
𝑙 𝑛𝜋 2
𝑙 𝑙 0

2 cos 𝑛𝜋 1
= − 2 + 2
𝑙 𝑛𝜋 𝑛𝜋
𝑙 𝑙
2𝑙 𝑛
= 1 − −1
𝑛2 𝜋2

𝑎𝑛 = 0, 𝑤𝑕𝑒𝑛 𝑛 𝑖𝑠 𝑒𝑣𝑒𝑛

4𝑙
𝑎𝑛 = , 𝑤𝑕𝑒𝑛 𝑛 𝑖𝑠 𝑜𝑑𝑑
𝑛2 𝜋2
𝑏𝑛 = 0

𝑎0 𝑛𝜋𝑥
𝑓 𝑥 = + 𝑎𝑛 cos
2 𝑙
𝑛=1


𝑙 4𝑙 1 𝑛𝜋𝑥
𝑓 𝑥 = + 2
cos
2 𝜋2 𝑛 𝑙
𝑛 =1,3,5,…

Putting 𝑥 = 0, we get

𝑙 4𝑙 1
𝑓 0 = +
2 𝜋2 𝑛2
𝑛=1,3,5,…

Here ‘0’ is a point of discontinuity, then

𝑓 0 + 0 + 𝑓(0 − 0)
𝑓 0 =
2
𝑙+𝑙
𝑓 0 = =𝑙
2

𝑙 4𝑙 1
𝑙= +
2 𝜋2 𝑛2
𝑛 =1,3,5,…


4𝑙 1 𝑙 𝑙
2
=𝑙− =
𝜋2 𝑛 2 2
𝑛 =1,3,5,…

1 1 𝜋2
∴ 2+ 2 + …=
1 3 8

11. Find Half Range Fourier Sine series 𝑓(𝑥) = 𝑥 𝑠𝑖𝑛𝑥 in 0 < 𝑥 < 𝜋.

Solution: We know that


𝑓 𝑥 = 𝑏𝑛 sin 𝑛𝑥
𝑛 =1

Where
𝜋
2
𝑏𝑛 = 𝑓 𝑥 𝑠𝑖𝑛 𝑛𝑥𝑑𝑥
𝜋
0

𝜋
2
= 𝑥 sin 𝑥 𝑠𝑖𝑛 𝑛𝑥𝑑𝑥
𝜋
0

𝜋
1
= 𝑥 cos 𝑛 − 1 𝑥 − cos 𝑛 + 1 𝑥 𝑑𝑥
𝜋
0

𝜋
1 sin 𝑛 − 1 𝑥 − cos 𝑛 − 1 𝑥
= 𝑥 − 1.
𝜋 𝑛−1 𝑛−1 2 0

𝜋
1 sin 𝑛 + 1 𝑥 − cos 𝑛 + 1 𝑥
− 𝑥 − 1.
𝜋 𝑛+1 𝑛+1 2 0

1 cos 𝑛 − 1 𝜋 1 cos 𝑛 + 1 𝜋 1
= 2
− 2
− 2
+ 2
𝜋 𝑛−1 𝑛−1 𝑛+1 𝑛+1

1 −1 𝑛−1 1 −1 𝑛+1 1
= 2
− 2
− 2
+ 2
𝜋 𝑛−1 𝑛−1 𝑛+1 𝑛+1
2 𝑛
1 − 𝑛+1 −1 + 𝑛−1 2+ 𝑛−1 2
−1 𝑛
− 𝑛+1 2
=
𝜋 𝑛2 − 1 2

1 − 𝑛2 + 2𝑛 + 1 −1 𝑛
+ 𝑛2 − 2𝑛 + 1 + 𝑛2 − 2𝑛 + 1 −1 𝑛
− 𝑛2 + 2𝑛 + 1
=
𝜋 𝑛2 − 1 2

1 −4𝑛 −1 𝑛 − 4𝑛
= ,
𝜋 𝑛2 − 1 2

𝑛≠1

0 𝑤𝑕𝑒𝑛 𝑛 ≠ 1 𝑎𝑛𝑑 𝑛 𝑖𝑠 𝑜𝑑𝑑


𝑏𝑛 = −8𝑛
𝑤𝑕𝑒𝑛 𝑛 ≠ 1 𝑎𝑛𝑑 𝑛 𝑖𝑠 𝑒𝑣𝑒𝑛
𝜋 𝑛2 − 1 2

When 𝑛 = 1
𝜋
2
𝑏1 = 𝑓 𝑥 𝑠𝑖𝑛 𝑥𝑑𝑥
𝜋
0

𝜋
2
= 𝑥 sin 𝑥 𝑠𝑖𝑛 𝑥𝑑𝑥
𝜋
0

𝜋
1
= 𝑥 1 − cos 2𝑥 𝑑𝑥
𝜋
0

𝜋
1 sin 2𝑥 𝑥 2 cos 2𝑥
= 𝑥 𝑥− − 1. −
𝜋 2 2 22 0

1 2 𝜋2 𝜋
= 𝜋 − =
𝜋 2 2

𝜋 8 𝑛
𝑓 𝑥 = sin 𝑥 − sin 𝑛𝑥
2 𝜋 𝑛2 −1 2
𝑛 =2,4,6,…

12. Find Half Range Fourier Sine series 𝑓(𝑥) = 𝑥 ( 𝜋 − 𝑥) in 0 < 𝑥 < 𝜋 and Prove that

1 1 1 𝜋3
− + − … =
13 33 53 32

Solution: We know that


𝑓 𝑥 = 𝑏𝑛 sin 𝑛𝑥
𝑛 =1
Where
𝜋
2
𝑏𝑛 = 𝑓 𝑥 𝑠𝑖𝑛 𝑛𝑥𝑑𝑥
𝜋
0

𝜋
2
= 𝑥 ( 𝜋 − 𝑥) 𝑠𝑖𝑛 𝑛𝑥𝑑𝑥
𝜋
0

𝜋
2
= 𝜋𝑥 − 𝑥 2 sin 𝑛𝑥 𝑑𝑥
𝜋
0

𝜋
2 2
cos 𝑛𝑥 sin 𝑛𝑥 cos 𝑛𝑥
= 𝜋𝑥 − 𝑥 − − 𝜋 − 2𝑥 − + −2
𝜋 𝑛 𝑛2 𝑛3 0

4 cos 𝑛𝜋 − 1 4 (−1)𝑛 − 1
=− = −
𝜋 𝑛3 𝜋 𝑛3
8
𝑏𝑛 = 𝜋𝑛3 𝑤𝑕𝑒𝑛 𝑛 𝑖𝑠 𝑜𝑑𝑑
0 𝑤𝑕𝑒𝑛 𝑛 𝑖𝑠 𝑒𝑣𝑒𝑛

8 1
𝑓 𝑥 = sin 𝑛𝑥
𝜋 𝑛3
𝑛=1,3,5,…

𝜋
Put 𝑥 = 2


𝜋 𝜋 8 1 𝜋
𝜋− = sin 𝑛
2 2 𝜋 𝑛3 2
𝑛 =1,3,5,…


1 𝜋 𝜋3
sin 𝑛 =
𝑛3 2 32
𝑛 =1,3,5,…

1 𝜋 1 3𝜋 1 5𝜋 𝜋3
sin + sin + sin + … =
13 2 33 2 53 2 32
1 1 1 𝜋3
− + − …=
13 33 53 32
1, 0 < 𝑥 < 𝜋
13. Find the Fourier series for 𝑓 𝑥 =
2, 𝜋 < 𝑥 < 2𝜋

Solution: We know that



𝑎0
𝑓 𝑥 = + 𝑎𝑛 cos 𝑛𝑥 + 𝑏𝑛 sin 𝑛𝑥
2
𝑛 =1

Where

2𝜋 𝜋 2𝜋
1
𝑎0 = 𝑓 𝑥 𝑑𝑥 = 𝑓 𝑥 𝑑𝑥 + 𝑓 𝑥 𝑑𝑥
𝜋
0 0 𝜋

𝜋 2𝜋
1 1
= 1𝑑𝑥 + 2 𝑑𝑥 = 𝜋 + 4𝜋 − 2𝜋 = 3
𝜋 𝜋
0 𝜋

2𝜋
1
𝑎𝑛 = 𝑓 𝑥 𝑐𝑜𝑠 𝑛𝑥𝑑𝑥
𝜋
0

𝜋 2𝜋
1
= 𝑓 𝑥 cos 𝑛𝑥 𝑑𝑥 + 𝑓 𝑥 cos 𝑛𝑥 𝑑𝑥
𝜋
0 𝜋

𝜋 2𝜋
1
= cos 𝑛𝑥 𝑑𝑥 + 2 cos 𝑛𝑥 𝑑𝑥
𝜋
0 𝜋

𝜋 2𝜋
1 sin 𝑛𝑥 sin 𝑛𝑥
𝑎𝑛 = +2 =0
𝜋 𝑛 0 𝑛 𝜋

2𝜋
1
𝑏𝑛 = 𝑓 𝑥 𝑠𝑖𝑛 𝑛𝑥𝑑𝑥
𝜋
0

𝜋 2𝜋
1
= 𝑓 𝑥 sin 𝑛𝑥 𝑑𝑥 + 𝑓 𝑥 sin 𝑛𝑥 𝑑𝑥
𝜋
0 𝜋

𝜋 2𝜋
1
= sin 𝑛𝑥 𝑑𝑥 + 2 sin 𝑛𝑥 𝑑𝑥
𝜋
0 𝜋

1 cos 𝑛𝑥 𝜋 cos 𝑛𝑥 2𝜋
= − +2 −
𝜋 𝑛 0 𝑛 𝜋

1 cos 𝑛𝜋 − 1 cos 2𝑛𝜋 − cos 𝑛𝜋 1 𝑛 𝑛


= − +2 − =− −1 − 1 + 2 − 2 −1
𝜋 𝑛 𝑛 𝑛𝜋
1 𝑛
𝑏𝑛 = − 1 − −1
𝑛𝜋
2
= − 𝑛𝜋 𝑤𝑕𝑒𝑛 𝑛 𝑖𝑠 𝑜𝑑𝑑
0 𝑤𝑕𝑒𝑛 𝑛 𝑖𝑠 𝑒𝑣𝑒𝑛

3 2 1
𝑓 𝑥 = − sin 𝑛𝑥
2 𝜋 𝑛
𝑛 =1,3,5,…

𝑙
𝑥 𝑓𝑜𝑟 0 < 𝑥 < 2
14. Find the Fourier sine series for 𝑓 𝑥 = 𝑙
𝑙−𝑥 𝑓𝑜𝑟 <𝑥<𝑙
2

Solution: We know that



𝑛𝜋𝑥
𝑓 𝑥 = 𝑏𝑛 sin
𝑙
𝑛 =1

Where
𝑙
2 𝑛𝜋𝑥
𝑏𝑛 = 𝑓 𝑥 sin 𝑑𝑥
𝑙 𝑙
0

𝑙
2 𝑙
2 𝑛𝜋𝑥 𝑛𝜋𝑥
= 𝑓 𝑥 sin 𝑑𝑥 + 𝑓 𝑥 sin 𝑑𝑥
𝑙 𝑙 𝑙
0 𝑙
2

𝑙
2 𝑙
2 𝑛𝜋𝑥 𝑛𝜋𝑥
= 𝑥 sin 𝑑𝑥 + (𝑙 − 𝑥) sin 𝑑𝑥
𝑙 𝑙 𝑙
0 𝑙
2

𝑙
𝑛𝜋𝑥 𝑛𝜋𝑥 2
2 cos 𝑙 sin 𝑙
= 𝑥 − 𝑛𝜋 −1 −
𝑙 𝑛𝜋 2
𝑙 𝑙 0

𝑙
𝑛𝜋𝑥 𝑛𝜋𝑥
2 cos 𝑙 sin 𝑙
+ 𝑙−𝑥 − 𝑛𝜋 − (−1) −
𝑙 𝑛𝜋 2
𝑙 𝑙 𝑙
2
𝑙 𝑙
𝑛𝜋 2 𝑛𝜋 2
2 𝑙 cos sin 𝑙
= − 𝑛𝜋𝑙 −1 − −0
𝑙 2 𝑛𝜋 2
𝑙 𝑙

𝑙 𝑙
𝑛𝜋 2 𝑛𝜋 2
2 𝑙 cos sin 𝑙
+ 0− 𝑙− − 𝑛𝜋𝑙 + (−1) −
𝑙 2 𝑛𝜋 2
𝑙 𝑙

𝑛𝜋 𝑛𝜋 𝑛𝜋 𝑛𝜋
2 cos 2 𝑙 2 𝑠𝑖𝑛 2 cos 2 𝑙 2 𝑠𝑖𝑛 2
= − + + +
𝑙 2𝑛𝜋 𝑛2 𝜋 2 2𝑛𝜋 𝑛2 𝜋 2

𝑛𝜋
4𝑙 𝑠𝑖𝑛
𝑏𝑛 = 2
𝑛2 𝜋 2
∞ 𝑛𝜋
4𝑙 𝑠𝑖𝑛 2 𝑛𝜋𝑥
𝑓 𝑥 = 2 sin
𝜋 𝑛2 𝑙
𝑛 =1

0 𝑓𝑜𝑟 − 1 < 𝑥 < 0


15. Find the Fourier series for 𝑓 𝑥 =
1 𝑓𝑜𝑟 0 < 𝑥 < 1

Solution: We know that



𝑎0 𝑛𝜋𝑥 𝑛𝜋𝑥
𝑓 𝑥 = + 𝑎𝑛 cos + 𝑏𝑛 sin
2 𝑙 𝑙
𝑛=1

Here 𝑙 = 1 then

𝑎0
𝑓 𝑥 = + 𝑎𝑛 cos 𝑛𝜋𝑥 + 𝑏𝑛 sin 𝑛𝜋𝑥
2
𝑛=1

Where
𝑙 1
1 1
𝑎0 = 𝑓 𝑥 𝑑𝑥 = 𝑓 𝑥 𝑑𝑥
𝑙 1
−𝑙 −1

0 1

= 𝑓 𝑥 𝑑𝑥 + 𝑓 𝑥 𝑑𝑥
−1 0

0 1
1
= 0 𝑑𝑥 + 1 𝑑𝑥 = 𝑥 0 =1
−1 0
𝑙 1
1 𝑛𝜋𝑥 1
𝑎𝑛 = 𝑓 𝑥 cos 𝑑𝑥 = 𝑓 𝑥 cos 𝑛𝜋𝑥 𝑑𝑥
𝑙 𝑙 1
−𝑙 −1

0 1

= 𝑓 𝑥 cos 𝑛𝜋𝑥 𝑑𝑥 + 𝑓 𝑥 cos 𝑛𝜋𝑥 𝑑𝑥


−1 0

0 1
1
sin 𝑛𝜋𝑥
= 0. cos 𝑛𝜋𝑥 𝑑𝑥 + 1. cos 𝑛𝜋𝑥 𝑑𝑥 = = 0.
𝑛𝜋 0
−1 0

𝑙 1
1 𝑛𝜋𝑥 1
𝑏𝑛 = 𝑓 𝑥 sin 𝑑𝑥 = 𝑓 𝑥 sin 𝑛𝜋𝑥 𝑑𝑥
𝑙 𝑙 1
−𝑙 −1

0 1

= 𝑓 𝑥 sin 𝑛𝜋𝑥 𝑑𝑥 + 𝑓 𝑥 sin 𝑛𝜋𝑥 𝑑𝑥


−1 0

0 1
cos 𝑛𝜋𝑥 1
= 0. sin 𝑛𝜋𝑥 𝑑𝑥 + 1. sin 𝑛𝜋𝑥 𝑑𝑥 = −
𝑛𝜋 0
−1 0

cos 𝑛𝜋 cos 0 1 𝑛
=− + = 1 − −1
𝑛𝜋 𝑛𝜋 𝑛𝜋
0 𝑤𝑕𝑒𝑛 𝑛 𝑖𝑠 𝑒𝑣𝑒𝑛
𝑏𝑛 = 2
𝑤𝑕𝑒𝑛 𝑛 𝑖𝑠 𝑜𝑑𝑑
𝑛𝜋

1 2 1
𝑓 𝑥 = + sin 𝑛𝜋𝑥
2 𝜋 𝑛
𝑛 =1,3,5,…

16. Find Half Range Fourier Sine series 𝑓 𝑥 = 𝑥 cos 𝑥 in 0 < 𝑥 < 𝜋.

Solution: We know that


𝑓 𝑥 = 𝑏𝑛 sin 𝑛𝑥
𝑛 =1

where
𝜋
2
𝑏𝑛 = 𝑓 𝑥 sin 𝑛𝑥𝑑𝑥
𝜋
0
𝜋
2
= 𝑥𝑐𝑜𝑠 𝑥 sin 𝑛𝑥𝑑𝑥
𝜋
0

𝜋
1
= 𝑥 sin 𝑛 − 1 𝑥 + sin 𝑛 + 1 𝑥 𝑑𝑥
𝜋
0

𝜋
1 cos 𝑛 − 1 𝑥 − sin 𝑛 − 1 𝑥
= 𝑥 − − 1.
𝜋 𝑛−1 𝑛−1 2 0

𝜋
1 cos 𝑛 + 1 𝑥 − sin 𝑛 + 1 𝑥
+ 𝑥 − − 1.
𝜋 𝑛+1 𝑛+1 2 0

1 cos 𝑛 − 1 𝜋 cos 𝑛 + 1 𝜋
= 𝜋 − +𝜋 −
𝜋 𝑛−1 𝑛+1

−1 𝑛−1 −1 𝑛+1
=− +
𝑛−1 𝑛+1

−(𝑛 + 1) −1 𝑛 − (𝑛 − 1) −1 𝑛
=− ,𝑛 ≠ 1
𝑛2 − 1

2𝑛 −1 𝑛
𝑏𝑛 = 2
𝑛 −1

When 𝑛 = 1
𝜋
2
𝑏1 = 𝑓 𝑥 𝑠𝑖𝑛 𝑥𝑑𝑥
𝜋
0

𝜋
2
= 𝑥 cos 𝑥 𝑠𝑖𝑛 𝑥𝑑𝑥
𝜋
0

𝜋
1
= 𝑥 sin 2𝑥 𝑑𝑥
𝜋
0

𝜋
1 cos 2𝑥 sin 2𝑥
= 𝑥 − − 1. − 2
𝜋 2 2 0

1 1
= −𝜋 cos 2𝜋 = −
2𝜋 2

1 𝑛 −1 𝑛
𝑓 𝑥 = − sin 𝑥 + 2 sin 𝑛𝑥
2 𝑛2 − 1
𝑛 =2
17. Find the Fourier series for 𝑓 𝑥 = 1 + 𝑥 + 𝑥 2 in ( − 𝜋, 𝜋) and deduce that

1 1 1 𝜋2
+ + …=
12 22 32 6

Solution: We know that



𝑎0
𝑓 𝑥 = + 𝑎𝑛 cos 𝑛𝑥 + 𝑏𝑛 sin 𝑛𝑥
2
𝑛 =1

Where

𝜋 𝜋
1 1
𝑎0 = 𝑓 𝑥 𝑑𝑥 = 1 + 𝑥 + 𝑥 2 𝑑𝑥
𝜋 𝜋
−𝜋 −𝜋

𝜋
1 𝑥2 𝑥3 1 𝜋2 𝜋3 −𝜋 2
−𝜋 3
= 𝑥+ + = 𝜋+ + − −𝜋 + +
𝜋 2 3 −𝜋
𝜋 2 3 2 3

2 𝜋3 2𝜋 2
= 𝜋+ =2+
𝜋 3 3
𝜋 𝜋
1 1
𝑎𝑛 = 𝑓 𝑥 cos 𝑛𝑥 𝑑𝑥 = 1 + 𝑥 + 𝑥 2 cos 𝑛𝑥 𝑑𝑥
𝜋 𝜋
−𝜋 −𝜋

𝜋 𝜋
2
= 𝑐𝑜𝑠 𝑛𝑥𝑑𝑥 + 𝑥 2 𝑐𝑜𝑠 𝑛𝑥𝑑𝑥
𝜋
0 0

𝜋
2 sin 𝑛𝑥 sin 𝑛𝑥 cos 𝑛𝑥 sin 𝑛𝑥
= + 𝑥2 −2𝑥 − + 2 −
𝜋 𝑛 𝑛 𝑛2 𝑛3 0

4(−1)𝑛
=
𝑛2
2𝜋 𝜋
1 1
𝑏𝑛 = 𝑓 𝑥 𝑠𝑖𝑛 𝑛𝑥𝑑𝑥 = 1 + 𝑥 + 𝑥 2 sin 𝑛𝑥 𝑑𝑥
𝜋 𝜋
0 −𝜋

𝜋
2
= 𝑥𝑠𝑖𝑛 𝑛𝑥𝑑𝑥
𝜋
0

𝜋
2 cos 𝑛𝑥 sin 𝑛𝑥 −2(−1)𝑛
= 𝑥 − −1 − =
𝜋 𝑛 𝑛2 0 𝑛

𝜋2 (−1)𝑛 (−1)𝑛
𝑓 𝑥 =1+ +4 cos 𝑛𝑥 − sin 𝑛𝑥
3 𝑛2 2𝑛
𝑛=1

Here ‘𝜋’ is a point of discontinuity which is an end point of the given interval. Therefore the
value of the Fourier series at 𝑥 = 𝜋 is the average value of 𝑓 𝑥 at 𝑥 = −𝜋 and 𝑥 = 𝜋

Putting 𝑥 = 𝜋, we get

𝑓 −𝜋 + 𝑓(𝜋) 𝜋2 (−1)𝑛 (−1)𝑛
=1+ +4 cos 𝑛𝜋 − sin 𝑛𝜋
2 3 𝑛2 2𝑛
𝑛=1


1 − 𝜋 + 𝜋2 + 1 + 𝜋 + 𝜋2 𝜋2 (−1)𝑛
= 1+ +4 (−1)𝑛
2 3 𝑛2
𝑛=1


2
𝜋2 (−1)𝑛
1+𝜋 =1+ +4 (−1)𝑛
3 𝑛2
𝑛 =1


1 2
𝜋 2 2𝜋 2
4 = 𝜋 − =
𝑛2 3 3
𝑛=1


1 𝜋2
=
𝑛2 6
𝑛 =1
Harmonic analysis:

18. Find the Fourier series expansion of period 2𝜋 for the function 𝑦 = 𝑓(𝑥) which is
defined in (0, 2𝜋) by means of the table of values given below. Find the series upto the third
harmonic.

𝜋 𝟐𝜋 𝟒𝜋 𝟓𝜋
𝒙 0 𝜋 𝟐𝜋
𝟑 𝟑 𝟑 𝟑

𝒇(𝒙) 1.0 1.4 1.9 1.7 1.5 1.2 1.0

Solution: Let 𝑦 = 𝑓(𝑥)

Here 𝑛 = 6 Since the last value of 𝑦 is repetition of the first, only the first six values will be
used.

We know that
𝑎0
𝑓 𝑥 = + 𝑎1 cos 𝑥 + 𝑎2 cos 2𝑥 + 𝑎3 cos 3𝑥 + 𝑏1 sin 𝑥 + 𝑏2 sin 2𝑥 + 𝑏3 sin 3𝑥
2
Where

2 2 1
𝑎0 = 𝑦= 𝑦= 8.7 = 2.9
𝑛 6 3
2 2 1
𝑎1 = 𝑦 cos 𝑥 = 𝑦 cos 𝑥 = −1.1 = −2.53
𝑛 6 3
2 2 1
𝑎2 = 𝑦 cos 2𝑥 = 𝑦 cos 2𝑥 = −0.3 = −0.1
𝑛 6 3
2 2 1
𝑎3 = 𝑦 cos 3𝑥 = 𝑦 cos 3𝑥 = 0.1 = 0.033
𝑛 6 3
2 2 1
𝑏1 = 𝑦 sin 𝑥 = 𝑦 sin 𝑥 = 0.524 = 0.175
𝑛 6 3
2 2 1
𝑏2 = 𝑦 sin 2𝑥 = 𝑦 sin 2𝑥 = −0.178 = −0.059
𝑛 6 3
2 2 1
𝑏3 = 𝑦 sin 3𝑥 = 𝑦 sin 3𝑥 = 0 =0
𝑛 6 3

𝑓 𝑥 = 1.45 − 2.53 cos 𝑥 − 0.1 cos 2𝑥 + 0.033 cos 3𝑥 + 0.175 sin 𝑥 − 0.059 sin 2𝑥
𝒙 𝒚 𝒄𝒐𝒔 𝒙 𝒄𝒐𝒔 𝟐𝒙 𝒄𝒐𝒔 𝟑𝒙 𝒔𝒊𝒏 𝒙 𝒔𝒊𝒏 𝟐𝒙 𝒔𝒊𝒏 𝟑𝒙 𝒚𝒄𝒐𝒔 𝒙 𝒚𝒄𝒐𝒔 𝟐𝒙 𝒚𝒄𝒐𝒔 𝟑𝒙 𝒚𝒔𝒊𝒏 𝒙 𝒚𝒔𝒊𝒏 𝟐𝒙 𝒚𝒔𝒊𝒏 𝟑𝒙

0 1.0 1 1 1 0 0 0 1 1 1 0 0 0

𝜋
1.4 0.5 -0.5 -1 0.866 0.866 0 0.7 -0.7 -1.4 1.212 1.212 0
𝟑

𝟐𝜋
1.9 -0.5 -0.5 1 0.866 -0.866 0 -0.95 -0.95 1.9 1.65 -1.65 0
𝟑

𝜋 1.7 -1 1 -1 0 0 0 -1.7 1.7 -1.7 0 0 0

𝟒𝜋
1.5 -0.5 -0.5 1 -0.866 0.866 0 -0.75 -0.75 1.5 -1.299 1.299 0
𝟑

𝟓𝜋
1.2 0.5 -0.5 -1 -0.866 -0.866 0 0.6 -0.6 -1.2 -1.039 -1.039 0
𝟑

SUM 8.7 -1.1 -0.3 0.1 0.524 -0.178 0


19. Find the Fourier series as far as the second harmonic to represent the function given in
the following data

𝒙 0 1 2 3 4 5

𝒇(𝒙) 9 18 24 28 26 20

Solution: Let 𝑦 = 𝑓(𝑥), Here 𝑛 = 6 and 2𝑙 = 6 𝑙=3

𝑎0 𝜋𝑥 𝜋𝑥 𝜋𝑥 𝜋𝑥
𝑓 𝑥 = + 𝑎1 cos + 𝑎2 cos 2 + 𝑏1 sin + 𝑏2 sin 2
2 𝑙 𝑙 𝑙 𝑙
𝑎0 𝜋𝑥 𝜋𝑥 𝜋𝑥 𝜋𝑥
𝑓 𝑥 = + 𝑎1 cos + 𝑎2 cos 2 + 𝑏1 sin + 𝑏2 sin 2
2 3 3 3 3
Where

2 2 1
𝑎0 = 𝑦= 𝑦= 125 = 41.67
𝑛 6 3
2 𝜋𝑥 2 𝜋𝑥 1
𝑎1 = 𝑦 cos = 𝑦 cos = −25 = −8.33
𝑛 3 6 3 3
2 𝜋𝑥 2 𝜋𝑥 1
𝑎2 = 𝑦 cos 2 = 𝑦 cos 2 = −7 = −2.33
𝑛 3 6 3 3
2 𝜋𝑥 2 𝜋𝑥 1
𝑏1 = 𝑦 sin = 𝑦 sin = −3.464 = −1.155
𝑛 3 6 3 3
2 𝜋𝑥 2 𝜋𝑥 1
𝑏2 = 𝑦 sin 2 = 𝑦 sin 2 = 0 =0
𝑛 3 6 3 3

𝜋𝑥 𝜋𝑥 𝜋𝑥
𝑓 𝑥 = 20.83 − 8.33 cos − 2.33 cos 2 − 1.155 sin
3 3 3
𝝅𝒙 𝝅𝒙 𝝅𝒙 𝝅𝒙 𝝅𝒙 𝝅𝒙 𝝅𝒙 𝝅𝒙
𝒙 𝒚 𝒄𝒐𝒔 𝒄𝒐𝒔 𝟐 𝒔𝒊𝒏 𝒔𝒊𝒏 𝟐 𝒚𝒄𝒐𝒔 𝒚𝒄𝒐𝒔 𝟐 𝒚𝒔𝒊𝒏 𝒚𝒔𝒊𝒏 𝟐
𝟑 𝟑 𝟑 𝟑 𝟑 𝟑 𝟑 𝟑

0 9 1 1 0 0 9 9 0 0

𝟏 18 0.5 -0.5 0.866 0.866 9 -9 15.588 15.588

𝟐 24 -0.5 -0.5 0.866 -0.866 -12 -12 20.784 -20.784

𝟑 28 -1 1 0 0 -28 28 0 0

𝟒 26 -0.5 -0.5 -0.866 0.866 -13 -13 -22.516 22.516

𝟓 20 0.5 -0.5 -0.866 -0.866 10 -10 -17.32 -17.32

SUM 125 -25 -7 -3.464 0


UNIT-II

FOURIER TRANSFORMS

Fourier integral theorem.

If is a given function 𝑓(𝑥) defined in −𝑙, 𝑙 and satisfies Dirichlet’s conditions, then
∞ ∞
1
𝑓 𝑥 = 𝑓 𝑡 cos 𝜆 𝑡 − 𝑥 𝑑𝑥 𝑑𝜆
𝜋
0 −∞

Infinite Fourier transform and its inverse:



1
𝐹 𝑓 𝑥 = 𝑓 𝑥 𝑒 𝑖𝑠𝑥 𝑑𝑥
2𝜋
−∞


−1
1
𝐹 𝐹 𝑠 = 𝑓 𝑥 𝑒 𝑖𝑠𝑥 𝑑𝑥
2𝜋
−∞

𝐹𝑓 𝑥 𝑎𝑛𝑑 𝐹 −1 𝐹 𝑠 are Fourier transform pairs.

Infinite Fourier cosine transform and its inverse

The infinite Fourier cosine transform is given by



2
𝐹𝑐 𝑓 𝑥 = 𝐹𝑐 𝑠 = 𝑓 𝑥 cos 𝑠𝑥 𝑑𝑥
𝜋
0

Then the inverse Fourier cosine transform is given by



−1 2
𝐹𝑐 𝐹𝑐 𝑠 = 𝑓(𝑥) = 𝐹𝑐 𝑠 cos 𝑠𝑥 𝑑𝑠
𝜋
0

Parseval’s identity for Fourier transform.

If 𝐹 𝑠 is the Fourier transform of 𝑓(𝑥), then


∞ ∞

|𝑓 𝑥 |2 𝑑𝑥 = |𝐹 𝑠 |2 𝑑𝑠
−∞ −∞

Fourier sine transform and its inverse of 𝒇(𝒙) in (𝟎, 𝒍).

The finite Fourier sine transform of a function 𝑓 𝑥 in 0, 𝑙 is given by


𝑙
𝑛𝜋𝑥
𝑓𝑠 𝑛 = 𝑓 𝑥 𝑠𝑖𝑛 𝑑𝑥,
𝑙
0

Then the inversion formula is given by



2 𝑛𝜋𝑥
𝑓 𝑥 = 𝑓𝑠 𝑛 𝑠𝑖𝑛 .
𝑙 𝑙
𝑛=1

Fourier cosine transform and its inverse of 𝒇(𝒙) in (𝟎, 𝒍).

The finite Fourier cosine transform of a function 𝑓 𝑥 in 0, 𝑙 is given by


𝑙
𝑛𝜋𝑥
𝑓𝑐 𝑛 = 𝑓 𝑥 𝑐𝑜𝑠 𝑑𝑥,
𝑙
0

Then the inversion formula is given by



1 2 𝑛𝜋𝑥
𝑓 𝑥 = 𝑓𝑐 (0) + 𝑓𝑐 𝑛 𝑐𝑜𝑠 .
𝑙 𝑙 𝑙
𝑛 =1

Infinite Fourier sine transform and its inverse

The infinite Fourier sine transform is given by



2
𝐹𝑠 𝑓 𝑥 = 𝐹𝑠 𝑠 = 𝑓 𝑥 sin 𝑠𝑥 𝑑𝑥
𝜋
0

Then the inverse Fourier cosine transform is given by



2
𝐹𝑠 −1 𝐹𝑠 𝑠 = 𝑓(𝑥) = 𝐹𝑠 𝑠 sin 𝑠𝑥 𝑑𝑠
𝜋
0

Parseval’s identity for Fourier sine transform.

If 𝐹𝑆 𝑠 is the Fourier transform of 𝑓(𝑥), then


∞ ∞

|𝑓 𝑥 |2 𝑑𝑥 = |𝐹𝑆 𝑠 |2 𝑑𝑠
0 0

Parseval’s identity for Fourier cosine transform.

If 𝐹𝐶 𝑠 is the Fourier transform of 𝑓(𝑥), then


∞ ∞

|𝑓 𝑥 |2 𝑑𝑥 = |𝐹𝐶 𝑠 |2 𝑑𝑠
0 0

properties of Fourier transform:

Linearity property:

If 𝑓(𝑥) and 𝑔(𝑥)are any two functions then

𝐹 𝑎𝑓(𝑥) + 𝑏𝑔(𝑥) = 𝑎𝐹 𝑓(𝑥) + 𝑏𝐹 𝑔(𝑥) , where a and b are constants.

Shifting property:

If 𝐹 𝑓(𝑥) = 𝐹 𝑠 , then 𝐹 𝑓(𝑥 − 𝑎) = 𝑒 𝑖𝑎𝑠 𝐹(𝑠)

Fourier transforms of the derivatives of a function.

𝑑 𝑛 𝑓(𝑥)
𝐹 = −𝑖𝑠 𝑛 𝐹 𝑠 .
𝑑𝑥 𝑛
𝟏 𝒔
𝑭𝒄 𝒇 𝒂𝒙 = 𝑭𝒄 , 𝒂 > 0.
𝒂 𝒂
Solution:

2
𝐹𝑐 𝑓 𝑎𝑥 = 𝑓 𝑎𝑥 cos 𝑠𝑥 𝑑𝑥
𝜋
0

Put 𝑎𝑥 = 𝑦 when 𝑥 = 0, 𝑦 = 0.
𝑑𝑦
𝑑𝑥 = When 𝑥 = ∞, 𝑦 = ∞
𝑎


2 𝑠𝑦 𝑑𝑦
= 𝑓 𝑦 cos
𝜋 𝑎 𝑎
0


1 2 𝑠
= 𝑓 𝑦 cos 𝑦 𝑑𝑦
𝑎 𝜋 𝑎
0

1 𝑠
= 𝐹𝑐
𝑎 𝑎
𝒅[𝑭 𝒔 ]
If 𝑭 𝒔 = 𝑭 𝒇 𝒙 , then prove that 𝑭 𝒙𝒇 𝒙 = −𝒊 𝒅𝒔

1 ∞
Solution: 𝐹 𝑠 = 𝑓 𝑥 𝑒 𝑖𝑠𝑥 𝑑𝑥
2𝜋 −∞

𝑑[𝐹 𝑠 ] 𝑑 1
= 𝑓 𝑥 𝑒 𝑖𝑠𝑥 𝑑𝑥
𝑑𝑠 𝑑𝑠 2𝜋
−∞


1 𝜕 𝑖𝑠𝑥
= 𝑓 𝑥 (𝑒 )𝑑𝑥
2𝜋 𝜕𝑠
−∞


1
= 𝑓 𝑥 𝑖𝑥(𝑒 𝑖𝑠𝑥 )𝑑𝑥
2𝜋
−∞

= 𝑖𝐹 𝑥𝑓 𝑥

𝑑[𝐹 𝑠 ]
𝐹 𝑥𝑓 𝑥 = −𝑖
𝑑𝑠

If 𝑭𝒔 𝒔 is the Fourier sine transform of 𝒇(𝒙), show that

𝟏
𝑭𝒔 𝒇 𝒙 𝐜𝐨𝐬 𝒂𝒙 = 𝑭 𝒔 + 𝒂 + 𝑭𝒔 𝒔 − 𝒂
𝟐 𝒔
Solution:

2
𝐹𝑠 𝑓 𝑥 𝑐𝑜𝑠𝑎𝑥 = 𝑓 𝑥 cos 𝑎𝑥 sin 𝑠𝑥 𝑑𝑥
𝜋
0


2 1
= . 𝑓 𝑥 sin 𝑎 + 𝑠 𝑥 − sin 𝑎 − 𝑠 𝑥 𝑑𝑥
𝜋 2
0

2 1 ∞ 2 1

= . 𝑓 𝑥 sin 𝑎 + 𝑠 𝑥 𝑑𝑥 − . 𝑓 𝑥 sin 𝑎 − 𝑠 𝑥 𝑑𝑥
𝜋 2 0 𝜋 2 0

1
= 𝐹 𝑠 + 𝑎 + 𝐹𝑠 𝑠 − 𝑎
2 𝑠

If 𝑭(𝒔) is the Fourier transform of 𝒇(𝒙), the Fourier transform of 𝒇(𝒙)𝒄𝒐𝒔 𝒂𝒙


1 ∞
𝐹 𝑓 𝑥 cos 𝑎𝑥 = 𝑓 𝑥 cos 𝑎𝑥 𝑒 𝑖𝑠𝑥 𝑑𝑥
2𝜋 −∞


1 𝑒 𝑖𝑎𝑥 + 𝑒 −𝑖𝑎𝑥 𝑖𝑠𝑥
= 𝑓 𝑥 𝑒 𝑑𝑥
2𝜋 2
−∞

∞ ∞
1 𝑒 𝑖(𝑠+𝑎 )𝑥 1 𝑒 𝑖(𝑠−𝑎 )𝑥
= 𝑓 𝑥 𝑑𝑥 + 𝑓 𝑥 𝑑𝑥
2𝜋 2 2𝜋 2
−∞ −∞
1
𝐹 𝑓 𝑥 cos 𝑎𝑥 = 𝐹 𝑠 + 𝑎 + 𝐹(𝑠 − 𝑎)
2

If 𝒇(𝒙) is an even function of 𝒙, its Fourier transform 𝑭(𝒔) will also be an even function

of s.

Proof: By definition

1
𝐹 𝑠 = 𝑓 𝑥 𝑒 𝑖𝑠𝑥 𝑑𝑥 … 1
2𝜋
−∞

Changing s into –s in both sides of (1),



1
𝐹 −𝑠 = 𝑓 𝑥 𝑒 −𝑖𝑠𝑥 𝑑𝑥 … 2
2𝜋
−∞

In the right hand side integral in (2), put 𝑥 = −𝑢.

then 𝑑𝑥 = −𝑑𝑢; when 𝑥 = ∞, 𝑢 = −∞

and when 𝑥 = −∞, 𝑢 = ∞.

So (2) becomes
−∞
1
𝐹 −𝑠 = 𝑓 −𝑢 𝑒 𝑖𝑠𝑢 . (−𝑑𝑢)
2𝜋


1
= 𝑓 −𝑢 𝑒 𝑖𝑠𝑢 𝑑𝑢
2𝜋
−∞

1 ∞
= 𝑓 −𝑥 𝑒 𝑖𝑠𝑥 𝑑𝑥 [Changing the dummy variable u into x]
2𝜋 −∞


1
= 𝑓 𝑥 𝑒 𝑖𝑠𝑥 𝑑𝑥 = 𝐹 𝑆 𝑏𝑦 1 .
2𝜋
−∞

Here 𝐹 𝑆 is an even function of s.

If 𝒇(𝒙) is an odd function of 𝒙, its Fourier transform 𝑭(𝒔) will also be an odd function of s.

Proof: By definition

1
𝐹 𝑠 = 𝑓 𝑥 𝑒 𝑖𝑠𝑥 𝑑𝑥 … 1
2𝜋
−∞
Changing s into –s in both sides of (1),

1
𝐹 −𝑠 = 𝑓 𝑥 𝑒 −𝑖𝑠𝑥 𝑑𝑥 … 2
2𝜋
−∞

In the right hand side integral in (2), put 𝑥 = −𝑢.

then 𝑑𝑥 = −𝑑𝑢; when 𝑥 = ∞, 𝑢 = −∞

and when 𝑥 = −∞, 𝑢 = ∞.

So (2) becomes
−∞
1
𝐹 −𝑠 = 𝑓 −𝑢 𝑒 𝑖𝑠𝑢 . (−𝑑𝑢)
2𝜋


1
= 𝑓 −𝑢 𝑒 𝑖𝑠𝑢 𝑑𝑢
2𝜋
−∞

1 ∞
= −∞
𝑓 −𝑥 𝑒 𝑖𝑠𝑥 𝑑𝑥 [Changing the dummy variable u into x]
2𝜋


1
= −𝑓 𝑥 𝑒 𝑖𝑠𝑥 𝑑𝑥 = −𝐹 𝑆 𝑏𝑦 1 .
2𝜋
−∞

Here 𝐹 𝑆 is an odd function of s.

If 𝑭𝒄 𝒇 𝒙 = 𝑭𝒄 𝒔 , prove that 𝑭𝒄 𝑭𝒄 𝒙 = 𝒇 𝒔
Proof:
By inverse cosine transform, we get


2
𝑓(𝑥) = 𝐹𝑐 𝑠 cos 𝑠𝑥 𝑑𝑠
𝜋
0
Interchanging 𝑠 𝑎𝑛𝑑 𝑥, we get

2
𝑓(𝑠) = 𝐹𝑐 𝑥 cos 𝑠𝑥 𝑑𝑥
𝜋
0
= 𝐹𝑐 𝐹𝑐 𝑥 𝐵𝑦 𝑑𝑒𝑓𝑖𝑛𝑖𝑡𝑖𝑜𝑛 𝑜𝑓 𝐹𝑜𝑢𝑟𝑖𝑒𝑟 𝑐𝑜𝑠𝑖𝑛𝑒 𝑡𝑟𝑎𝑛𝑠𝑓𝑜𝑟𝑚

Fourier transforms of derivates of a function:



𝑑𝑓(𝑥) 1
𝐹 = 𝑓′ 𝑥 𝑒 𝑖𝑠𝑥 𝑑𝑥
𝑑𝑥 2𝜋
−∞

1
= 𝑒 𝑖𝑠𝑥 𝑑 𝑓(𝑥)
2𝜋
−∞


1 ∞
= 𝑒 𝑖𝑠𝑥 𝑓 𝑥 −∞
− 𝑓 𝑥 𝑖𝑠𝑒 𝑖𝑠𝑥 𝑑𝑥
2𝜋
−∞


𝑖𝑠
=− 𝑓 𝑥 𝑒 𝑖𝑠𝑥 𝑑𝑥
2𝜋
−∞

(assuming 𝑓(𝑥) → 0 𝑎𝑠 𝑥 → ±∞)

= −𝑖𝑠𝐹 𝑠 .

𝑑 2 𝑓(𝑥) 1
𝐹 = 𝑓′′ 𝑥 𝑒 𝑖𝑠𝑥 𝑑𝑥
𝑑𝑥 2 2𝜋
−∞


1
= 𝑒 𝑖𝑠𝑥 𝑑 𝑓′(𝑥)
2𝜋
−∞


1 ∞
= 𝑒 𝑖𝑠𝑥 𝑓′ 𝑥 −∞
− 𝑓′ 𝑥 𝑖𝑠𝑒 𝑖𝑠𝑥 𝑑𝑥
2𝜋
−∞


𝑖𝑠
=− 𝑓′ 𝑥 𝑒 𝑖𝑠𝑥 𝑑𝑥
2𝜋
−∞

(assuming 𝑓′(𝑥) → 0 𝑎𝑠 𝑥 → ±∞)



𝑖𝑠
=− 𝑒 𝑖𝑠𝑥 𝑑 𝑓(𝑥)
2𝜋
−∞


𝑖𝑠 ∞
=− 𝑒 𝑖𝑠𝑥 𝑓 𝑥 −∞
− 𝑓 𝑥 𝑖𝑠𝑒 𝑖𝑠𝑥 𝑑𝑥
2𝜋
−∞


2
−𝑖𝑠
= 𝑓 𝑥 𝑒 𝑖𝑠𝑥 𝑑𝑥
2𝜋
−∞

(assuming 𝑓′(𝑥) → 0 𝑎𝑠 𝑥 → ±∞)

= −𝑖𝑠 2 𝐹 𝑠 .

In general,
𝑑 𝑛 𝑓(𝑥)
𝐹 = −𝑖𝑠 𝑛 𝐹 𝑠 .
𝑑𝑥 𝑛

Convolution theorem of the Fourier transform.

If 𝐹(𝑠) and 𝐺(𝑠) are the functions of 𝑓(𝑥) and 𝑔(𝑥) respectively then the Fourier transform
of the convolution of 𝑓(𝑥) and 𝑔(𝑥) is the product of their Fourier transform

𝐹 (𝑓 ∗ 𝑔 𝑥] = 𝐹 𝑆 . 𝐺(𝑠)
∞ ∞ ∞
1 1 1
𝑓 ∗ 𝑔 𝑥 𝑒 𝑖𝑠𝑥 𝑑𝑥 = 𝑓 𝑥 𝑒 𝑖𝑠𝑥 𝑑𝑥 𝑔 𝑥 𝑒 𝑖𝑠𝑥 𝑑𝑥
2𝜋 2𝜋 2𝜋
−∞ −∞ −∞

1 − 𝑥 𝑓𝑜𝑟 𝑥 ≤ 1
1. Find the Fourier transform of 𝑓 𝑥 =
0 𝑓𝑜𝑟 𝑥 > 1
And deduce that

𝑠𝑖𝑛4 𝑡 𝜋
4
𝑑𝑡 =
𝑡 3
0
Solution:

1
𝐹 𝑓(𝑥) = 𝑓(𝑥)𝑒 𝑖𝑠𝑥 𝑑𝑥
2𝜋
−∞
1
1
𝐹 𝑓(𝑥) = 1 − 𝑥 𝑒 𝑖𝑠𝑥 𝑑𝑥
2𝜋
−1
0 1
1 −𝑥, 𝑥<0
= 1 − −𝑥 𝑒 𝑖𝑠𝑥 𝑑𝑥 + 1 − 𝑥 𝑒 𝑖𝑠𝑥 𝑑𝑥 ∵ 𝑥 =
2𝜋 𝑥, 𝑥≥0
−1 0
0 1
1 𝑖𝑠𝑥
= 1+𝑥 𝑒 𝑑𝑥 + 1 − 𝑥 𝑒 𝑖𝑠𝑥 𝑑𝑥
2𝜋
−1 0
𝑖𝑠𝑥 𝑖𝑠𝑥 0 1
1 𝑒 𝑒 𝑒 𝑖𝑠𝑥 𝑒 𝑖𝑠𝑥
= 1+𝑥 −1 2
+ 1−𝑥 − −1
2𝜋 𝑖𝑠 𝑖𝑠 −1
𝑖𝑠 𝑖𝑠 2 0
1 𝑒 𝑖𝑠0 𝑒 𝑖𝑠0 𝑒 −𝑖𝑠 𝑒 −𝑖𝑠
= 1+0 − − 1−1 −
2𝜋 𝑖𝑠 𝑖𝑠 2 𝑖𝑠 𝑖𝑠 2
1 𝑒 𝑖𝑠 𝑒 𝑖𝑠 𝑒 𝑖𝑠0 𝑒 𝑖𝑠0
+ 1−1 + − 1−0 +
2𝜋 𝑖𝑠 𝑖𝑠 2 𝑖𝑠 𝑖𝑠 2
1
1 1 𝑒 −𝑖𝑠 𝑒 𝑖𝑠 1 1
= − 2
− 0 − 2
+ 0 + 2
− + 2
2𝜋 𝑖𝑠 𝑖𝑠 𝑖𝑠 𝑖𝑠 𝑖𝑠 𝑖𝑠
1 1 𝑒 −𝑖𝑠 𝑒 𝑖𝑠 1
= − 2
+ 2
+ 2

2𝜋 𝑖𝑠 𝑖𝑠 𝑖𝑠 𝑖𝑠 2
1 1 𝑒 −𝑖𝑠 𝑒 𝑖𝑠 1 1 2 𝑒 𝑖𝑠 + 𝑒 −𝑖𝑠
= − − + = −
2𝜋 𝑠 2 𝑠2 𝑠2 𝑠2 2𝜋 𝑠 2 𝑠2
1 2 2 cos 𝑠 2 1
= 2
− = 1 − cos 𝑠
2𝜋 𝑠 𝑠2 2𝜋 𝑠 2
2𝑠
2 2𝑠𝑖𝑛 2
=
𝜋 𝑠2
By Parseval’s identity
∞ ∞
2 2
𝑓(𝑥) 𝑑𝑥 = 𝐹 𝑓(𝑥) 𝑑𝑠
−∞ −∞
2
1 ∞ 2 𝑠
2
2 2𝑠𝑖𝑛 2
1− 𝑥 𝑑𝑥 = 𝑑𝑠
𝜋 𝑠2
−1 −∞
2
1 ∞ 2𝑠
2 2𝑠𝑖𝑛 2
2 1 − 𝑥 2 𝑑𝑥 = 2 𝑑𝑠
𝜋 𝑠2
0 0
∞ 𝑠4 3 1
8 𝑠𝑖𝑛 2 1−𝑥 1−1 3
1−0 3
1
𝑑𝑠 = = − =
𝜋 𝑠4 −3 0
−3 −3 3
0
∞ 𝑠
𝑠𝑖𝑛4 2 𝜋
𝑑𝑠 =
𝑠4 24
0
𝑠
𝑃𝑢𝑡 𝑡 = 𝑠 = 2𝑡, 𝑑𝑠 = 2𝑑𝑡, 𝑤𝑕𝑒𝑛 𝑠 = 0 𝑡 = 0 𝑎𝑛𝑑 𝑠 = ∞ 𝑡=∞
2 ∞
𝑠𝑖𝑛4 𝑡 𝜋
4
2𝑑𝑡 =
2𝑡 24
0

𝑠𝑖𝑛4 𝑡 𝜋
4
𝑑𝑡 =
𝑡 3
0
2. Using Parseval’s identity calculate
∞ ∞
𝑑𝑥 𝑥 2 𝑑𝑥
𝑖) 𝑖𝑖) 𝑖𝑓 𝑎 > 0
𝑥 + 𝑎2
2 2 𝑥 2 + 𝑎2 2
0 0
Solution: i) We know that
2 𝑎
𝐹𝑐 𝑒 −𝑎𝑥 =
𝜋 𝑎 + 𝑠2
2

Here 𝑓 𝑥 = 𝑒 −𝑎𝑥

Using Parseval’s identity


∞ ∞
2 2
𝑓(𝑥) 𝑑𝑥 = 𝐹𝑐 𝑓(𝑥) 𝑑𝑠
0 0
∞ ∞ 2
2 𝑎
𝑒 −𝑎𝑥 2 𝑑𝑥 = 𝑑𝑠
𝜋 𝑎 + 𝑠2
2
0 0
∞ ∞
2
−2𝑎𝑥
2𝑎2 1
𝑒 𝑑𝑥 = 𝑑𝑠
𝜋 𝑎2 + 𝑠 2
0 0

2 2 −2𝑎𝑥 ∞
2𝑎 1 𝑒 𝑒 −∞ 𝑒0
𝑑𝑠 = = −
𝜋 𝑎 + 𝑠2
2 −2𝑎 0
−2𝑎 −2𝑎
0

2
2𝑎2 1 1
𝑑𝑠 =
𝜋 𝑎2 + 𝑠 2 2𝑎
0

𝑑𝑥 𝜋
= (∵ 𝑠 𝑖𝑠 𝑑𝑢𝑚𝑚𝑦 𝑣𝑎𝑟𝑖𝑎𝑏𝑙𝑒 𝑖𝑡 𝑖𝑠 𝑟𝑒𝑝𝑙𝑎𝑐𝑒𝑑 𝑏𝑦 𝑥)
𝑥2 + 𝑎2 2 4𝑎3
0
ii) We know that
2 𝑠
𝐹𝑠 𝑒 −𝑎𝑥 =
𝜋 𝑎2 + 𝑠 2

Here 𝑓 𝑥 = 𝑒 −𝑎𝑥

Using Parseval’s identity


∞ ∞
2
𝑓(𝑥) 2 𝑑𝑥 = 𝐹𝑠 𝑓(𝑥) 𝑑𝑠
0 0
∞ ∞ 2
2 𝑠
𝑒 −𝑎𝑥 2 𝑑𝑥 = 𝑑𝑠
𝜋 𝑎 + 𝑠2
2
0 0
∞ ∞
2 𝑠 2
𝑒 −2𝑎𝑥 𝑑𝑥 = 𝑑𝑠
𝜋 𝑎 + 𝑠2
2
0 0

−2𝑎𝑥 ∞
2 𝑠 2 𝑒 𝑒 −∞ 𝑒0
𝑑𝑠 = = −
𝜋 𝑎 + 𝑠2
2 −2𝑎 0
−2𝑎 −2𝑎
0

2 𝑠 2 1
𝑑𝑠 =
𝜋 𝑎2 + 𝑠 2 2𝑎
0

2
𝑥 𝑑𝑥 𝜋
= (∵ 𝑠 𝑖𝑠 𝑑𝑢𝑚𝑚𝑦 𝑣𝑎𝑟𝑖𝑎𝑏𝑙𝑒 𝑖𝑡 𝑖𝑠 𝑟𝑒𝑝𝑙𝑎𝑐𝑒𝑑 𝑏𝑦 𝑥)
𝑥2 + 𝑎2 2 4𝑎
0

1 𝑓𝑜𝑟 |𝑥| ≤ 𝑎
3. Find the Fourier transform 𝑓 𝑥 = 𝑖𝑓 𝑎 > 0
0 𝑓𝑜𝑟 𝑥 > 𝑎
Deduce that
∞ ∞
2
sin 𝑡 𝜋 sin 𝑡 𝜋
𝑖 𝑑𝑡 = 𝑖𝑖) 𝑑𝑡 =
𝑡 2 𝑡 2
0 0
Solution:

1
𝐹 𝑓(𝑥) = 𝑓(𝑥)𝑒 𝑖𝑠𝑥 𝑑𝑥
2𝜋
−∞
𝑎 𝑎
1 𝑖𝑠𝑥
𝑒 𝑖𝑠𝑥
1
𝐹 𝑓(𝑥) = 1𝑒 𝑑𝑥 =
2𝜋 2𝜋 𝑖𝑠 −𝑎
−𝑎
𝑖𝑠𝑎 −𝑖𝑠𝑎 𝑖𝑠𝑎 −𝑖𝑠𝑎
1 𝑒 𝑒 1 1 𝑒 −𝑒
= − =
2𝜋 𝑖𝑠 𝑖𝑠 2𝜋 𝑠 𝑖
1 2 sin 𝑎𝑠 2 sin 𝑎𝑠
= =
2𝜋 𝑠 𝜋 𝑠
i) By inverse Fourier transform

1
𝑓(𝑥) = 𝐹 𝑓(𝑥) 𝑒 −𝑖𝑠𝑥 𝑑𝑠
2𝜋
−∞

1 2 sin 𝑎𝑠 −𝑖𝑠𝑥
1= 𝑒 𝑑𝑠 … (1)
2𝜋 𝜋 𝑠
−∞
Put 𝑥 = 0 𝑎𝑛𝑑 𝑎 = 1 𝑖𝑛 1 , 𝑤𝑒 𝑔𝑒𝑡

1 sin 𝑠
1= 𝑑𝑠
𝜋 𝑠
−∞

2 sin 𝑠
1= 𝑑𝑠
𝜋 𝑠
0

sin 𝑠 𝜋
𝑑𝑠 =
𝑠 2
0
ii) By Parseval’s identity, we get
∞ ∞
2
𝑓(𝑥) 2 𝑑𝑥 = 𝐹 𝑓(𝑥) 𝑑𝑠
−∞ −∞
𝑎 ∞ 2
2 sin 𝑎𝑠
1 2 𝑑𝑥 = 𝑑𝑠
𝜋 𝑠
−𝑎 −∞
𝑎 ∞
2
2 sin 𝑎𝑠
2 𝑑𝑥 = 2 𝑑𝑠
𝜋 𝑠
0 0

2
2 sin 𝑎𝑠 𝑎
𝑑𝑠 = 𝑥 0
𝜋 𝑠
0

2
sin 𝑎𝑠 𝜋
𝑑𝑠 = 𝑎 … (2)
𝑠 2
0
Put 𝑎 = 1 𝑖𝑛 2 , 𝑤𝑒 𝑔𝑒𝑡

2
sin 𝑠 𝜋
𝑑𝑠 =
𝑠 2
0
1 − 𝑥 2 𝑓𝑜𝑟 |𝑥| ≤ 1
4. Find the Fourier transform of 𝑓 𝑥 =
0 𝑓𝑜𝑟 |𝑥| > 1
Hence prove that

sin 𝑠 − 𝑠 cos 𝑠 𝑠 3𝜋
cos 𝑑𝑠 =
𝑠3 2 16
0
Solution:

1
𝐹 𝑓(𝑥) = 𝑓(𝑥)𝑒 𝑖𝑠𝑥 𝑑𝑥
2𝜋
−∞
1
1
𝐹 𝑓(𝑥) = 1 − 𝑥 2 𝑒 𝑖𝑠𝑥 𝑑𝑥
2𝜋
−1
1
1 2
𝑒 𝑖𝑠𝑥 𝑒 𝑖𝑠𝑥 𝑒 𝑖𝑠𝑥
= 1−𝑥 − −2𝑥 + (−2)
2𝜋 𝑖𝑠 𝑖𝑠 2 𝑖𝑠 3 −1
1 𝑒 𝑖𝑠 𝑒 𝑖𝑠 𝑒 𝑖𝑠
= 1−1 +2 − 2
2𝜋 𝑖𝑠 𝑖𝑠 2 𝑖𝑠 3
1 𝑒 −𝑖𝑠 𝑒 −𝑖𝑠 𝑒 −𝑖𝑠
− 1−1 + 2 −1 −2
2𝜋 𝑖𝑠 𝑖𝑠 2 𝑖𝑠 3
1 𝑒 𝑖𝑠 𝑒 −𝑖𝑠 𝑒 𝑖𝑠 𝑒 −𝑖𝑠
= 2 + 2 − 2 + 2
2𝜋 𝑖𝑠 2 𝑖𝑠 2 𝑖𝑠 3 𝑖𝑠 3
2 𝑒 𝑖𝑠 𝑒 −𝑖𝑠 𝑒 𝑖𝑠 𝑒 −𝑖𝑠
= − 2 + − 2 − − 3 + − 3
2𝜋 𝑠 𝑠 𝑖𝑠 𝑖𝑠
2 1 1 𝑒 𝑖𝑠 − 𝑒 −𝑖𝑠
= − 2 𝑒 𝑖𝑠 + 𝑒 −𝑖𝑠 + 3
𝜋 𝑠 𝑠 𝑖

2 2 cos 𝑠 2 sin 𝑠 2 1
= − + = 2 sin 𝑠 − 𝑠 cos 𝑠
𝜋 𝑠2 𝑠3 𝜋 𝑠3

By Inverse Fourier transform, we get



1
𝑓(𝑥) = 𝐹 𝑓(𝑥) 𝑒 −𝑖𝑠𝑥 𝑑𝑠
2𝜋
−∞

1 2 1
𝑓(𝑥) = 2 sin 𝑠 − 𝑠 cos 𝑠 𝑒 −𝑖𝑠𝑥 𝑑𝑠
2𝜋 𝜋 𝑠3
−∞

2 sin 𝑠 − 𝑠 cos 𝑠
𝑓(𝑥) = cos 𝑠𝑥 − 𝑖𝑠𝑖𝑛 𝑠𝑥 𝑑𝑠
𝜋 𝑠3
−∞
∞ ∞
2 sin 𝑠 − 𝑠 cos 𝑠 2 sin 𝑠 − 𝑠 cos 𝑠
𝑓(𝑥) = 3
cos 𝑠𝑥 𝑑𝑠 − sin 𝑠𝑥 𝑑𝑠
𝜋 𝑠 𝜋 𝑠3
−∞ −∞

4 sin 𝑠 − 𝑠 cos 𝑠
𝑓(𝑥) = cos 𝑠𝑥 𝑑𝑠 − 0 … (1)
𝜋 𝑠3
0
∵ 𝑇𝑕𝑒 𝑓𝑢𝑛𝑐𝑡𝑖𝑜𝑛 𝑖𝑛 𝑓𝑖𝑟𝑠𝑡 𝑖𝑛𝑡𝑒𝑟𝑔𝑟𝑎𝑙 𝑖𝑠 𝑒𝑣𝑒𝑛 𝑎𝑛𝑑 𝑡𝑕𝑒 𝑓𝑢𝑛𝑐𝑡𝑖𝑜𝑛 𝑖𝑛 𝑠𝑒𝑐𝑜𝑛𝑑 𝑖𝑛𝑡𝑒𝑔𝑟𝑎𝑙 𝑖𝑠 𝑜𝑑𝑑
1
𝑃𝑢𝑡 𝑥 = 𝑖𝑛 1 , 𝑤𝑒 𝑔𝑒𝑡
2

1 4 sin 𝑠 − 𝑠 cos 𝑠 𝑠
𝑓 = cos 𝑑𝑠
2 𝜋 𝑠3 2
0

2
1 4 sin 𝑠 − 𝑠 cos 𝑠 𝑠
1− = 3
cos 𝑑𝑠
2 𝜋 𝑠 2
0

4 sin 𝑠 − 𝑠 cos 𝑠 𝑠 1 3
cos 𝑑𝑠 = 1 − =
𝜋 𝑠3 2 4 4
0

sin 𝑠 − 𝑠 cos 𝑠 𝑠 3𝜋
3
cos 𝑑𝑠 =
𝑠 2 16
0

2𝑥 2
5. Find the Fourier cosine transform of 𝑒 −𝑎 ,𝑎 > 0
Solution:

2
𝐹𝑐 𝑓(𝑥) = 𝑓 𝑥 cos 𝑠𝑥 𝑑𝑥
𝜋
0

2𝑥 2 2 2𝑥 2
𝐹𝑐 𝑒 −𝑎 = 𝑒 −𝑎 cos 𝑠𝑥 𝑑𝑥
𝜋
0

1 2 2𝑥 2
= 𝑒 −𝑎 cos 𝑠𝑥 𝑑𝑥
2 𝜋
−∞

1 2𝑥 2
= 𝑒 −𝑎 𝑅. 𝑃 𝑜𝑓 𝑒 𝑖𝑠𝑥 𝑑𝑥
2𝜋
−∞

1 2𝑥 2
= 𝑅. 𝑃 𝑜𝑓 𝑒 −𝑎 𝑒 𝑖𝑠𝑥 𝑑𝑥
2𝜋
−∞

1 2 𝑥 2 +𝑖𝑠𝑥
= 𝑅. 𝑃 𝑜𝑓 𝑒 −𝑎 𝑑𝑥
2𝜋
−∞

1 𝑖𝑠𝑥
−𝑎 2 𝑥 2 − 2
= 𝑅. 𝑃 𝑜𝑓 𝑒 𝑎 𝑑𝑥
2𝜋
−∞

𝑖𝑠𝑥 𝑖𝑠 2 𝑖𝑠 2
1 −𝑎 2 𝑥 2 − 2 +
𝑎 2𝑎 2 −
2𝑎 2
= 𝑅. 𝑃 𝑜𝑓 𝑒 𝑑𝑥
2𝜋
−∞

𝑖𝑠 2 𝑖𝑠 2
1 −𝑎 2 𝑥− 2 −
2𝑎 2𝑎 2
= 𝑅. 𝑃 𝑜𝑓 𝑒 𝑑𝑥
2𝜋
−∞

𝑖𝑠 2 𝑖𝑠 2
1 − 𝑎𝑥 −
2𝑎

2𝑎
= 𝑅. 𝑃 𝑜𝑓 𝑒 𝑑𝑥
2𝜋
−∞

1 𝑖𝑠 2 𝑖𝑠 2
𝑒− 𝑎𝑥 −
= 𝑅. 𝑃 𝑜𝑓 2𝑎 𝑒 2𝑎 𝑑𝑥
2𝜋
−∞

1 𝑠2 𝑖𝑠 2
− − 𝑎𝑥 −
= 𝑅. 𝑃 𝑜𝑓 𝑒 4𝑎 2 𝑒 2𝑎 𝑑𝑥
2𝜋
−∞
𝑖𝑠 𝑑𝑦
𝑃𝑢𝑡 𝑦 = 𝑎𝑥 − , 𝑑𝑦 = 𝑎𝑑𝑥 𝑑𝑥 = , 𝑤𝑕𝑒𝑛 𝑥 = ±∞ 𝑦 = ±∞
2𝑎 𝑎 ∞
1 − 𝑠 22 2 𝑑𝑦
= 𝑅. 𝑃 𝑜𝑓 𝑒 4𝑎 𝑒 −𝑦
2𝜋 𝑎
−∞
1 𝑠2 𝜋

= 𝑅. 𝑃 𝑜𝑓 𝑒 4𝑎 2
2𝜋 𝑎
1 𝑠2
2𝑥 2 −
𝐹𝑐 𝑒 −𝑎 = 𝑒 4𝑎 2
𝑎 2
−𝑥 2 2
6. Show that 𝑓 𝑥 = 𝑒 is self reciprocal under the Fourier cosine transform, deduce
−𝑥 2 2
that 𝑔 𝑥 = 𝑥𝑒 is self reciprocal under the Fourier sine transform.
Solution:

2
𝐹𝑐 𝑓(𝑥) = 𝑓 𝑥 cos 𝑠𝑥 𝑑𝑥
𝜋
0

𝑥2 2 𝑥2
𝐹𝑐 𝑒 − 2 = 𝑒− 2 cos 𝑠𝑥 𝑑𝑥
𝜋
0

1 2 𝑥2
= 𝑒− 2 cos 𝑠𝑥 𝑑𝑥
2 𝜋
−∞

1 𝑥2

= 𝑒 2 𝑅. 𝑃 𝑜𝑓 𝑒 𝑖𝑠𝑥 𝑑𝑥
2𝜋
−∞

1 𝑥2
= 𝑅. 𝑃 𝑜𝑓 𝑒− 2 𝑒 𝑖𝑠𝑥 𝑑𝑥
2𝜋
−∞

1 𝑥2
− +𝑖𝑠𝑥
= 𝑅. 𝑃 𝑜𝑓 𝑒 2 𝑑𝑥
2𝜋
−∞

1 1
𝑥 2 −2𝑖𝑠𝑥
= 𝑅. 𝑃 𝑜𝑓 𝑒− 2 𝑑𝑥
2𝜋
−∞

1 1
𝑥 2 −2𝑖𝑠𝑥 + 𝑖𝑠 2 − 𝑖𝑠 2
= 𝑅. 𝑃 𝑜𝑓 𝑒− 2 𝑑𝑥
2𝜋
−∞

1 1
𝑥−𝑖𝑠 2 − 𝑖𝑠 2
= 𝑅. 𝑃 𝑜𝑓 𝑒− 2 𝑑𝑥
2𝜋
−∞

𝑥−𝑖𝑠 2 𝑖𝑠 2
1 −
= 𝑅. 𝑃 𝑜𝑓 𝑒 2 𝑒 2 𝑑𝑥
2𝜋
−∞

𝑠2 𝑥−𝑖𝑠 2
1 − −
= 𝑅. 𝑃 𝑜𝑓 𝑒 2 𝑒 2 𝑑𝑥
2𝜋
−∞
𝑥 − 𝑖𝑠 𝑑𝑥
𝑃𝑢𝑡 𝑦 = , 𝑑𝑦 = 𝑑𝑥 = 2𝑑𝑦, 𝑤𝑕𝑒𝑛 𝑥 = ±∞ 𝑦 = ±∞
2 2

1 𝑠2 2
= 𝑅. 𝑃 𝑜𝑓 𝑒− 4 𝑒 −𝑦 2𝑑𝑦
2𝜋
−∞
1 𝑠2
= 𝑅. 𝑃 𝑜𝑓 𝑒− 4 2 𝜋
2𝜋
𝑥2 𝑠2
𝐹𝑐 𝑒 − 2 = 𝑒− 2

𝑑
𝐹𝑠 𝑥𝑓(𝑥) = − 𝐹 𝑓(𝑥)
𝑑𝑠 𝑐
𝑥2 𝑑 𝑥2
𝐹𝑠 𝑥𝑒 − 2 = − 𝐹𝑐 𝑒 − 2
𝑑𝑠
2
𝑑 − 𝑠 2𝑠 − 𝑠 2
=− 𝑒 2 =− − 𝑒 2
𝑑𝑠 2
𝑥2 𝑠2
𝐹𝑠 𝑥𝑒 − 2 = 𝑠𝑒 − 2

7. Find the Fourier sine transform of


𝑒 −𝑎𝑥
,𝑎 > 0
𝑥
Solution:

2
𝐹𝑠 𝑓(𝑥) = 𝑓 𝑥 sin 𝑠𝑥 𝑑𝑥
𝜋
0

𝑒 −𝑎𝑥 2 𝑒 −𝑎𝑥
𝐹𝑠 = sin 𝑠𝑥 𝑑𝑥
𝑥 𝜋 𝑥
0
Differentiating with respect to 𝑠, we get

𝑑 𝑒 −𝑎𝑥 𝑑 2 𝑒 −𝑎𝑥
𝐹𝑠 = sin 𝑠𝑥 𝑑𝑥
𝑑𝑠 𝑥 𝑑𝑠 𝜋 𝑥
0

𝑑 𝑒 −𝑎𝑥 2 𝑒 −𝑎𝑥 𝑑
𝐹𝑠 = sin 𝑠𝑥 𝑑𝑥
𝑑𝑠 𝑥 𝜋 𝑥 𝑑𝑠
0

𝑑 𝑒 −𝑎𝑥 2 𝑒 −𝑎𝑥
𝐹𝑠 = 𝑥 cos 𝑠𝑥 𝑑𝑥
𝑑𝑠 𝑥 𝜋 𝑥
0

𝑑 𝑒 −𝑎𝑥 2
𝐹𝑠 = 𝑒 −𝑎𝑥 cos 𝑠𝑥 𝑑𝑥
𝑑𝑠 𝑥 𝜋
0

𝑑 𝑒 −𝑎𝑥 2 𝑒 −𝑎𝑥
𝐹 = −𝑎 cos 𝑠𝑥 + 𝑠 sin 𝑠𝑥
𝑑𝑠 𝑠 𝑥 𝜋 𝑎2 + 𝑠 2 0

2 𝑒 −∞ 𝑒0
= −𝑎 cos 𝑠∞ + 𝑠 sin 𝑠∞ − −𝑎 cos 𝑠0 + 𝑠 sin 𝑠0
𝜋 𝑎2 + 𝑠 2 𝑎2 + 𝑠 2
𝑑 𝑒 −𝑎𝑥 2 𝑎
𝐹𝑠 =
𝑑𝑠 𝑥 𝜋 𝑎 + 𝑠2
2

𝑒 −𝑎𝑥 2 𝑎 2 𝑑𝑠
𝐹𝑠 = 𝑑𝑠 = 𝑎
𝑥 𝜋 𝑎 + 𝑠2
2 𝜋 𝑎2 + 𝑠2

𝑒 −𝑎𝑥 2 1 𝑠
𝐹𝑠 = 𝑎 𝑡𝑎𝑛−1
𝑥 𝜋 𝑎 𝑎

𝑒 −𝑎𝑥 2 𝑠
𝐹𝑠 = 𝑡𝑎𝑛−1
𝑥 𝜋 𝑎

2
8. Find the Fourier cosine transform of 𝑒 −𝑥
Solution:

2
𝐹𝑐 𝑓(𝑥) = 𝑓 𝑥 cos 𝑠𝑥 𝑑𝑥
𝜋
0

2 2 2
𝐹𝑐 𝑒 −𝑥 = 𝑒 −𝑥 cos 𝑠𝑥 𝑑𝑥
𝜋
0

1 2 2
= 𝑒 −𝑥 cos 𝑠𝑥 𝑑𝑥
2 𝜋
−∞

1 2
= 𝑒 −𝑥 𝑅. 𝑃 𝑜𝑓 𝑒 𝑖𝑠𝑥 𝑑𝑥
2𝜋
−∞

1 2
= 𝑅. 𝑃 𝑜𝑓 𝑒 −𝑥 𝑒 𝑖𝑠𝑥 𝑑𝑥
2𝜋
−∞

1 2 +𝑖𝑠𝑥
= 𝑅. 𝑃 𝑜𝑓 𝑒 −𝑥 𝑑𝑥
2𝜋
−∞

1 2 −𝑖𝑠𝑥
= 𝑅. 𝑃 𝑜𝑓 𝑒− 𝑥 𝑑𝑥
2𝜋
−∞

𝑖𝑠 2 𝑖𝑠 2
1 − 𝑥 2 −𝑖𝑠𝑥 +
2

2
= 𝑅. 𝑃 𝑜𝑓 𝑒 𝑑𝑥
2𝜋
−∞

𝑖𝑠 2 𝑖𝑠 2
1 − 𝑥−
2

2
= 𝑅. 𝑃 𝑜𝑓 𝑒 𝑑𝑥
2𝜋
−∞

1 𝑖𝑠 2 𝑖𝑠 2
− 𝑥−
= 𝑅. 𝑃 𝑜𝑓 𝑒 2 𝑒 2 𝑑𝑥
2𝜋
−∞

1 𝑠2 𝑖𝑠 2
− 𝑥−
= 𝑅. 𝑃 𝑜𝑓 𝑒− 4 𝑒 2 𝑑𝑥
2𝜋
−∞
𝑖𝑠
𝑃𝑢𝑡 𝑦 = 𝑥 − , 𝑑𝑦 = 𝑑𝑥 𝑑𝑥 = 𝑑𝑦, 𝑤𝑕𝑒𝑛 𝑥 = ±∞ 𝑦 = ±∞
2 ∞
1 𝑠2 2

= 𝑅. 𝑃 𝑜𝑓 𝑒 4 𝑒 −𝑦 𝑑𝑦
2𝜋
−∞
1 𝑠2
= 𝑅. 𝑃 𝑜𝑓 𝑒− 4 𝜋
2𝜋
2 1 − 𝑠2
𝐹𝑐 𝑒 −𝑥 = 𝑒 4
2

𝑥 𝑓𝑜𝑟 0 ≤ 𝑥 ≤ 1
9. Find the Fourier sine transform of 𝑓 𝑥 = 2 − 𝑥 𝑓𝑜𝑟 1 ≤ 𝑥 ≤ 2
0 𝑓𝑜𝑟 𝑥 > 2
Solution:

2
𝐹𝑠 𝑓(𝑥) = 𝑓 𝑥 sin 𝑠𝑥 𝑑𝑥
𝜋
0
1 2
2
= 𝑓 𝑥 sin 𝑠𝑥 𝑑𝑥 + 𝑓 𝑥 sin 𝑠𝑥 𝑑𝑥
𝜋
0 1
1 2
2
= 𝑥 sin 𝑠𝑥 𝑑𝑥 + 2 − 𝑥 sin 𝑠𝑥 𝑑𝑥
𝜋
0 1
1 2
2 cos 𝑠𝑥 sin 𝑠𝑥 cos 𝑠𝑥 sin 𝑠𝑥
= 𝑥 − −1 − 2 + 2−𝑥 − − (−1) − 2
𝜋 𝑠 𝑠 0 𝑠 𝑠 1

2 cos 𝑠 sin 𝑠 cos 2𝑠 sin 2𝑠


= 1 − +1 −0+ 2−2 − −
𝜋 𝑠 𝑠2 𝑠 𝑠2
cos 𝑠 sin 𝑠
− 2−1 − −
𝑠 𝑠2
2 cos 𝑠 sin 𝑠 sin 2𝑠 cos 𝑠 sin 𝑠
= − + 2 − 2 + + 2
𝜋 𝑠 𝑠 𝑠 𝑠 𝑠

2 2sin 𝑠 sin 2𝑠 2 2sin 𝑠 2 sin 𝑠 cos 𝑠


= − 2 = −
𝜋 𝑠2 𝑠 𝜋 𝑠2 𝑠2

2 2sin 𝑠 2 2sin 𝑠 2
𝑠
= 1 − cos 𝑠 = 2𝑠𝑖𝑛
𝜋 𝑠2 𝜋 𝑠2 2

2 sin 𝑠 𝑠
𝐹𝑠 𝑓(𝑥) = 4 2
𝑠𝑖𝑛2
𝜋 𝑠 2

𝑎2 − 𝑥 2 𝑓𝑜𝑟 |𝑥| ≤ 𝑎
10. Find the complex Fourier transform of 𝑓 𝑥 =
0 𝑓𝑜𝑟 𝑥 > 𝑎

Hence deduce
∞ ∞ 2
sin 𝑥 − 𝑥 cos 𝑥 𝜋 sin 𝑥 − 𝑥 cos 𝑥 𝜋
𝑖) 𝑑𝑥 = 𝑖𝑖) 𝑑𝑥 =
𝑥3 4 𝑥3 15
0 0

Solution:

1
𝐹 𝑓(𝑥) = 𝑓(𝑥)𝑒 𝑖𝑠𝑥 𝑑𝑥
2𝜋
−∞
𝑎
1
𝐹 𝑓(𝑥) = 𝑎2 − 𝑥 2 𝑒 𝑖𝑠𝑥 𝑑𝑥
2𝜋
−𝑎
𝑖𝑠𝑥 𝑎
1 2 2
𝑒 𝑒 𝑖𝑠𝑥 𝑒 𝑖𝑠𝑥
= 𝑎 −𝑥 − −2𝑥 + (−2)
2𝜋 𝑖𝑠 𝑖𝑠 2 𝑖𝑠 3 −𝑎
1 𝑒 𝑖𝑠𝑎 𝑒 𝑖𝑠𝑎 𝑒 𝑖𝑠𝑎
= 𝑎2 − 𝑎2 + 2𝑎 − 2
2𝜋 𝑖𝑠 𝑖𝑠 2 𝑖𝑠 3
1 𝑒 −𝑖𝑠𝑎 𝑒 −𝑖𝑠𝑎 𝑒 −𝑖𝑠𝑎
− 𝑎2 − 𝑎2 + 2 −𝑎 − 2
2𝜋 𝑖𝑠 𝑖𝑠 2 𝑖𝑠 3
1 𝑒 𝑖𝑠𝑎 𝑒 −𝑖𝑠𝑎 𝑒 𝑖𝑠𝑎 𝑒 −𝑖𝑠𝑎
= 2 + 2𝑎 − 2 + 2𝑎
2𝜋 𝑖𝑠 2 𝑖𝑠 2 𝑖𝑠 3 𝑖𝑠 3
𝑖𝑠𝑎 −𝑖𝑠𝑎 𝑖𝑠𝑎 −𝑖𝑠𝑎
2 𝑒 𝑒 𝑒 𝑒
= − 2 + − 2 − − 3 + − 3
2𝜋 𝑠 𝑠 𝑖𝑠 𝑖𝑠
2 𝑎 𝑖𝑠𝑎 −𝑖𝑠𝑎
1 𝑒 𝑖𝑠𝑎 − 𝑒 −𝑖𝑠𝑎
= − 𝑒 +𝑒 + 3
𝜋 𝑠2 𝑠 𝑖

2 2 acos 𝑎𝑠 2 sin 𝑎𝑠 2 1
= − 2
+ 3
=2 sin 𝑎𝑠 − 𝑎𝑠 cos 𝑎𝑠
𝜋 𝑠 𝑠 𝜋 𝑠3

i) By Inverse Fourier transform, we get



1
𝑓(𝑥) = 𝐹 𝑓(𝑥) 𝑒 −𝑖𝑠𝑥 𝑑𝑠
2𝜋
−∞

1 2 1
𝑓 𝑥 = 2 sin 𝑎𝑠 − 𝑎𝑠 cos 𝑎𝑠 𝑒 −𝑖𝑠𝑥 𝑑𝑠 … (1)
2𝜋 𝜋 𝑠3
−∞
Put 𝑥 = 0 𝑖𝑛 1 , 𝑤𝑒 𝑔𝑒𝑡

2 sin 𝑎𝑠 − 𝑎𝑠 cos 𝑎𝑠
𝑓 0 = 𝑑𝑠
𝜋 𝑠3
−∞


2 sin 𝑎𝑠 − 𝑎𝑠 cos 𝑎𝑠
𝑑𝑠 = 𝑎2 − 0 … (2)
𝜋 𝑠3
−∞

Put 𝑎 = 1 𝑖𝑛 2 , 𝑤𝑒 𝑔𝑒𝑡

2 sin 𝑠 − 𝑠 cos 𝑠
𝑑𝑠 = 1
𝜋 𝑠3
−∞


4 sin 𝑠 − 𝑠 cos 𝑠
𝑑𝑠 = 1
𝜋 𝑠3
0


sin 𝑥 − 𝑥 cos 𝑥 𝜋
𝑑𝑥 = (∵ 𝑠 𝑖𝑠 𝑑𝑢𝑚𝑚𝑦 𝑣𝑎𝑟𝑖𝑎𝑏𝑙𝑒 𝑖𝑡 𝑖𝑠 𝑟𝑒𝑝𝑙𝑎𝑐𝑒𝑑 𝑏𝑦 𝑥)
𝑥3 4
0

ii) By Parseval’s identity, we get


∞ ∞
2
𝑓(𝑥) 2 𝑑𝑥 = 𝐹 𝑓(𝑥) 𝑑𝑠
−∞ −∞

𝑎 ∞ 2
2 sin 𝑎𝑠 − 𝑎𝑠 cos 𝑎𝑠
𝑎2 − 𝑥 2 2 𝑑𝑥 = 2 𝑑𝑠
𝜋 𝑠3
−𝑎 −∞
𝑎 ∞ 2
2 sin 𝑎𝑠 − 𝑎𝑠 cos 𝑎𝑠
2 𝑎2 − 𝑥 2 2 𝑑𝑥 = 2 2 𝑑𝑠
𝜋 𝑠3
0 0

𝑎 ∞ 2
4
8
2 2 4
sin 𝑎𝑠 − 𝑎𝑠 cos 𝑎𝑠
𝑎 − 2𝑎 𝑥 + 𝑥 𝑑𝑥 = 𝑑𝑠
𝜋 𝑠3
0 0

∞ 2 𝑎
8 sin 𝑎𝑠 − 𝑎𝑠 cos 𝑎𝑠 4
𝑥3 𝑥5 2
𝑑𝑠 = 𝑎 𝑥 − 2𝑎 +
𝜋 𝑠3 3 5 0
0

∞ 2
8 sin 𝑎𝑠 − 𝑎𝑠 cos 𝑎𝑠 2𝑎5 𝑎5 8𝑎5
𝑑𝑠 = 𝑎5 − + = … (3)
𝜋 𝑠3 3 5 15
0

𝑝𝑢𝑡 𝑎 = 1 𝑖𝑛 3 , 𝑤𝑒 𝑔𝑒𝑡
∞ 2
1 sin 𝑠 − 𝑠 cos 𝑠 1
𝑑𝑠 =
𝜋 𝑠3 15
0

∞ 2
sin 𝑥 − 𝑥 cos 𝑥 𝜋
𝑑𝑥 = (∵ 𝑠 𝑖𝑠 𝑑𝑢𝑚𝑚𝑦 𝑣𝑎𝑟𝑖𝑎𝑏𝑙𝑒 𝑖𝑡 𝑖𝑠 𝑟𝑒𝑝𝑙𝑎𝑐𝑒𝑑 𝑏𝑦 𝑥)
𝑥3 15
0

11. If 𝐹(𝑠) is the Fourier transform of 𝑓(𝑥), find the Fourier transform of

𝑓 𝑥 − 𝑎 𝑎𝑛𝑑 𝑓(𝑎𝑥)
𝑆𝑜𝑙𝑢𝑡𝑖𝑜𝑛:

1
𝐹 𝑠 = 𝐹 𝑓(𝑥) = 𝑓(𝑥)𝑒 𝑖𝑠𝑥 𝑑𝑥
2𝜋
−∞


1
𝐹 𝑓(𝑥 − 𝑎) = 𝑓(𝑥 − 𝑎)𝑒 𝑖𝑠𝑥 𝑑𝑥
2𝜋
−∞

𝑝𝑢𝑡 𝑦 = 𝑥 − 𝑎 𝑥 = 𝑦 + 𝑎, 𝑑𝑥 = 𝑑𝑦, 𝑤𝑕𝑒𝑛 𝑥 = ±∞ 𝑦 = ±∞

∞ ∞
1 1
= 𝑓(𝑦)𝑒 𝑖𝑠 𝑦+𝑎
𝑑𝑦 = 𝑓(𝑦)𝑒 𝑖𝑠𝑦 𝑒 𝑖𝑠𝑎 𝑑𝑦
2𝜋 2𝜋
−∞ −∞


1
= 𝑒 𝑖𝑠𝑎 𝑓 𝑦 𝑒 𝑖𝑠𝑦 𝑑𝑦 = 𝑒 𝑖𝑠𝑎 𝐹[𝑠]
2𝜋
−∞

𝐹 𝑓(𝑥 − 𝑎) = 𝑒 𝑖𝑠𝑎 𝐹[𝑠]



1
𝐹 𝑓(𝑎𝑥) = 𝑓(𝑎𝑥)𝑒 𝑖𝑠𝑥 𝑑𝑥
2𝜋
−∞

𝑦 𝑑𝑦
𝑝𝑢𝑡 𝑦 = 𝑎𝑥 𝑥= , 𝑑𝑥 = , 𝑤𝑕𝑒𝑛 𝑥 = ±∞ 𝑦 = ±∞
𝑎 𝑎
∞ ∞
1 𝑖𝑠
𝑦 𝑑𝑦 1 1 𝑠
= 𝑓(𝑦)𝑒 𝑎 = 𝑓(𝑦)𝑒 𝑖 𝑎 𝑦 𝑑𝑦
2𝜋 𝑎 𝑎 2𝜋
−∞ −∞

1 𝑠
𝐹 𝑓(𝑎𝑥) = 𝐹
𝑎 𝑎

12. Evaluate the following integral using Fourier transforms



𝑑𝑥
𝑥 2 + 𝑎2 𝑥 2 + 𝑏2
0
Solution: We know that
2 𝑎 2 𝑏
𝐹𝑐 𝑒 −𝑎𝑥 = 2 2
, 𝐹𝑐 𝑒 −𝑏𝑥 =
𝜋 𝑠 +𝑎 𝜋 𝑠 2 + 𝑏2
Here 𝑓 𝑥 = 𝑒 −𝑎𝑥 𝑎𝑛𝑑 𝑔 𝑥 = 𝑒 −𝑏𝑥
By convolution theorem, we get
∞ ∞

𝑓 𝑥 . 𝑔 𝑥 𝑑𝑥 = 𝐹𝑐 𝑓 𝑥 𝐹𝑐 𝑔 𝑥 𝑑𝑠
0 0
∞ ∞
−𝑎𝑥 −𝑏𝑥
2 𝑎 2 𝑏
𝑒 .𝑒 𝑑𝑥 = 𝑑𝑠
𝜋 𝑠 + 𝑎2
2 𝜋 𝑠 + 𝑏2
2
0 0
∞ ∞
2𝑎𝑏 1
𝑒 −(𝑎 +𝑏)𝑥 𝑑𝑥 = 𝑑𝑠
𝜋 𝑠2 + 𝑎2 𝑠 2 + 𝑏2
0 0
∞ ∞
2𝑎𝑏 𝑑𝑠 𝑒 −(𝑎 +𝑏)𝑥
=
𝜋 𝑠 2 + 𝑎2 𝑠 2 + 𝑏2 −(𝑎 + 𝑏) 0
0

2𝑎𝑏 𝑑𝑠 𝑒 −∞ 𝑒0 1
2 2 2 2
= − =
𝜋 𝑠 +𝑎 𝑠 +𝑏 −(𝑎 + 𝑏) −(𝑎 + 𝑏) 𝑎 + 𝑏
0

𝑑𝑥 𝜋
= (∵ 𝑠 𝑖𝑠 𝑑𝑢𝑚𝑚𝑦 𝑣𝑎𝑟𝑖𝑎𝑏𝑙𝑒 𝑖𝑡 𝑖𝑠 𝑟𝑒𝑝𝑙𝑎𝑐𝑒𝑑 𝑏𝑦 𝑥)
𝑥2 + 𝑎2 2
𝑥 +𝑏 2 2𝑎𝑏(𝑎 + 𝑏)
0

13. Find the Fourier transform of 𝑒 −𝑎 |𝑥| , 𝑎 > 0 and hence deduce that

𝑑𝑥 𝜋
=
𝑥2 + 𝑎2 2 4𝑎3
0

Solution:

1
𝐹 𝑓(𝑥) = 𝑓(𝑥)𝑒 𝑖𝑠𝑥 𝑑𝑥
2𝜋
−∞

1
𝐹 𝑓(𝑥) = 𝑒 −𝑎 |𝑥| 𝑒 𝑖𝑠𝑥 𝑑𝑥
2𝜋
−∞
0 ∞
1 −𝑥, 𝑥<0
= 𝑒 −𝑎 (−𝑥 ) 𝑒 𝑖𝑠𝑥 𝑑𝑥 + 𝑒 −𝑎𝑥 𝑒 𝑖𝑠𝑥 𝑑𝑥 ∵ 𝑥 =
2𝜋 𝑥, 𝑥≥0
−∞ 0

0 ∞
1
= 𝑒 (𝑎+𝑖𝑠)𝑥 𝑑𝑥 + 𝑒 −(𝑎−𝑖𝑠)𝑥 𝑑𝑥
2𝜋
−∞ 0

0 ∞
𝑒 (𝑎+𝑖𝑠)𝑥
1 𝑒 −(𝑎 −𝑖𝑠)𝑥
= −
2𝜋 𝑎 + 𝑖𝑠 −∞
𝑎 − 𝑖𝑠 0

1 𝑒0 𝑒 −∞ 𝑒 −∞ 𝑒0
= − − −
2𝜋 𝑎 + 𝑖𝑠 𝑎 + 𝑖𝑠 𝑎 − 𝑖𝑠 𝑎 − 𝑖𝑠

1 1 1 1 𝑎 − 𝑖𝑠 + 𝑎 + 𝑖𝑠
= + =
2𝜋 𝑎 + 𝑖𝑠 𝑎 − 𝑖𝑠 2𝜋 𝑎2 − 𝑖𝑠 2
1 2𝑎
𝐹 𝑓(𝑥) =
2𝜋 𝑎2 + 𝑠2

By Parseval’s identity, we get


∞ ∞
2
𝑓(𝑥) 2 𝑑𝑥 = 𝐹 𝑓(𝑥) 𝑑𝑠
−∞ −∞
∞ ∞ 2
−𝑎 |𝑥| 2
1 2𝑎
𝑒 𝑑𝑥 = 𝑑𝑠
2𝜋 𝑎2 + 𝑠 2
−∞ −∞
∞ ∞ 2
1 2𝑎
2 𝑒 −𝑎𝑥 2 𝑑𝑥 = 2 𝑑𝑠
2𝜋 𝑎 + 𝑠 2
2
0 0
∞ ∞
−2𝑎𝑥
2 𝑎 2
𝑒 𝑑𝑥 = 𝑑𝑠
𝜋 𝑎2 + 𝑠 2
0 0


2 ∞
2𝑎2 1 𝑒 −2𝑎𝑥 𝑒 −∞ 𝑒0 1
𝑑𝑠 = = − =
𝜋 𝑎 + 𝑠2
2 −2𝑎 0
−2𝑎 −2𝑎 2𝑎
0


2
2𝑎2 1 1
𝑑𝑠 =
𝜋 𝑎 + 𝑠2
2 2𝑎
0


𝑑𝑥 𝜋
= (∵ 𝑠 𝑖𝑠 𝑑𝑢𝑚𝑚𝑦 𝑣𝑎𝑟𝑖𝑎𝑏𝑙𝑒 𝑖𝑡 𝑖𝑠 𝑟𝑒𝑝𝑙𝑎𝑐𝑒𝑑 𝑏𝑦 𝑥)
𝑥2 + 𝑎2 2 4𝑎3
0

14. Find the Fourier cosine transform of 𝑓(𝑥) = 𝑒 −4𝑥 . Deduce that
∞ ∞
cos 2𝑥 𝑑𝑥 𝜋 −8 𝑥 𝑠𝑖𝑛 2𝑥 𝑑𝑥 𝜋 −8
= 𝑒 𝑎𝑛𝑑 = 𝑒
𝑥 2 + 16 8 𝑥 2 + 16 2
0 0


2
𝐹𝑐 𝑓(𝑥) = 𝑓 𝑥 cos 𝑠𝑥 𝑑𝑥
𝜋
0


2
𝐹𝑐 𝑒 −4𝑥 = 𝑒 −4𝑥 cos 𝑠𝑥 𝑑𝑥
𝜋
0


2 𝑒 −4𝑥
= −4cos 𝑠𝑥 + 𝑠𝑠𝑖𝑛 𝑠𝑥
𝜋 −4 2 + 𝑠 2 0
2 𝑒 −4 ∞ 𝑒 −4 0
= −4cos 𝑠∞ + 𝑠𝑠𝑖𝑛 𝑠∞ − −4cos 𝑠0 + 𝑠𝑠𝑖𝑛 𝑠0
𝜋 16 + 𝑠 2 16 + 𝑠 2

2 4
𝐹𝑐 𝑒 −4𝑥 =
𝜋 16 + 𝑠 2

By inverse cosine transform,



2
𝑓(𝑥) = 𝐹𝑐 𝑓(𝑥) cos 𝑠𝑥 𝑑𝑠
𝜋
0


2 2 4
𝑒 −4𝑥 = cos 𝑠𝑥 𝑑𝑠
𝜋 𝜋 16 + 𝑠 2
0


1 𝜋
2
cos 𝑠𝑥 𝑑𝑠 = 𝑒 −4𝑥
16 + 𝑠 8
0

Interchanging 𝑥 𝑎𝑛𝑑 𝑠, we get



1 𝜋
2
cos 𝑠𝑥 𝑑𝑥 = 𝑒 −4𝑠 … (1)
16 + 𝑥 8
0

Differentiating (1) with respect to s, we get



𝑑 1 𝑑 𝜋 −4𝑠
cos 𝑠𝑥 𝑑𝑠 = 𝑒
𝑑𝑠 16 + 𝑥 2 𝑑𝑠 8
0

Differentiating under integral sign we get,



1 𝑑 𝑑 𝜋 −4𝑠
cos 𝑠𝑥 𝑑𝑠 = 𝑒
16 + 𝑥 2 𝑑𝑠 𝑑𝑠 8
0


1 𝜋
2
−𝑥 sin 𝑠𝑥 𝑑𝑠 = −4𝑒 −4𝑠
16 + 𝑥 8
0


𝑥 𝜋 −4𝑠
sin 𝑠𝑥 𝑑𝑠 = 𝑒 … (2)
16 + 𝑥 2 2
0

𝑃𝑢𝑡 𝑠 = 2 𝑖𝑛 1 𝑎𝑛𝑑 (2), 𝑤𝑒 𝑔𝑒𝑡



cos 2𝑥 𝑑𝑥 𝜋 −8
= 𝑒
𝑥 2 + 16 8
0


𝑥 𝑠𝑖𝑛 2𝑥 𝑑𝑥 𝜋 −8
= 𝑒
𝑥 2 + 16 2
0

15. Find the Fourier sine and cosine transform 𝑓(𝑥) = 𝑒 −𝑥 and hence find the Fourier
1 𝑥
cosine transform of and Fourier sine transform of
1+𝑥 2 1+𝑥 2

Solution:

2
𝐹𝑐 𝑓(𝑥) = 𝑓 𝑥 cos 𝑠𝑥 𝑑𝑥
𝜋
0


2
𝐹𝑐 𝑒 −𝑥 = 𝑒 −𝑥 cos 𝑠𝑥 𝑑𝑥
𝜋
0

2 𝑒 −𝑥 ∞
= −cos 𝑠𝑥 + 𝑠𝑠𝑖𝑛 𝑠𝑥
𝜋 1 + 𝑠2 0

2 𝑒 −∞ 𝑒 −0
= −cos 𝑠∞ + 𝑠𝑠𝑖𝑛 𝑠∞ − −cos 𝑠0 + 𝑠𝑠𝑖𝑛 𝑠0
𝜋 1 + 𝑠2 1 + 𝑠2

2 1
𝐹𝑐 𝑒 −𝑥 =
𝜋 1 + 𝑠2

By inverse cosine transform,



2
𝑓(𝑥) = 𝐹𝑐 𝑓(𝑥) cos 𝑠𝑥 𝑑𝑠
𝜋
0


−𝑥
2 2 1
𝑒 = cos 𝑠𝑥 𝑑𝑠
𝜋 𝜋 1 + 𝑠2
0


2 1 𝜋 −𝑥
cos 𝑠𝑥 𝑑𝑠 = 𝑒
𝜋 1 + 𝑠2 2
0
Interchanging 𝑥 𝑎𝑛𝑑 𝑠, we get

2 1 𝜋 −𝑠
cos 𝑠𝑥 𝑑𝑥 = 𝑒
𝜋 1 + 𝑥2 2
0

1 𝜋 −𝑠
𝐹𝑐 = 𝑒
1 + 𝑥2 2

2
𝐹𝑠 𝑓(𝑥) = 𝑓 𝑥 sin 𝑠𝑥 𝑑𝑥
𝜋
0


2
𝐹𝑠 𝑒 −𝑥 = 𝑒 −𝑥 cos 𝑠𝑥 𝑑𝑥
𝜋
0

2 𝑒 −𝑥 ∞
= −sin 𝑠𝑥 − 𝑠𝑐𝑜𝑠 𝑠𝑥
𝜋 1 + 𝑠2 0

2 𝑒 −∞ 𝑒 −0
= −sin 𝑠∞ − 𝑠𝑐𝑜𝑠 𝑠∞ − −sin 𝑠0 − 𝑠𝑐𝑜𝑠 𝑠0
𝜋 1 + 𝑠2 1 + 𝑠2

2 𝑠
𝐹𝑠 𝑒 −𝑥 =
𝜋 1 + 𝑠2

By inverse sine transform,



2
𝑓(𝑥) = 𝐹𝑠 𝑓(𝑥) sin 𝑠𝑥 𝑑𝑠
𝜋
0


2 2 2
𝑒 −𝑥 = sin 𝑠𝑥 𝑑𝑠
𝜋 𝜋 1 + 𝑠2
0


2 𝑠 𝜋 −𝑥
sin 𝑠𝑥 𝑑𝑠 = 𝑒
𝜋 1 + 𝑠2 2
0

Interchanging 𝑥 𝑎𝑛𝑑 𝑠, we get



2 𝑥 𝜋 −𝑠
cos 𝑠𝑥 𝑑𝑥 = 𝑒
𝜋 1 + 𝑥2 2
0
𝑥 𝜋 −𝑠
𝐹𝑠 = 𝑒
1 + 𝑥2 2
𝑥
Aliter: For finding 𝐹𝑠
1+𝑥 2

𝑑
𝐹𝑠 𝑥𝑓(𝑥) = − 𝐹 𝑓(𝑥)
𝑑𝑠 𝑐
1 𝑑 1
𝐹𝑠 𝑥 2
= − 𝐹𝑐
1+𝑥 𝑑𝑠 1 + 𝑥2
𝑑 𝜋 −𝑠 𝜋
=− 𝑒 =− −𝑒 −𝑠
𝑑𝑠 2 2

𝑥 𝜋 −𝑠
𝐹𝑠 = 𝑒
1 + 𝑥2 2
Partial Differential equation
Partial differential equation:

The equations which involving coefficients as partial derivatives are called partial differential
equations.

Solution of a partial differential equation:

The relation between the dependent and the independent variables without involving any
partial derivatives is called solution of a partial differential equation.

Complete integral:

A solution of a partial differential equation which contains the maximum possible number of
arbitrary constants is called complete integral.

Particular integral:

A solution obtained by giving particular values to the arbitrary constants in a complete


integral is called a particular integral.

General integral:

A solution of a partial differential equation which contains the maximum possible number of
arbitrary functions is called general integral.

Let 𝐹 𝑥, 𝑦, 𝑧, 𝑝, 𝑞 = 0 … 1 be a partial differential equation and its complete integral be

𝛷 𝑥, 𝑦, 𝑧, 𝑎, 𝑏 = 0 … 2

Substituting 𝑏 = 𝑓 𝑎 𝑖𝑛 2 , 𝑤𝑒 𝑔𝑒𝑡

𝛷 𝑥, 𝑦, 𝑧, 𝑎, 𝑓(𝑎) = 0 … 3

Differentiating (3) partially w.r.t. a, we get

𝜕𝛷
= 0 … (4)
𝜕𝑎
The elimination of a from (3) and (4), if it exits, is called general integral.

Singular integral:

Let 𝐹 𝑥, 𝑦, 𝑧, 𝑝, 𝑞 = 0 … 1 be a partial differential equation and its complete integral be

𝛷 𝑥, 𝑦, 𝑧, 𝑎, 𝑏 = 0 … 2

Differentiating (2) partially w.r.t. a and b in turn, we get


𝜕𝛷
= 0 … (3)
𝜕𝑎
𝜕𝛷
= 0 … (4)
𝜕𝑏
The elimination of a and b from (2), (3) and (4), if it exits, is called singular integral.

1. Form partial differential equation by eliminating the arbitrary functions 𝑓 𝑎𝑛𝑑 𝑔 in

𝑧 = 𝑓(𝑥 3 + 2𝑦) + 𝑔(𝑥 3 − 2𝑦).

Solution:

𝑧 = 𝑓 𝑥 3 + 2𝑦 + 𝑔 𝑥 3 − 2𝑦 … (1)

Differentiating (1) partially with respect to 𝑥,

𝜕𝑧
𝑝= = 𝑓′ 𝑥 3 + 2𝑦 3𝑥 2 + 𝑔′ 𝑥 3 − 2𝑦 3𝑥 2 … (2)
𝜕𝑥
Differentiating (1) partially with respect to 𝑦,

𝜕𝑧
𝑞= = 𝑓′ 𝑥 3 + 2𝑦 2 + 𝑔′ 𝑥 3 − 2𝑦 −2 … (3)
𝜕𝑦

Differentiating (2) partially with respect to 𝑥,

𝜕2 𝑧
𝑟= = 𝑓′′ 𝑥 3 + 2𝑦 9𝑥 4 + 𝑔′′ 𝑥 3 − 2𝑦 9𝑥 4 + 𝑓′ 𝑥 3 + 2𝑦 6𝑥 + 𝑔′ 𝑥 3 − 2𝑦 6𝑥 … (4)
𝜕𝑥 2
Differentiating (3) partially with respect to 𝑦,

𝜕2𝑧
𝑡= = 𝑓′′ 𝑥 3 + 2𝑦 4 + 𝑔′′ 𝑥 3 − 2𝑦 4 … (5)
𝜕𝑦 2

Differentiating (3) partially with respect to 𝑥,

𝜕2 𝑧
𝑠= = 𝑓′′ 𝑥 3 + 2𝑦 6𝑥 2 − 𝑔′′ 𝑥 3 − 2𝑦 6𝑥 2 … (6)
𝜕𝑥𝜕𝑦

𝑟 = 9𝑥 4 𝑓′′ 𝑥 3 + 2𝑦 + 𝑔′′ 𝑥 3 − 2𝑦 + 6𝑥 𝑓′ 𝑥 3 + 2𝑦 + 𝑔′ 𝑥 3 − 2𝑦

𝑡 𝑝 𝑡 2𝑝
𝑟 = 9𝑥 4 + 6𝑥 2 = 9𝑥 4 + (𝑓𝑟𝑜𝑚 2 & 5 )
4 3𝑥 4 𝑥

4𝑥𝑟 = 9𝑥 5 𝑡 + 8𝑝

2. Form partial differential equation by eliminating the arbitrary functions 𝑓 𝑎𝑛𝑑 𝑔 in

𝑧 = 𝑥𝑓(2𝑥 + 𝑦) + 𝑔(2𝑥 + 𝑦).


Solution:

𝑧 = 𝑥𝑓 2𝑥 + 𝑦 + 𝑔 2𝑥 + 𝑦 … (1)

Differentiating (1) partially with respect to 𝑥,

𝜕𝑧
𝑝= = 𝑥𝑓′ 2𝑥 + 𝑦 2 + 𝑓 2𝑥 + 𝑦 + 𝑔′ 2𝑥 + 𝑦 2 … (2)
𝜕𝑥
Differentiating (1) partially with respect to 𝑦,

𝜕𝑧
𝑞= = 𝑥𝑓′ 2𝑥 + 𝑦 + 𝑔′ 2𝑥 + 𝑦 … (3)
𝜕𝑦

Differentiating (2) partially with respect to 𝑥,

𝜕2𝑧
𝑟= = 𝑥𝑓′′ 2𝑥 + 𝑦 4 + 𝑓′ 2𝑥 + 𝑦 2 + 𝑓′ 2𝑥 + 𝑦 2 + 𝑔′′ 2𝑥 + 𝑦 4
𝜕𝑥 2
𝜕2𝑧
𝑟 = 2 = 𝑥𝑓′′ 2𝑥 + 𝑦 4 + 𝑓′ 2𝑥 + 𝑦 4 + 𝑔′′ 2𝑥 + 𝑦 4 … (4)
𝜕𝑥
Differentiating (3) partially with respect to 𝑦,

𝜕2 𝑧
𝑡= = 𝑥𝑓′′ 2𝑥 + 𝑦 + 𝑔′′ 2𝑥 + 𝑦 … (5)
𝜕𝑦 2

Differentiating (2) partially with respect to 𝑦,

𝜕2 𝑧
𝑠= = 𝑥𝑓′′ 2𝑥 + 𝑦 2 + 𝑓′ 2𝑥 + 𝑦 + 𝑔′′ 2𝑥 + 𝑦 2 … (6)
𝜕𝑦𝜕𝑥

From (4), (5) and (6), we get

4𝑠 − 𝑟 = 4𝑥𝑓′′ 2𝑥 + 𝑦 + 4 𝑔′′ 2𝑥 + 𝑦 = 4𝑡

4𝑠 − 𝑟 = 4𝑡

3. Find the singular solution of 𝑧 = 𝑝𝑥 + 𝑞𝑦 + 1 + 𝑝2 + 𝑞 2

Solution:

𝑧 = 𝑝𝑥 + 𝑞𝑦 + 1 + 𝑝2 + 𝑞 2 … (1)

(1) is of Type-II

Let 𝑧 = 𝑎𝑥 + 𝑏𝑦 + 𝑐 … 2 𝑏𝑒 𝑡𝑕𝑒 𝑐𝑜𝑚𝑝𝑙𝑒𝑡𝑒 𝑠𝑜𝑙𝑢𝑡𝑖𝑜𝑛 𝑜𝑓 (1)

Differentiating partially with respect to 𝑥 𝑎𝑛𝑑 𝑦, 𝑤𝑒 𝑔𝑒𝑡


𝜕𝑧 𝜕𝑧
𝑝= = 𝑎, 𝑞 = =𝑏
𝜕𝑥 𝜕𝑦

Substituting the values of 𝑝 and 𝑞 in (1),

𝑧 = 𝑎𝑥 + 𝑏𝑦 + 1 + 𝑎2 + 𝑏2 … 3 𝑖𝑠 𝑡𝑕𝑒 𝑐𝑜𝑚𝑝𝑙𝑒𝑡𝑒 𝑆𝑜𝑙𝑢𝑡𝑖𝑜𝑛 𝑜𝑓 1 .

To find singular solution:

𝜕𝑧 𝜕𝑧
= 0& =0
𝜕𝑎 𝜕𝑏
𝜕𝑧 2𝑎 −𝑎
=𝑥+ =0 𝑥= … (4)
𝜕𝑎 2 1 + 𝑎2 + 𝑏2 1 + 𝑎2 + 𝑏2
𝜕𝑧 2𝑏 −𝑏
=𝑦+ =0 𝑦= … (5)
𝜕𝑏 2 1 + 𝑎2 + 𝑏2 1 + 𝑎2 + 𝑏2

𝑎2 𝑏2 1 + 𝑎2 + 𝑏2 − 𝑎2 − 𝑏2
1 − 𝑥2 − 𝑦2 = 1 − − =
1 + 𝑎2 + 𝑏2 1 + 𝑎2 + 𝑏2 1 + 𝑎2 + 𝑏2
1 1
1 − 𝑥2 − 𝑦2 = 1 − 𝑥2 − 𝑦2 = … (6)
1 + 𝑎2 + 𝑏2 1 + 𝑎2 + 𝑏2

Substituting (4 and (5) in (6), we get


−𝑥
𝑥 = −𝑎 1 − 𝑥 2 − 𝑦 2 𝑎= … (7)
1 − 𝑥2 − 𝑦2
−𝑦
𝑦 = −𝑏 1 − 𝑥 2 − 𝑦 2 𝑏= … (8)
1 − 𝑥2 − 𝑦2

Substituting (6),(7) and (8) in (3), we get

−𝑥 −𝑦 1
𝑧= 𝑥+ 𝑦+
1 − 𝑥2 − 𝑦2 1 − 𝑥2 − 𝑦2 1 − 𝑥2 − 𝑦2

1 − 𝑥2 − 𝑦2
𝑧=
1 − 𝑥2 − 𝑦2

𝑧= 1 − 𝑥 2 − 𝑦 2 𝑖𝑠 𝑡𝑕𝑒 𝑠𝑖𝑛𝑔𝑢𝑙𝑎𝑟 𝑠𝑜𝑙𝑢𝑡𝑖𝑜𝑛.

4. Solve 𝑝(1 − 𝑞 2 ) = 𝑞(1 − 𝑧)

Solution:

𝑝(1 − 𝑞 2 ) = 𝑞 1 − 𝑧 … (1)

This is of type 𝐹 𝑧, 𝑝, 𝑞 = 0
Let 𝑢 = 𝑥 + 𝑎𝑦 … (2)

𝜕𝑧 𝜕𝑧 𝜕𝑢 𝜕𝑧
𝑝= = = .1
𝜕𝑥 𝜕𝑢 𝜕𝑥 𝜕𝑢
𝜕𝑧 𝜕𝑧 𝜕𝑢 𝜕𝑧
𝑞= = = .𝑎
𝜕𝑦 𝜕𝑢 𝜕𝑦 𝜕𝑢

Substituting the values of 𝑝 and 𝑞 in (1),


2
𝜕𝑧 𝜕𝑧 𝜕𝑧
1− 𝑎 = 𝑎 1−𝑧
𝜕𝑢 𝜕𝑢 𝜕𝑢

=𝑎 1−𝑧
2 2
𝜕𝑧 1 − 𝑎 − 𝑎𝑧 𝜕𝑧
1−𝑎 1−𝑧 = 𝑎 =
𝜕𝑢 𝑎2 𝜕𝑢
𝜕𝑧 1
= 1 − 𝑎 − 𝑎𝑧
𝜕𝑢 𝑎
𝑎𝜕𝑧
= 𝜕𝑢
1 − 𝑎 − 𝑎𝑧

Integrating on both sides,

𝑎𝜕𝑧
= 𝜕𝑢
1 − 𝑎 − 𝑎𝑧

−2 1 − 𝑎 − 𝑎𝑧 = 𝑢 + 𝑐 … (3)

Substituting (2) in (3)

−2 1 − 𝑎 − 𝑎𝑧 = 𝑥 + 𝑎𝑦 + 𝑐 is the complete solution.

5. Solve 𝑧 = 𝑝2 + 𝑞 2

Solution:

𝑧 = 𝑝2 + 𝑞 2 … (1)

This is of type 𝐹 𝑧, 𝑝, 𝑞 = 0

Let 𝑢 = 𝑥 + 𝑎𝑦 … (2)

𝜕𝑧 𝜕𝑧 𝜕𝑢 𝜕𝑧
𝑝= = = .1
𝜕𝑥 𝜕𝑢 𝜕𝑥 𝜕𝑢
𝜕𝑧 𝜕𝑧 𝜕𝑢 𝜕𝑧
𝑞= = = .𝑎
𝜕𝑦 𝜕𝑢 𝜕𝑦 𝜕𝑢
Substituting the values of 𝑝 and 𝑞 in (1),
2 2
𝜕𝑧 𝜕𝑧
𝑧= + .𝑎
𝜕𝑢 𝜕𝑢
2
𝜕𝑧
𝑧= 1 + 𝑎2
𝜕𝑢
2
𝑧 𝜕𝑧
2
=
1+𝑎 𝜕𝑢

𝑧 𝜕𝑧
2
=
1+𝑎 𝜕𝑢

1 + 𝑎2 𝜕𝑧
= 𝜕𝑢
𝑧

2 𝑧 1 + 𝑎2 = 𝑢 + 𝑐 … (3)

Substituting (2) in (3)

2 𝑧 1 + 𝑎2 = 𝑥 + 𝑎𝑦 + 𝑐 is the complete solution.

6. Solve 𝑦 − 𝑥𝑧 𝑝 + 𝑦𝑧 − 𝑥 𝑞 = 𝑥 + 𝑦 (𝑥 − 𝑦)

Solution:

The equation is of the form 𝑃𝑝 + 𝑄𝑞 = 𝑅

Here 𝑃 = 𝑦 − 𝑥𝑧 , 𝑄 = 𝑦𝑧 − 𝑥 , 𝑅 = 𝑥 + 𝑦 (𝑥 − 𝑦)

𝑑𝑥 𝑑𝑦 𝑑𝑧
= =
𝑦 − 𝑥𝑧 𝑦𝑧 − 𝑥 𝑥 + 𝑦 (𝑥 − 𝑦)

𝑦𝑑𝑥 + 𝑥𝑑𝑦 𝑑𝑧
= 2
𝑦2 − 𝑥𝑦𝑧 + 𝑥𝑦𝑧 − 𝑥 2 𝑥 − 𝑦2

𝑦𝑑𝑥 + 𝑥𝑑𝑦 𝑑𝑧
2 2
= 2
𝑦 −𝑥 𝑥 − 𝑦2

𝑦𝑑𝑥 + 𝑥𝑑𝑦 = −𝑑𝑧

𝑑 𝑥𝑦 = −𝑑𝑧

Integrating on both sides, we get

𝑥𝑦 = −𝑧 + 𝑐1 𝑐1 = 𝑥𝑦 + 𝑧
𝑥𝑑𝑥 + 𝑦𝑑𝑦 𝑑𝑧
= 2
𝑥𝑦 − 𝑥 𝑧 + 𝑦 𝑧 − 𝑥𝑦 𝑥 − 𝑦 2
2 2

𝑥𝑑𝑥 + 𝑦𝑑𝑦 𝑑𝑧
=
−𝑧 𝑥 2 − 𝑦 2 𝑥2 − 𝑦2

𝑥𝑑𝑥 + 𝑦𝑑𝑦 = −𝑧𝑑𝑧

Integrating on both sides, we get

𝑥2 𝑦2 𝑧2
+ =− + 𝑐2 𝑐3 = 𝑥 2 +𝑦 2 + 𝑧 2
2 2 2

The general solution is

Φ 𝑐1 , 𝑐3 = 0

Φ 𝑥𝑦 + 𝑧, 𝑥 2 +𝑦 2 + 𝑧 2 = 0

7. Solve 𝑥 ( 𝑧 2 – 𝑦 2 ) 𝑝 + 𝑦 (𝑥 2 – 𝑧 2 )𝑞 = 𝑧(𝑦 2 – 𝑥 2 ).

Solution:

The equation is of the form 𝑃𝑝 + 𝑄𝑞 = 𝑅

Here 𝑃 = 𝑥 ( 𝑧 2 – 𝑦 2 ) , 𝑄 = 𝑦 (𝑥 2 – 𝑧 2 ), 𝑅 = 𝑧(𝑦 2 – 𝑥 2 )

𝑑𝑥 𝑑𝑦 𝑑𝑧
= =
𝑧22
𝑥( – 𝑦 ) 𝑦 (𝑥 – 𝑧 ) 𝑧(𝑦 – 𝑥 2 )
2 2 2

1 1
𝑑𝑥 +
𝑥 𝑦 𝑑𝑦 𝑑𝑧
=
𝑧2 − 𝑦2 + 𝑥2 − 𝑧2 𝑧(𝑦 2 – 𝑥 2 )

1 1
𝑥 𝑑𝑥 + 𝑦 𝑑𝑦 𝑑𝑧
2 2
=
−(𝑦 – 𝑥 ) 𝑧(𝑦 – 𝑥 2 )
2

1 1 𝑑𝑧
𝑑𝑥 + 𝑑𝑦 = −
𝑥 𝑦 𝑧

Integrating on both sides, we get

log 𝑥 + log 𝑦 = − log 𝑧 + log 𝑐1

log 𝑥 + log 𝑦 + log 𝑧 = log 𝑐1

log 𝑥𝑦𝑧 = log 𝑐1 𝑐1 = 𝑥𝑦𝑧


𝑥𝑑𝑥 + 𝑦𝑑𝑦 𝑑𝑧
=
𝑥 2𝑧2 2 2 2 2 2
− 𝑥 𝑦 +𝑥 𝑦 −𝑦 𝑧 2 𝑧(𝑦 – 𝑥 2 )
2

𝑥𝑑𝑥 + 𝑦𝑑𝑦 𝑑𝑧
=
𝑥 2𝑧2 − 𝑦2𝑧2 𝑧(𝑦 2 – 𝑥 2 )

𝑥𝑑𝑥 + 𝑦𝑑𝑦 𝑑𝑧
2 2 2
=
−𝑧 (𝑦 – 𝑥 ) 𝑧(𝑦 – 𝑥 2 )
2

𝑥𝑑𝑥 + 𝑦𝑑𝑦 = −𝑧𝑑𝑧

Integrating on both sides, we get

𝑥2 𝑦2 𝑧2
+ =− + 𝑐2 𝑐3 = 𝑥 2 +𝑦 2 + 𝑧 2
2 2 2

The general solution is

Φ 𝑐1 , 𝑐3 = 0

Φ 𝑥𝑦𝑧, 𝑥 2 +𝑦 2 + 𝑧 2 = 0

8. Find the singular solution of 𝑧 = 𝑝𝑥 + 𝑞𝑦 + 𝑝2 − 𝑞 2

Solution:

𝑧 = 𝑝𝑥 + 𝑞𝑦 + 𝑝2 − 𝑞 2 … (1)

(1) is of Type-II

Let 𝑧 = 𝑎𝑥 + 𝑏𝑦 + 𝑐 … 2 𝑏𝑒 𝑡𝑕𝑒 𝑐𝑜𝑚𝑝𝑙𝑒𝑡𝑒 𝑠𝑜𝑙𝑢𝑡𝑖𝑜𝑛 𝑜𝑓 (1)

Differentiating partially with respect to 𝑥 𝑎𝑛𝑑 𝑦, 𝑤𝑒 𝑔𝑒𝑡

𝜕𝑧 𝜕𝑧
𝑝= = 𝑎, 𝑞 = =𝑏
𝜕𝑥 𝜕𝑦

Substituting the values of 𝑝 and 𝑞 in (1),

𝑧 = 𝑎𝑥 + 𝑏𝑦 + 𝑎2 − 𝑏2 … 3 𝑖𝑠 𝑡𝑕𝑒 𝑐𝑜𝑚𝑝𝑙𝑒𝑡𝑒 𝑆𝑜𝑙𝑢𝑡𝑖𝑜𝑛 𝑜𝑓 1 .

To find singular solution:

𝜕𝑧 𝜕𝑧
= 0& =0
𝜕𝑎 𝜕𝑏
𝜕𝑧 −𝑥
= 𝑥 + 2𝑎 = 0 𝑎= … (4)
𝜕𝑎 2
𝜕𝑧 𝑦
= 𝑦 − 2𝑏 = 0 𝑏= … (5)
𝜕𝑏 2
Substituting (4) and (5) in (3), we get

−𝑥 𝑦 −𝑥 2 𝑦 2
𝑧= 𝑥+ 𝑦+ −
2 2 2 2
−𝑥 2 𝑦 2 𝑥 2 𝑦 2
𝑧= + + −
2 2 4 4
𝑥2 𝑦2
𝑧=− + 4𝑧 = 𝑦 2 − 𝑥 2 𝑖𝑠 𝑡𝑕𝑒 𝑠𝑖𝑛𝑔𝑢𝑙𝑎𝑟 𝑠𝑜𝑙𝑢𝑡𝑖𝑜𝑛.
4 4

9. Solve 𝑝2 𝑦 1 + 𝑥 2 = 𝑞𝑥 2

Solution:

𝑝2 1 + 𝑥 2 𝑞
𝑝2 𝑦 1 + 𝑥 2 = 𝑞𝑥 2 2
= … (1)
𝑥 𝑦

It is of variable separable type.

𝑝2 1 + 𝑥 2 𝑎2 𝑥 2 𝑎𝑥
𝐿𝑒𝑡 = 𝑎2 𝑝2 = 𝑝= … (2)
𝑥2 1 + 𝑥2 1 + 𝑥2
𝑞
𝐿𝑒𝑡 =𝑎 𝑞 = 𝑎𝑦 … (3)
𝑦

𝑑𝑧 = 𝑝𝑑𝑥 + 𝑞𝑑𝑦 … (4)

Substituting (2) and (3) in (4), we get


𝑎𝑥
𝑑𝑧 = 𝑑𝑥 + 𝑎𝑦𝑑𝑦
1 + 𝑥2

Integrating on both sides, we get

1 2𝑎𝑥
𝑑𝑧 = 𝑑𝑥 + 𝑎𝑦𝑑𝑦
2 1 + 𝑥2

𝑎 𝑦2
𝑧= 2 1 + 𝑥2 + 𝑎 + 𝑐
2 2
𝑦2
𝑧 = 𝑎 1+ 𝑥2 + 𝑎 + 𝑐 𝑖𝑠 𝑡𝑕𝑒 𝑐𝑜𝑚𝑝𝑙𝑒𝑡𝑒 𝑠𝑜𝑙𝑢𝑡𝑖𝑜𝑛 𝑜𝑓 (1).
2

10. Solve 𝑧 2 𝑝2 + 𝑞 2 = 𝑥 2 + 𝑦 2 .

Solution:
2 2
𝑧𝑝 + 𝑧𝑞 = 𝑥 2 + 𝑦 2 … (1)

It is of the form 𝐹 𝑧 𝑘 𝑝, 𝑧 𝑘 𝑞 = 0
Here 𝑘 = 1, 𝑃𝑢𝑡 𝑍 = 𝑧 𝑘+1 = 𝑧 2 … (2)

𝜕𝑍 𝜕𝑍 𝜕𝑧 𝑃
𝑃= = = 2𝑧𝑝 𝑧𝑝 = … (3)
𝜕𝑥 𝜕𝑧 𝜕𝑥 2
𝜕𝑍 𝜕𝑍 𝜕𝑧 𝑄
𝑄= = = 2𝑧𝑞 𝑧𝑞 = … (4)
𝜕𝑦 𝜕𝑧 𝜕𝑦 2

Substituting (3) and (4) in (1), we get


2 2
𝑃 𝑄
+ = 𝑥2 + 𝑦2
2 2

𝑃2 + 𝑄2 = 4𝑥 2 + 4𝑦 2

It is of variable separable type

𝑃2 − 4𝑥 2 = 4𝑦 2 − 𝑄2 = 𝑎2

𝑎 2
𝑃2 − 4𝑥 2 = 𝑎2 𝑃2 = 4𝑥 2 + 𝑎2 𝑃= 2 𝑥 2 + … (5)
2

𝑎 2
4𝑦 2 − 𝑄2 = 𝑎2 4𝑦 2 − 𝑎2 = 𝑄2 𝑄 = 2 𝑦2 − … (6)
2

𝑑𝑍 = 𝑃𝑑𝑥 + 𝑄𝑑𝑦 … (7)

𝑎 2 𝑎 2
𝑑𝑍 = 2 𝑥 2 + 𝑑𝑥 + 2 𝑦 2 − 𝑑𝑦 𝑓𝑟𝑜𝑚 5 , (6)𝑎𝑛𝑑 (7)
2 2

Integrating on both sides, we get

𝑎 2 𝑎 2
𝑑𝑍 = 2 𝑥2 + 𝑑𝑥 + 2 𝑦2 − 𝑑𝑦
2 2

𝑥 𝑎 2 𝑎2 2𝑥 𝑦 𝑎 2 2𝑦
𝑍=2 𝑥2 + + 𝑠𝑖𝑛𝑕−1 + 𝑦2 − − 𝑐𝑜𝑠𝑕−1 +𝑐
2 2 8 𝑎 2 2 𝑎

𝑥 𝑎 2 𝑎2 2𝑥 𝑦 𝑎 2 2𝑦
𝑧2 = 2 𝑥2 + + 𝑠𝑖𝑛𝑕−1 + 𝑦2 − − 𝑐𝑜𝑠𝑕−1 +𝑐
2 2 8 𝑎 2 2 𝑎

𝑓𝑟𝑜𝑚 (2)

is the general solution of (1).


11. Solve 𝑥 2 + 𝑦 2 + 𝑦𝑧 𝑝 + 𝑥 2 + 𝑦 2 − 𝑥𝑧 𝑞 = 𝑧(𝑥 + 𝑦)

Solution:

The equation is of the form 𝑃𝑝 + 𝑄𝑞 = 𝑅

Here 𝑃 = 𝑥 2 + 𝑦 2 + 𝑦𝑧 , 𝑄 = 𝑥 2 + 𝑦 2 − 𝑥𝑧, 𝑅 = 𝑧(𝑥 + 𝑦)

𝑑𝑥 𝑑𝑦 𝑑𝑧
= 2 =
𝑥2 2
+ 𝑦 + 𝑦𝑧 2
𝑥 + 𝑦 − 𝑥𝑧 𝑧(𝑥 + 𝑦)

𝑑𝑥 − 𝑑𝑦 𝑑𝑧
=
𝑥 2 + 𝑦 2 + 𝑦𝑧 − 𝑥 2 − 𝑦 2 + 𝑥𝑧 𝑧(𝑥 + 𝑦)

𝑑𝑥 − 𝑑𝑦 𝑑𝑧
= 𝑑𝑥 − 𝑑𝑦 = 𝑑𝑧
𝑦𝑧 + 𝑥𝑧 𝑧(𝑥 + 𝑦)

Integrating on both sides, we get

𝑥 − 𝑦 = 𝑧 + 𝑐1 𝑐1 = 𝑥 − 𝑦 − 𝑧

𝑥𝑑𝑥 + 𝑦𝑑𝑦 𝑑𝑧
=
𝑥 3 + 𝑥𝑦 2 + 𝑥𝑦𝑧 + 𝑦𝑥 2 + 𝑦 3 − 𝑥𝑦𝑧 𝑧(𝑥 + 𝑦)

𝑥𝑑𝑥 + 𝑦𝑑𝑦 𝑑𝑧
=
𝑥3 2 2
+ 𝑥𝑦 + 𝑦𝑥 + 𝑦 3 𝑧(𝑥 + 𝑦)

𝑥𝑑𝑥 + 𝑦𝑑𝑦 𝑑𝑧
=
𝑥 3 + 𝑦𝑥 2 + 𝑥𝑦 2 + 𝑦 3 𝑧 𝑥 + 𝑦

𝑥𝑑𝑥 + 𝑦𝑑𝑦 𝑑𝑧
=
𝑥2 2
𝑥+𝑦 +𝑦 𝑥+𝑦 𝑧(𝑥 + 𝑦)

𝑥𝑑𝑥 + 𝑦𝑑𝑦 𝑑𝑧
=
𝑥 + 𝑦 𝑥2 + 𝑦2 𝑧(𝑥 + 𝑦)

𝑥𝑑𝑥 + 𝑦𝑑𝑦 𝑑𝑧
=
𝑥2 + 𝑦2 𝑧

Integrating on both sides, we get

1
log 𝑥 2 + 𝑦 2 + 𝑧 2 = log 𝑧 + 𝑙𝑜𝑔 𝑐2
2

log 𝑥 2 + 𝑦 2 = 2 log 𝑧 𝑐2

log 𝑥 2 + 𝑦 2 = log 𝑧 2 𝑐2 2

𝑥2 + 𝑦2
𝑥 2 + 𝑦 2 = 𝑧 2 𝑐3 𝑐3 =
𝑧2
The general solution is

Φ 𝑐1 , 𝑐3 = 0

𝑥2 + 𝑦2
Φ 𝑥 − 𝑦 − 𝑧, =0
𝑧2

12. Solve 𝑦 2 + 𝑧 2 𝑝 − 𝑥𝑦𝑞 + 𝑥𝑧 = 0

Solution:

𝑦 2 + 𝑧 2 𝑝 − 𝑥𝑦𝑞 = −𝑥𝑧

The equation is of the form 𝑃𝑝 + 𝑄𝑞 = 𝑅

Here 𝑃 = 𝑦 2 + 𝑧 2 , 𝑄 = −𝑥𝑦, 𝑅 = −𝑥𝑧

𝑑𝑥 𝑑𝑦 𝑑𝑧
= =
𝑦2 +𝑧 2 −𝑥𝑦 −𝑥𝑧

𝑑𝑦 𝑑𝑧
=
−𝑥𝑦 −𝑥𝑧

𝑑𝑦 𝑑𝑧
=
𝑦 𝑧

Integrating on both sides, we get

log 𝑦 = log 𝑧 + log 𝑐1

log 𝑦 − log 𝑧 = log 𝑐1


𝑦 𝑦
log = log 𝑐1 𝑐1 =
𝑧 𝑧
𝑥𝑑𝑥 + 𝑦𝑑𝑦 + 𝑧𝑑𝑧 𝑥𝑑𝑥 + 𝑦𝑑𝑦 + 𝑧𝑑𝑧
=
𝑥𝑦 2 + 𝑥𝑧 2 − 𝑥𝑦 2 − 𝑥𝑧 2 0

𝑥𝑑𝑥 + 𝑦𝑑𝑦 + 𝑧𝑑𝑧 = 0

𝑥2 𝑦2 𝑧2
+ + = 𝑐2 𝑐3 = 𝑥 2 +𝑦 2 + 𝑧 2
2 2 2

The general solution is

Φ 𝑐1 , 𝑐3 = 0
𝑦 2 2
Φ , 𝑥 +𝑦 + 𝑧 2 =0
𝑧

13. Solve 𝑥 2 − 𝑦 2 − 𝑧 2 𝑝 + 2𝑥𝑦𝑞 − 2𝑥𝑧 = 0


Solution:

𝑥 2 − 𝑦 2 − 𝑧 2 𝑝 + 2𝑥𝑦𝑞 = 2𝑥𝑧

The equation is of the form 𝑃𝑝 + 𝑄𝑞 = 𝑅

Here 𝑃 = 𝑥 2 − 𝑦 2 − 𝑧 2 , 𝑄 = 2𝑥𝑦, 𝑅 = 2𝑥𝑧

𝑑𝑥 𝑑𝑦 𝑑𝑧
= =
𝑥2 2
−𝑦 −𝑧 2 2𝑥𝑦 2𝑥𝑧

𝑑𝑦 𝑑𝑧
=
2𝑥𝑦 2𝑥𝑧

𝑑𝑦 𝑑𝑧
=
𝑦 𝑧

Integrating on both sides, we get

log 𝑦 = log 𝑧 + log 𝑐1

log 𝑦 − log 𝑧 = log 𝑐1


𝑦 𝑦
log = log 𝑐1 𝑐1 =
𝑧 𝑧
𝑑𝑥 𝑑𝑦 𝑑𝑧
= =
𝑥2 − 𝑦2 − 𝑧2 2𝑥𝑦 2𝑥𝑧

𝑥𝑑𝑥 + 𝑦𝑑𝑦 + 𝑧𝑑𝑧 𝑑𝑧


=
𝑥3 2 2 2
− 𝑥𝑦 − 𝑥𝑧 + 2 𝑥𝑦 + 2𝑥𝑧 2 𝑧(𝑦 – 𝑥 2 )
2

𝑥𝑑𝑥 + 𝑦𝑑𝑦 𝑑𝑦
=
𝑥 3 + 𝑥𝑦 2 + 𝑥𝑧 2 2𝑥𝑦

𝑥𝑑𝑥 + 𝑦𝑑𝑦 + 𝑧𝑑𝑧 𝑑𝑦


2 2 2
=
𝑥 𝑥 +𝑦 + 𝑧 2𝑥𝑦

2𝑥𝑑𝑥 + 2𝑦𝑑𝑦 + 2𝑧𝑑𝑧 𝑑𝑦


=
𝑥 2 +𝑦 2 + 𝑧 2 𝑦

Integrating on both sides, we get

2𝑥𝑑𝑥 + 2𝑦𝑑𝑦 + 2𝑧𝑑𝑧 𝑑𝑦


=
𝑥 2 +𝑦 2 + 𝑧 2 𝑦

log 𝑥 2 +𝑦 2 + 𝑧 2 = log 𝑦 + 𝑙𝑜𝑔 𝑐2

log 𝑥 2 +𝑦 2 + 𝑧 2 = log 𝑦
2 2 2
𝑥 2 +𝑦 2 + 𝑧 2
𝑥 +𝑦 + 𝑧 = 𝑦𝑐2 𝑐2 =
𝑦

The general solution is

Φ 𝑐1 , 𝑐2 = 0

𝑦 𝑥 2 +𝑦 2 + 𝑧 2
Φ , =0
𝑧 𝑦

2
14. Solve 𝐷 2 − 𝐷𝐷 ′ − 2𝐷 ′ 𝑧 = 2𝑥 + 3𝑦 + 𝑒 3𝑥+4𝑦

Solution:

The auxiliary equation is 𝑚2 − 𝑚 − 2 = 0

𝑚 = −1,2

The complementary function CF is 𝑧 = 𝜙1 𝑦 − 𝑥 + 𝜙2 𝑦 + 2𝑥

To find particular integral:

2𝑥 + 3𝑦
𝑃. 𝐼1 =
𝐷 2 − 𝐷𝐷 ′ − 2𝐷 ′ 2
2𝑥 + 3𝑦 2𝑥 + 3𝑦
= =
𝐷′ 𝐷′ 2 𝐷′ 𝐷′ 2
𝐷2 1− −2 2 𝐷2 1 − +2 2
𝐷 𝐷 𝐷 𝐷

−1
𝐷′ 𝐷′ 2
1− +2 2 2𝑥 + 3𝑦
𝐷 𝐷
=
𝐷2

𝐷′ 𝐷′ 2
1+ +2 2 2𝑥 + 3𝑦
𝐷 𝐷
−1
= 1+𝑥 = 1 − 𝑥 + 𝑥2 − ⋯
𝐷2

2
𝐷′ 𝐷′
2𝑥 + 3𝑦 + 2𝑥 + 3𝑦 + 2 2 2𝑥 + 3𝑦
𝐷 𝐷
=
𝐷2
3
2𝑥 + 3𝑦 +
𝐷 1 3
= = 2𝑥 + 3𝑦 +
𝐷2 𝐷2 𝐷3
2𝑥 3 3𝑥 2 𝑦 3𝑥 3 5𝑥 3 3𝑥 2 𝑦 1 𝑚 𝑚! 𝑥 𝑚 +𝑛
𝑃. 𝐼1 = + + = + 𝑥 =
3! 2! 3! 3! 2! 𝐷𝑛 𝑚+𝑛 !

𝑒 3𝑥+4𝑦
𝑃. 𝐼2 =
𝐷 2 − 𝐷𝐷 ′ − 2𝐷 ′ 2
𝑒 3𝑥+4𝑦 𝑒 3𝑥+4𝑦
= 2 =
3 − 3 (4) − 2(4)2 9 − 12 − 32

𝑒 3𝑥+4𝑦
𝑃. 𝐼2 = −
35
The general solution is

𝐶𝐹 + 𝑃. 𝐼1 + 𝑃. 𝐼2

5𝑥 3 3𝑥 2 𝑦 𝑒 3𝑥+4𝑦
𝑧 = 𝜙1 𝑦 − 𝑥 + 𝜙2 𝑦 + 2𝑥 + + −
3! 2! 35
2
15. Solve 𝐷 2 + 3𝐷𝐷 ′ − 4𝐷 ′ 𝑧 = sin 𝑦

Solution:

The auxiliary equation is 𝑚2 + 3𝑚 − 4 = 0

𝑚 = −4,1

The complementary function CF is 𝑧 = 𝜙1 𝑦 − 4𝑥 + 𝜙2 𝑦 + 𝑥

To find particular integral:


sin 𝑦
𝑃. 𝐼 =
𝐷 2 + 3𝐷𝐷 ′ − 4𝐷 ′ 2
𝐼𝑚𝑎𝑔𝑖𝑛𝑎𝑟𝑦 𝑝𝑎𝑟𝑡 𝑜𝑓 𝑒 𝑖𝑦
=
𝐷 2 + 3𝐷𝐷 ′ − 4𝐷 ′ 2
𝐼𝑚𝑎𝑔𝑖𝑛𝑎𝑟𝑦 𝑝𝑎𝑟𝑡 𝑜𝑓 𝑒 𝑖𝑦 𝐼𝑚𝑎𝑔𝑖𝑛𝑎𝑟𝑦 𝑝𝑎𝑟𝑡 𝑜𝑓 𝑒 𝑖𝑦
= =
(0)2 + 3 0 (𝑖) − 4(𝑖)2 4
sin 𝑦
𝑃. 𝐼 =
4
The general solution is 𝐶. 𝐹 + 𝑃. 𝐼
sin 𝑦
𝑧 = 𝜙1 𝑦 − 4𝑥 + 𝜙2 𝑦 + 𝑥 +
4
2
16. Solve 𝐷 2 + 𝐷𝐷 ′ − 6𝐷 ′ 𝑧 = 𝑥 2 𝑦 + 𝑒 3𝑥+𝑦

Solution:

The auxiliary equation is 𝑚2 + 𝑚 − 6 = 0

𝑚 = −3,2

The complementary function CF is 𝑧 = 𝜙1 𝑦 − 3𝑥 + 𝜙2 𝑦 + 2𝑥

To find particular integral:

𝑥2𝑦
𝑃. 𝐼1 =
𝐷 2 + 𝐷𝐷 ′ − 6𝐷 ′ 2
𝑥2𝑦 𝑥2𝑦
= =
𝐷′ 𝐷′ 2 𝐷′ 𝐷′ 2
𝐷2 1+ −6 2 𝐷2 1 + −6 2
𝐷 𝐷 𝐷 𝐷

−1
2
𝐷′ 𝐷′
1+ −6 2 𝑥2𝑦
𝐷 𝐷
=
𝐷2
2
𝐷′ 𝐷′
1− −6 2 𝑥2𝑦
𝐷 𝐷
−1
= 1+𝑥 = 1 − 𝑥 + 𝑥2 − ⋯
𝐷2

2𝐷′ 2 𝐷′ 2 2
𝑥 𝑦− 𝑥 𝑦−6 2 𝑥 𝑦
𝐷 𝐷
=
𝐷2

𝑥2
𝑥2𝑦 −
𝐷 1 2 1 2
= = 𝑥 𝑦 − 𝑥
𝐷2 𝐷2 𝐷3
2𝑥 4 𝑦 2𝑥 5 𝑥 4 𝑦 𝑥 5 1 𝑚 𝑚! 𝑥 𝑚 +𝑛
𝑃. 𝐼1 = − = − 𝑥 =
4! 5! 12 60 𝐷𝑛 𝑚+𝑛 !

𝑒 3𝑥+𝑦 𝑒 3𝑥+𝑦
𝑃. 𝐼2 = = 2
𝐷 2 + 𝐷𝐷 ′ − 6𝐷 ′ 2 3 + 3 1 − 6(1)2

𝑒 3𝑥+𝑦
𝑃. 𝐼2 =
6
The general solution is

𝐶𝐹 + 𝑃. 𝐼1 + 𝑃. 𝐼2

𝑥4𝑦 𝑥5 𝑒 3𝑥+𝑦
𝑧 = 𝜙1 𝑦 − 3𝑥 + 𝜙2 𝑦 + 2𝑥 + − +
12 60 6
17. Solve 𝐷 3 − 2𝐷 2 𝐷 ′ 𝑧 = 4 sin⁡
(𝑥 + 𝑦) + 𝑒 𝑥+2𝑦

The auxiliary equation is 𝑚3 − 2𝑚2 = 0 𝑚2 𝑚 − 2 = 0

𝑚 = 0,0,2

The complementary function CF is 𝑧 = 𝜙1 𝑦 + 𝑥𝜙2 𝑦 + 𝜙3 𝑦 + 2𝑥

To find particular integral:

4 sin⁡
(𝑥 + 𝑦)
𝑃. 𝐼1 =
𝐷 3 − 2𝐷 2 𝐷 ′
4 𝐼𝑚𝑎𝑔𝑖𝑛𝑎𝑟𝑦 𝑝𝑎𝑟𝑡 𝑜𝑓 𝑒 𝑖(𝑥+𝑦)
=
𝐷 3 − 2𝐷 2 𝐷 ′
4 𝐼𝑚𝑎𝑔𝑖𝑛𝑎𝑟𝑦 𝑝𝑎𝑟𝑡 𝑜𝑓 𝑒 𝑖(𝑥+𝑦) 4 𝐼𝑚𝑎𝑔𝑖𝑛𝑎𝑟𝑦 𝑝𝑎𝑟𝑡 𝑜𝑓 𝑒 𝑖(𝑥+𝑦)
= =
(𝑖)3 − 2 𝑖 2 (𝑖) −𝑖 + 2(𝑖)

4 𝐼𝑚𝑎𝑔𝑖𝑛𝑎𝑟𝑦 𝑝𝑎𝑟𝑡 𝑜𝑓 𝑒 𝑖(𝑥+𝑦)


=
𝑖
1
= −4 𝐼𝑚𝑎𝑔𝑖𝑛𝑎𝑟𝑦 𝑝𝑎𝑟𝑡 𝑜𝑓 𝑖𝑒 𝑖(𝑥+𝑦) ∵ = −𝑖
𝑖
= −4 𝐼𝑚𝑎𝑔𝑖𝑛𝑎𝑟𝑦 𝑝𝑎𝑟𝑡 𝑜𝑓 𝑖(cos 𝑥 + 𝑦 + 𝑖𝑠𝑖𝑛 (𝑥 + 𝑦))

= −4 𝐼𝑚𝑎𝑔𝑖𝑛𝑎𝑟𝑦 𝑝𝑎𝑟𝑡 𝑜𝑓 (𝑖 cos 𝑥 + 𝑦 + 𝑖 2 𝑠𝑖𝑛 (𝑥 + 𝑦))


𝑃. 𝐼1 = −4 cos 𝑥 + 𝑦

𝑒 𝑥+2𝑦
𝑃. 𝐼2 = 3
𝐷 − 2𝐷 2 𝐷 ′
𝑒 𝑥+2𝑦 𝑒 𝑥+2𝑦
𝑃. 𝐼2 = 3 =−
1 − 2 1 2 (2) 3

The general solution is

𝐶𝐹 + 𝑃. 𝐼1 + 𝑃. 𝐼2

𝑒 𝑥+2𝑦
𝑧 = 𝜙1 𝑦 + 𝜙2 𝑦 + 2𝑥 + 𝜙3 𝑦 − 2𝑥 − 4 cos 𝑥 + 𝑦 −
3
2
18. Solve 𝐷 2 − 𝐷𝐷 ′ − 20𝐷 ′ 𝑧 = sin(4𝑥 − 𝑦) + 𝑒 5𝑥+𝑦

Solution:

The auxiliary equation is 𝑚2 − 𝑚 − 20 = 0

𝑚 = −4,5

The complementary function CF is 𝑧 = 𝜙1 𝑦 − 4𝑥 + 𝜙2 𝑦 + 5𝑥

To find particular integral:

sin(4𝑥 − 𝑦)
𝑃. 𝐼1 =
𝐷 2 − 𝐷𝐷 ′ − 20𝐷 ′ 2
𝐼𝑚𝑎𝑔𝑖𝑛𝑎𝑟𝑦 𝑝𝑎𝑟𝑡 𝑜𝑓𝑒 𝑖(4𝑥−𝑦)
=
𝐷 2 − 𝐷𝐷 ′ − 20𝐷 ′ 2
𝐼𝑚𝑎𝑔𝑖𝑛𝑎𝑟𝑦 𝑝𝑎𝑟𝑡 𝑜𝑓𝑒 𝑖(4𝑥−𝑦)
=
4𝑖 2 − 4𝑖 −𝑖 − 20 −𝑖 2

𝐼𝑚𝑎𝑔𝑖𝑛𝑎𝑟𝑦 𝑝𝑎𝑟𝑡 𝑜𝑓𝑒 𝑖(4𝑥−𝑦)


=
−16 − 4 + 20
𝑖𝑓 𝑑𝑒𝑛𝑜𝑚𝑖𝑛𝑎𝑡𝑜𝑟 𝑏𝑒𝑐𝑜𝑚𝑒𝑠 𝑧𝑒𝑟𝑜 𝑡𝑕𝑒𝑛 𝑚𝑢𝑙𝑖𝑝𝑙𝑦 𝑛𝑢𝑚𝑒𝑟𝑎𝑡𝑜𝑟 𝑏𝑦 𝑥
𝑎𝑛𝑑 𝑑𝑖𝑓𝑓𝑒𝑟𝑒𝑛𝑡𝑖𝑎𝑡𝑖𝑛𝑔 𝑑𝑒𝑛𝑜𝑚𝑖𝑛𝑎𝑡𝑜𝑟 𝑝𝑎𝑟𝑡𝑖𝑎𝑙𝑙𝑦 𝑤𝑖𝑡𝑕 𝑟𝑒𝑠𝑝𝑒𝑐𝑡 𝑡𝑜 𝐷

𝐼𝑚𝑎𝑔𝑖𝑛𝑎𝑟𝑦 𝑝𝑎𝑟𝑡 𝑜𝑓𝑥𝑒 𝑖(4𝑥−𝑦)


=
2𝐷 − 𝐷′
𝐼𝑚𝑎𝑔𝑖𝑛𝑎𝑟𝑦 𝑝𝑎𝑟𝑡 𝑜𝑓𝑥𝑒 𝑖(4𝑥−𝑦)
=
2 4𝑖 − −𝑖

𝐼𝑚𝑎𝑔𝑖𝑛𝑎𝑟𝑦 𝑝𝑎𝑟𝑡 𝑜𝑓𝑥𝑒 𝑖(4𝑥−𝑦)


=
9𝑖
𝐼𝑚𝑎𝑔𝑖𝑛𝑎𝑟𝑦 𝑝𝑎𝑟𝑡 𝑜𝑓𝑖𝑥𝑒 𝑖(4𝑥−𝑦) 1
=− ∵ = −𝑖
9 𝑖
𝐼𝑚𝑎𝑔𝑖𝑛𝑎𝑟𝑦 𝑝𝑎𝑟𝑡 𝑜𝑓𝑖𝑥 cos 4𝑥 − 𝑦 + 𝑖𝑠𝑖𝑛 4𝑥 − 𝑦
=−
9
𝐼𝑚𝑎𝑔𝑖𝑛𝑎𝑟𝑦 𝑝𝑎𝑟𝑡 𝑜𝑓 𝑖𝑥cos 4𝑥 − 𝑦 + 𝑥𝑖 2 𝑠𝑖𝑛 4𝑥 − 𝑦
=−
9
𝑥cos 4𝑥 − 𝑦
𝑃. 𝐼1 = −
9
𝑒 5𝑥+𝑦
𝑃. 𝐼2 =
𝐷 2 − 𝐷𝐷 ′ − 20𝐷 ′ 2
𝑒 5𝑥+𝑦
= 2
5 − 5 (1) − 20(1)2
𝑖𝑓 𝑑𝑒𝑛𝑜𝑚𝑖𝑛𝑎𝑡𝑜𝑟 𝑏𝑒𝑐𝑜𝑚𝑒𝑠 𝑧𝑒𝑟𝑜 𝑡𝑕𝑒𝑛 𝑚𝑢𝑙𝑖𝑝𝑙𝑦 𝑛𝑢𝑚𝑒𝑟𝑎𝑡𝑜𝑟 𝑏𝑦 𝑥
𝑎𝑛𝑑 𝑑𝑖𝑓𝑓𝑒𝑟𝑒𝑛𝑡𝑖𝑎𝑡𝑖𝑛𝑔 𝑑𝑒𝑛𝑜𝑚𝑖𝑛𝑎𝑡𝑜𝑟 𝑝𝑎𝑟𝑡𝑖𝑎𝑙𝑙𝑦 𝑤𝑖𝑡𝑕 𝑟𝑒𝑠𝑝𝑒𝑐𝑡 𝑡𝑜 𝐷

𝑒 5𝑥+𝑦 𝑒 5𝑥+𝑦
= =
2𝐷 − 𝐷′ 2(5) − 1

𝑒 5𝑥+𝑦
𝑃. 𝐼2 =
9
The general solution is

𝐶𝐹 + 𝑃. 𝐼1 + 𝑃. 𝐼2

𝑥cos 4𝑥 − 𝑦 𝑒 5𝑥+𝑦
𝑧 = 𝜙1 𝑦 − 4𝑥 + 𝜙2 𝑦 + 5𝑥 − +
9 9

2
19. Solve 𝐷 2 − 4𝐷𝐷 ′ + 4𝐷 ′ 𝑧 = 𝑥𝑦 + 𝑒 2𝑥+𝑦
Solution:

The auxiliary equation is 𝑚2 − 4𝑚 + 4 = 0

𝑚 = 2,2

The complementary function CF is 𝑧 = 𝜙1 𝑦 + 2𝑥 + 𝑥𝜙2 𝑦 + 2𝑥

To find particular integral:


𝑥𝑦
𝑃. 𝐼1 =
𝐷 2 − 4𝐷𝐷 ′ + 4𝐷 ′ 2
𝑥𝑦 𝑥𝑦
= =
𝐷′ 𝐷′ 2 𝐷′ 𝐷′ 2
𝐷2 1−4 +4 2 𝐷2 1− 4 −4 2
𝐷 𝐷 𝐷 𝐷

−1
𝐷′ 𝐷′ 2
1− 4 −4 2 𝑥𝑦
𝐷 𝐷
=
𝐷2

𝐷′ 𝐷′ 2
1+ 4 −4 2 𝑥𝑦
𝐷 𝐷
−1
= 1−𝑥 = 1 + 𝑥 + 𝑥2 + ⋯
𝐷2

𝐷′ 𝐷′ 2
𝑥𝑦 + 4 𝑥𝑦 − 4 2 𝑥𝑦
𝐷 𝐷
=
𝐷2
𝑥
𝑥𝑦 + 4 1 4
𝐷
= = 𝑥𝑦 + 𝑥
𝐷2 𝐷2 𝐷3
𝑥 3 𝑦 4𝑥 4 𝑥 3 𝑦 𝑥 4 1 𝑚 𝑚! 𝑥 𝑚 +𝑛
𝑃. 𝐼1 = + = + 𝑥 =
6 24 6 6 𝐷𝑛 𝑚+𝑛 !

𝑒 2𝑥+𝑦 𝑒 2𝑥+𝑦
𝑃. 𝐼2 = = 2
𝐷 2 − 4𝐷𝐷 ′ + 4𝐷 ′ 2 2 − 4 2 (1) + 4(1)2

𝑥𝑒 2𝑥+𝑦 𝑥𝑒 2𝑥+𝑦
= =
2𝐷 − 4𝐷 ′ 2(2) − 4(1)
𝑖𝑓 𝑑𝑒𝑛𝑜𝑚𝑖𝑛𝑎𝑡𝑜𝑟 𝑏𝑒𝑐𝑜𝑚𝑒𝑠 𝑧𝑒𝑟𝑜 𝑡𝑕𝑒𝑛 𝑚𝑢𝑙𝑖𝑝𝑙𝑦 𝑛𝑢𝑚𝑒𝑟𝑎𝑡𝑜𝑟 𝑏𝑦 𝑥
𝑎𝑛𝑑 𝑑𝑖𝑓𝑓𝑒𝑟𝑒𝑛𝑡𝑖𝑎𝑡𝑖𝑛𝑔 𝑑𝑒𝑛𝑜𝑚𝑖𝑛𝑎𝑡𝑜𝑟 𝑝𝑎𝑟𝑡𝑖𝑎𝑙𝑙𝑦 𝑤𝑖𝑡𝑕 𝑟𝑒𝑠𝑝𝑒𝑐𝑡 𝑡𝑜 𝐷

𝑥 2 𝑒 2𝑥+𝑦
𝑃. 𝐼2 =
2
The general solution is

𝐶𝐹 + 𝑃. 𝐼1 + 𝑃. 𝐼2

𝑥 3 𝑦 𝑥 4 𝑥 2 𝑒 2𝑥+𝑦
𝑧 = 𝜙1 𝑦 + 2𝑥 + 𝑥𝜙2 𝑦 + 2𝑥 + + +
6 6 2
2
20. Solve 𝐷 2 − 6𝐷𝐷 ′ + 5𝐷 ′ 𝑧 = 𝑥𝑦 + 𝑒 𝑥 sin 𝑕𝑦

Solution:

The auxiliary equation is 𝑚2 − 6𝑚 + 5 = 0

𝑚 = 1,5

The complementary function CF is 𝑧 = 𝜙1 𝑦 + 𝑥 + 𝜙2 𝑦 + 5𝑥

To find particular integral:


𝑥𝑦
𝑃. 𝐼1 =
𝐷 2 − 6𝐷𝐷 ′ + 5𝐷 ′ 2
𝑥𝑦 𝑥𝑦
= =
𝐷′ 𝐷′ 2 𝐷′ 𝐷′ 2
𝐷2 1−6 +5 2 𝐷2 1− 6 −5 2
𝐷 𝐷 𝐷 𝐷

−1
2
𝐷′ 𝐷′
1− 6 −5 2 𝑥𝑦
𝐷 𝐷
=
𝐷2

𝐷′ 𝐷′ 2
1+ 6 −5 2 𝑥𝑦
𝐷 𝐷
−1
= 1−𝑥 = 1 + 𝑥 + 𝑥2 + ⋯
𝐷2

𝐷′ 𝐷′ 2
𝑥𝑦 + 6 𝑥𝑦 − 5 2 𝑥𝑦
𝐷 𝐷
=
𝐷2
𝑥
𝑥𝑦 + 6 1 6
𝐷
= = 𝑥𝑦 + 𝑥
𝐷2 𝐷2 𝐷3
𝑥 3 𝑦 6𝑥 4 𝑥 3 𝑦 𝑥 4 1 𝑚 𝑥 𝑚 +𝑛
𝑃. 𝐼1 = + = + 𝑥 =
6 24 6 4 𝐷𝑛 𝑚+𝑛 !

𝑒 𝑦 − 𝑒 −𝑦
𝑥
𝑒 𝑥 sin 𝑕𝑦 𝑒
2
𝑃. 𝐼2 = = 2
𝐷 2 − 6𝐷𝐷 ′ + 5𝐷 ′ 2 𝐷 − 6𝐷𝐷 ′ + 5𝐷 ′ 2
𝑒 𝑥+𝑦 𝑒 𝑥 −𝑦
= −
2 𝐷 2 − 6𝐷𝐷 ′ + 5𝐷 ′ 2 2 𝐷 2 − 6𝐷𝐷 ′ + 5𝐷 ′ 2

𝑒 𝑥+𝑦 𝑒 𝑥−𝑦
= −
2 12 − 6 1 (1) + 5(1)2 2 12 − 6 1 (−1) + 5 −1 2

𝑥𝑒 𝑥+𝑦 𝑒 𝑥−𝑦
= −
2 2𝐷 − 6𝐷 ′ 2 11
𝑖𝑓 𝑑𝑒𝑛𝑜𝑚𝑖𝑛𝑎𝑡𝑜𝑟 𝑏𝑒𝑐𝑜𝑚𝑒𝑠 𝑧𝑒𝑟𝑜 𝑡𝑕𝑒𝑛 𝑚𝑢𝑙𝑖𝑝𝑙𝑦 𝑛𝑢𝑚𝑒𝑟𝑎𝑡𝑜𝑟 𝑏𝑦 𝑥
𝑎𝑛𝑑 𝑑𝑖𝑓𝑓𝑒𝑟𝑒𝑛𝑡𝑖𝑎𝑡𝑖𝑛𝑔 𝑑𝑒𝑛𝑜𝑚𝑖𝑛𝑎𝑡𝑜𝑟 𝑝𝑎𝑟𝑡𝑖𝑎𝑙𝑙𝑦 𝑤𝑖𝑡𝑕 𝑟𝑒𝑠𝑝𝑒𝑐𝑡 𝑡𝑜 𝐷

𝑥𝑒 𝑥+𝑦 𝑒 𝑥−𝑦 𝑥𝑒 𝑥+𝑦 𝑒 𝑥 −𝑦


𝑃. 𝐼2 = − = −
2 2(1) − 6(1) 22 −8 22

The general solution is

𝐶𝐹 + 𝑃. 𝐼1 + 𝑃. 𝐼2

𝑥 3 𝑦 𝑥 4 𝑥𝑒 𝑥 +𝑦 𝑒 𝑥−𝑦
𝑧 = 𝜙1 𝑦 + 𝑥 + 𝜙2 𝑦 + 5𝑥 + + − −
6 4 8 22
2
21. Solve 𝐷 2 − 𝐷𝐷 ′ − 30𝐷 ′ 𝑧 = 𝑥𝑦 + 𝑒 6𝑥+𝑦

Solution:

The auxiliary equation is 𝑚2 − 𝑚 − 30 = 0

𝑚 = −5,6

The complementary function CF is 𝑧 = 𝜙1 𝑦 − 5𝑥 + 𝜙2 𝑦 + 6𝑥

To find particular integral:


𝑥𝑦
𝑃. 𝐼1 =
𝐷 2 − 𝐷𝐷 ′ − 30𝐷 ′ 2
𝑥𝑦 𝑥𝑦
= =
𝐷′ 𝐷′ 2 𝐷′ 𝐷′ 2
𝐷2 1 − − 30 2 𝐷2 1− + 30 2
𝐷 𝐷 𝐷 𝐷

−1
𝐷′ 𝐷′ 2
1− + 30 2 𝑥𝑦
𝐷 𝐷
=
𝐷2
2
𝐷′ 𝐷′
1+ + 30 2 𝑥𝑦
𝐷 𝐷
−1
= 1−𝑥 = 1 + 𝑥 + 𝑥2 + ⋯
𝐷2
2
𝐷′ 𝐷′
𝑥𝑦 + 𝑥𝑦 + 30 2 𝑥𝑦
𝐷 𝐷
=
𝐷2

1
𝑥𝑦 + 𝑥
𝐷 1 𝑥
= = 𝑥𝑦 +
𝐷2 𝐷2 𝐷3
𝑥3𝑦 𝑥4 1 𝑚 𝑚! 𝑥 𝑚 +𝑛
𝑃. 𝐼1 = + 𝑥 =
3! 4! 𝐷𝑛 𝑚+𝑛 !

𝑒 6𝑥+𝑦
𝑃. 𝐼2 =
𝐷 2 − 𝐷𝐷 ′ − 30𝐷 ′ 2
𝑒 6𝑥+𝑦
= 2
6 − 6 (1) − 30(1)2
𝑖𝑓 𝑑𝑒𝑛𝑜𝑚𝑖𝑛𝑎𝑡𝑜𝑟 𝑏𝑒𝑐𝑜𝑚𝑒𝑠 𝑧𝑒𝑟𝑜 𝑡𝑕𝑒𝑛 𝑚𝑢𝑙𝑖𝑝𝑙𝑦 𝑛𝑢𝑚𝑒𝑟𝑎𝑡𝑜𝑟 𝑏𝑦 𝑥
𝑎𝑛𝑑 𝑑𝑖𝑓𝑓𝑒𝑟𝑒𝑛𝑡𝑖𝑎𝑡𝑖𝑛𝑔 𝑑𝑒𝑛𝑜𝑚𝑖𝑛𝑎𝑡𝑜𝑟 𝑝𝑎𝑟𝑡𝑖𝑎𝑙𝑙𝑦 𝑤𝑖𝑡𝑕 𝑟𝑒𝑠𝑝𝑒𝑐𝑡 𝑡𝑜 𝐷

𝑥𝑒 6𝑥+𝑦 𝑥𝑒 6𝑥+𝑦
= =
2𝐷 − 𝐷′ 2(6) − 1
𝑥𝑒 6𝑥+𝑦
𝑃. 𝐼2 =
11
The general solution is

𝐶𝐹 + 𝑃. 𝐼1 + 𝑃. 𝐼2

𝑥 3 𝑦 𝑥 4 𝑥𝑒 6𝑥+𝑦
𝑧 = 𝜙1 𝑦 − 5𝑥 + 𝜙2 𝑦 + 6𝑥 + + +
3! 4! 11
UNIT-IV

Applications to partial differential equations

Classification of second order partial differential equation:

𝜕2𝑢 𝜕2𝑢 𝜕2 𝑢
𝐿𝑒𝑡 𝐴 + 𝐵 + 𝐶 = 0… 1
𝜕𝑥 2 𝜕𝑥𝜕𝑦 𝜕𝑦 2

𝑏𝑒 𝑡𝑕𝑒 𝑠𝑒𝑐𝑜𝑛𝑑 𝑜𝑟𝑑𝑒𝑟 𝑝𝑎𝑟𝑡𝑖𝑎𝑙 𝑑𝑖𝑓𝑓𝑒𝑟𝑒𝑛𝑡𝑖𝑎𝑙 𝑒𝑞𝑢𝑎𝑡𝑖𝑜𝑛.

The classifications of (1) are as follows

𝑖) 𝑃𝑎𝑟𝑎𝑏𝑜𝑙𝑎 𝑖𝑓 𝐵2 − 4𝐴𝐶 = 0

𝑖𝑖) 𝐸𝑙𝑙𝑖𝑝𝑠𝑒 𝑖𝑓 𝐵2 − 4𝐴𝐶 < 0

𝑖𝑖𝑖) 𝐻𝑦𝑝𝑒𝑟𝑏𝑜𝑙𝑎 𝑖𝑓 𝐵2 − 4𝐴𝐶 > 0

1. Classify the partial differential equation 𝟑𝒖𝒙𝒙 + 𝟒𝒖𝒙𝒚 + 𝟑𝒖𝒚 − 𝟐𝒖𝒙 = 𝟎.

Solution: Given 3𝑢𝑥𝑥 + 4𝑢𝑥𝑦 + 3𝑢𝑦 − 2𝑢𝑥 = 0.

𝐴 = 3, 𝐵 = 4, 𝐶 = 0

𝐵2 − 4𝐴𝐶 = 16 > 0, Hyperbolic

2. Classify the partial differential equation 𝒖𝒙𝒙 + 𝒙𝒖𝒙𝒚 = 𝟎.

Solution:

Here 𝐴 = 1, 𝐵 = 𝑥, 𝐶 = 0

𝐵2 − 4𝐴𝐶 = 𝑥 2

𝑖 𝑃𝑎𝑟𝑎𝑏𝑜𝑙𝑖𝑐 𝑖𝑓 𝑥 = 0.

𝑖𝑖 𝐻𝑦𝑝𝑒𝑟𝑏𝑜𝑙𝑖𝑐 𝑖𝑓 𝑥 > 0 𝑎𝑛𝑑 𝑥 < 0.

Initial and boundary value problems:

The values of a required solution, on the boundary of some domain will be given. These are
called boundary conditions. In other cases, when time t is one of the variables, the values of
the solution at t=0 may be presented. These are called initial conditions.

The partial differential equation together with these conditions constitutes a boundary
value problem or an initial value problem, according to the nature of the condition.
One dimensional wave equation:

One dimensional wave equation with initial and boundary conditions in which the initial
position of the string is 𝑓(𝑥) and the initial velocity imparted at each point 𝑥 is 𝑔(𝑥) is

𝜕2𝑦 2
𝜕2 𝑦
= 𝛼
𝜕𝑡 2 𝜕𝑥 2
The boundary conditions are

𝜕𝑦 (𝑥, 0)
𝑖 𝑦 0, 𝑡 = 0 𝑖𝑖 𝑦 𝑙, 𝑡 = 0, 𝑖𝑖𝑖 𝑦 𝑥, 0 = 𝑓 𝑥 𝑖𝑣 = 𝑔(𝑥)
𝜕𝑡

Possible solutions of one dimensional wave equation:

The possible solutions of one dimensional wave equation are

𝑦 𝑥, 𝑡 = 𝑐1 𝑒 𝑝𝑥 + 𝑐2 𝑒 −𝑝𝑥 𝑐3 𝑒 𝑝𝑎𝑡 + 𝑐4 𝑒 −𝑝𝑎𝑡 … (1)

𝑦 𝑥, 𝑡 = 𝑐5 cos 𝑝𝑥 + 𝑐6 sin 𝑝𝑥 𝑐7 cos 𝑝𝑎𝑡 + 𝑐8 sin 𝑝𝑎𝑡 … (2)

𝑦 𝑥, 𝑡 = 𝑐9 𝑥 + 𝑐10 𝑐11 𝑡 + 𝑐12 … (3)

Assumptions made in the derivation of one dimensional wave equation:

(i) The mass of the string per unit length is constant

(ii) The string is perfectly elastic and does not offer any resistance to bending

(iii) The tension caused by stretching the string before fixing it at the end points is so large
that the action of the gravitational force on the string can be neglected.

(iv) The string performs a small transverse motion in a vertical plane that is every particle of
the string moves strictly vertically so that the deflection and the slope at every point of the
string remain small in absolute value.

Steady state conditions of one dimensional heat flow equation:

when steady state conditions exist the heat flow equation is independent of time t.

𝜕𝑢
=0
𝜕𝑡

The heat flow equation becomes

𝜕2𝑢
=0
𝜕𝑥 2
The solution of heat flow equation is 𝑢 𝑥 = 𝑐1 𝑥 + 𝑐2
Steady state heat flow equation in two dimensions in Cartesian form:

The two dimensional heat flow equation for the steady state is given by

𝜕2𝑢 𝜕2𝑢
+ = 0. 𝐿𝑎𝑝𝑙𝑎𝑐𝑒 𝑒𝑞𝑢𝑎𝑡𝑖𝑜𝑛
𝜕𝑥 2 𝜕𝑦 2

Possible solutions of the Laplace equation:

The possible solutions of the Laplace equation are

𝑢 𝑥, 𝑦 = 𝑐1 𝑒 𝑝𝑥 + 𝑐2 𝑒 −𝑝𝑥 𝑐3 cos 𝑝𝑦 + 𝑐4 sin 𝑝𝑦 … (1)

𝑢 𝑥, 𝑦 = 𝑐5 cos 𝑝𝑥 + 𝑐6 sin 𝑝𝑥 𝑐7 𝑒 𝑝𝑦 + 𝑐8 𝑒 −𝑝𝑦 … (2)

𝑢 𝑥, 𝑦 = 𝑐9 𝑥 + 𝑐10 𝑐11 𝑦 + 𝑐12 … (3)

The polar form of two dimensional heat flow equation in steady state:

𝜕2 𝑢
2
𝜕𝑢 𝜕 2 𝑢
𝑟 +𝑟 + = 0. 𝐿𝑎𝑝𝑙𝑎𝑐𝑒 𝑒𝑞𝑢𝑎𝑡𝑖𝑜𝑛
𝜕𝑟 2 𝜕𝑟 𝜕𝜃 2
Differential equation for two dimensional heat flow equation for the unsteady state:

The two dimensional heat flow equation for the unsteady state is given by
2
𝜕2 𝑢 𝜕2 𝑢 𝜕𝑢
𝛼 + =
𝜕𝑥 2 𝜕𝑦 2 𝜕𝑡

1. A string is stretched and fastened to two points 𝑥 = 0 and 𝑥 = 𝑙 apart. Motion is started
by displacing the string into the form 𝑦 = 𝑘(𝑙𝑥 − 𝑥 2 ) from which it is released at time

𝑡 = 0. Find the displacement of any point on the string at a distance of 𝑥 from one end at
time t.

Solution:

The displacement 𝑦 𝑥, 𝑡 from one end is governed by the partial differential equation

𝜕2 𝑦 2
𝜕2 𝑦
= 𝑎 … (1)
𝜕𝑡 2 𝜕𝑥 2
The boundary conditions are

𝑖) 𝑦 0, 𝑡 = 0, 𝑡 > 0

𝑖𝑖) 𝑦 𝑙, 𝑡 = 0, 𝑡 > 0

𝜕𝑦
𝑖𝑖𝑖) = 0, 0 < 𝑥 < 𝑙
𝜕𝑡 (𝑥,0)

𝑖𝑣) 𝑦 𝑥, 0 = 𝑘 𝑙𝑥 − 𝑥 2 , 0 < 𝑥 < 𝑙


By the method of separation of variables we get three possible solutions for (1). They are

𝐼) 𝑦 𝑥, 𝑡 = 𝐴𝑡 + 𝐵 𝐶𝑥 + 𝐷

𝐼𝐼) 𝑦 𝑥, 𝑡 = 𝐴𝑒 −𝛼𝑎𝑡 + 𝐵𝑒 𝛼𝑎𝑡 𝐶𝑒 −𝛼𝑥 + 𝐷𝑒 𝛼𝑥

𝐼𝐼𝐼) 𝑦 𝑥, 𝑡 = 𝐴 𝑐𝑜𝑠 𝛼𝑎𝑡 + 𝐵 𝑠𝑖𝑛 𝛼𝑎𝑡 𝐶 𝑐𝑜𝑠 𝛼𝑥 + 𝐷 𝑠𝑖𝑛 𝛼𝑥

The solution III is suitable for (1). Since the solution III is periodic in t and it also satisfies all
the boundary conditions.

Now applying the boundary condition (i) for solution III, we get

𝑦 0, 𝑡 = 0 𝐴 𝑐𝑜𝑠 𝛼𝑎𝑡 + 𝐵 𝑠𝑖𝑛 𝛼𝑎𝑡 𝐶 = 0

∴𝐶=0 𝑏𝑒𝑐𝑎𝑢𝑠𝑒 𝐴 cos 𝛼𝑎𝑡 + 𝐵 sin 𝛼𝑎𝑡 ≠ 0

𝐼𝑓 𝐴 𝑐𝑜𝑠 𝛼𝑎𝑡 + 𝐵 𝑠𝑖𝑛 𝛼𝑎𝑡 = 0 𝑦 𝑥, 𝑡 = 0

Now applying the boundary condition (i) and put 𝐶 = 0 for solution III, we get

𝑦 𝑙, 𝑡 = 0 𝐴 𝑐𝑜𝑠 𝛼𝑎𝑡 + 𝐵 𝑠𝑖𝑛 𝛼𝑎𝑡 𝐷 𝑠𝑖𝑛 𝛼𝑙 = 0

𝑛𝜋
𝑠𝑖𝑛 𝛼𝑙 = 0 𝛼𝑙 = 𝑛𝜋 𝛼=
𝑙
𝐴 cos 𝛼𝑎𝑡 + 𝐵 sin 𝛼𝑎𝑡 ≠ 0 𝑎𝑛𝑑 𝐷 ≠ 0
𝑆𝑖𝑛𝑐𝑒 𝐴 𝑐𝑜𝑠 𝛼𝑎𝑡 + 𝐵 𝑠𝑖𝑛 𝛼𝑎𝑡 = 0 𝑎𝑛𝑑 𝐷 = 0 𝑦 𝑥, 𝑡 = 0

𝑛𝜋
Substituting 𝛼 = 𝑎𝑛𝑑 𝐶 = 0 in solution III, we get
𝑙

𝑛𝜋𝑎𝑡 𝑛𝜋𝑎𝑡 𝑛𝜋𝑥


𝑦 𝑥, 𝑡 = 𝐴 𝑐𝑜𝑠 + 𝐵 𝑠𝑖𝑛 𝐷 𝑠𝑖𝑛 … (2)
𝑙 𝑙 𝑙

By putting 𝐴𝐷 = 𝐴𝑛 𝑎𝑛𝑑 𝐵𝐷 = 𝐵𝑛 𝑖𝑛 2 , 𝑤𝑒 𝑔𝑒𝑡

𝑛𝜋𝑎𝑡 𝑛𝜋𝑎𝑡 𝑛𝜋𝑥


𝑦 𝑥, 𝑡 = 𝐴𝑛 𝑐𝑜𝑠 + 𝐵𝑛 𝑠𝑖𝑛 𝑠𝑖𝑛 … (3)
𝑙 𝑙 𝑙

The most general form of (3) is



𝑛𝜋𝑎𝑡 𝑛𝜋𝑎𝑡 𝑛𝜋𝑥
𝑦 𝑥, 𝑡 = 𝐴𝑛 𝑐𝑜𝑠 + 𝐵𝑛 𝑠𝑖𝑛 𝑠𝑖𝑛 … (4)
𝑙 𝑙 𝑙
𝑛 =1


𝜕𝑦 𝑛𝜋𝑎 𝑛𝜋𝑎𝑡 𝑛𝜋𝑎 𝑛𝜋𝑎𝑡 𝑛𝜋𝑥
= −𝐴𝑛 𝑠𝑖𝑛 + 𝐵𝑛 𝑐𝑜𝑠 𝑠𝑖𝑛 … (5)
𝜕𝑡 𝑙 𝑙 𝑙 𝑙 𝑙
𝑛=1

Now applying the boundary condition (iii) for (5), we get



𝜕𝑦 𝑛𝜋𝑎 𝑛𝜋𝑥
=0 𝐵𝑛 𝑠𝑖𝑛 =0
𝜕𝑡 (𝑥,0) 𝑙 𝑙
𝑛=1

∴ 𝐵𝑛 = 0

Substituting 𝐵𝑛 = 0 in (4), we get



𝑛𝜋𝑎𝑡 𝑛𝜋𝑥
𝑦 𝑥, 𝑡 = 𝐴𝑛 𝑐𝑜𝑠 𝑠𝑖𝑛 … (6)
𝑙 𝑙
𝑛 =1

Now applying the boundary condition (iv) for (6), we get



𝑛𝜋𝑥
𝑦 𝑥, 0 = 𝑘(𝑙𝑥 − 𝑥 2 ) 𝐴𝑛 𝑠𝑖𝑛 = 𝑘(𝑙𝑥 − 𝑥 2 )
𝑙
𝑛 =1

The above equation is Fourier Sine series.


𝑙
2 𝑛𝜋𝑥
𝐴𝑛 = 𝑘(𝑙𝑥 − 𝑥 2 ) 𝑠𝑖𝑛 𝑑𝑥
𝑙 𝑙
0

𝑙
𝑛𝜋𝑥 𝑛𝜋𝑥 𝑛𝜋𝑥
2𝑘 −𝑐𝑜𝑠 𝑙 −𝑠𝑖𝑛 𝑐𝑜𝑠
= 𝑙𝑥 − 𝑥 2 − 𝑙 − 2𝑥 𝑙 + −2 𝑙
𝑙 𝑛𝜋 𝑛𝜋 2 𝑛𝜋 3
𝑙 𝑙 𝑙 0

𝑛𝜋𝑙 𝑛𝜋𝑙 𝑛𝜋𝑙


2𝑘 −𝑐𝑜𝑠 𝑙 −𝑠𝑖𝑛 𝑙 𝑐𝑜𝑠 𝑙
= 𝑙2 − 𝑙2 𝑛𝜋 − 𝑙 − 2𝑙 + −2
𝑙 𝑛𝜋 2 𝑛𝜋 3
𝑙 𝑙 𝑙

2𝑘 −𝑠𝑖𝑛 0 𝑐𝑜𝑠 0
− (0) − 𝑙 − 0 + −2
𝑙 𝑛𝜋 2 𝑛𝜋 3
𝑙 𝑙

3
2𝑘 𝑐𝑜𝑠 𝑛𝜋 1 4𝑘 𝑙
= −2 − −2 = 1 − (−1)𝑛
𝑙 𝑛𝜋 3 𝑛𝜋 3
𝑙 𝑛𝜋
𝑙 𝑙

4𝑘𝑙 2
𝐴𝑛 = 3 3 1 − (−1)𝑛
𝑛 𝜋

8𝑘𝑙 2
𝐴𝑛 = 𝑛3 𝜋 3 𝑓𝑜𝑟 𝑛 𝑖𝑠 𝑜𝑑𝑑
0 𝑓𝑜𝑟 𝑛 𝑖𝑠 𝑒𝑣𝑒𝑛

Substituting the value of 𝐴𝑛 in (6), we get



8𝑘𝑙 2 𝑛𝜋𝑎𝑡 𝑛𝜋𝑥
𝑦 𝑥, 𝑡 = 3 3
𝑐𝑜𝑠 𝑠𝑖𝑛
𝑛 𝜋 𝑙 𝑙
𝑛 =1,3,5,…


8𝑘𝑙 2 1 𝑛𝜋𝑎𝑡 𝑛𝜋𝑥
𝑦 𝑥, 𝑡 = 3 3
𝑐𝑜𝑠 𝑠𝑖𝑛
𝜋 𝑛 𝑙 𝑙
𝑛 =1,3,5,…

2. A tightly stretched string with fixed end points 𝑥 = 0 and 𝑥 = 𝑙 is initially at rest in its
equilibrium position. If it is set vibrating giving each point a velocity 𝜆𝑥(𝑙 − 𝑥), then show
that

8𝜆𝑙 3 1 𝑛𝜋𝑥 𝑛𝑎𝜋𝑡
𝑦 𝑥, 𝑡 = sin sin
𝑎𝜋 4 𝑛4 𝑙 𝑙
𝑛 =1,3,5

Solution:

The displacement 𝑦 𝑥, 𝑡 from one end is governed by the partial differential equation

𝜕2 𝑦 2
𝜕2 𝑦
= 𝑎 … (1)
𝜕𝑡 2 𝜕𝑥 2
The boundary conditions are

𝑖) 𝑦 0, 𝑡 = 0, 𝑡 > 0

𝑖𝑖) 𝑦 𝑙, 𝑡 = 0, 𝑡 > 0

𝑖𝑖𝑖) 𝑦 𝑥, 0 = 0 , 0 < 𝑥 < 𝑙

𝜕𝑦
𝑖𝑣) = 𝜆𝑥 𝑙 − 𝑥 , 0 < 𝑥 < 𝑙
𝜕𝑡 (𝑥,0)

By the method of separation of variables we get three possible solutions for (1). They are

𝐼) 𝑦 𝑥, 𝑡 = 𝐴𝑡 + 𝐵 𝐶𝑥 + 𝐷

𝐼𝐼) 𝑦 𝑥, 𝑡 = 𝐴𝑒 −𝛼𝑎𝑡 + 𝐵𝑒 𝛼𝑎𝑡 𝐶𝑒 −𝛼𝑥 + 𝐷𝑒 𝛼𝑥

𝐼𝐼𝐼) 𝑦 𝑥, 𝑡 = 𝐴 𝑐𝑜𝑠 𝛼𝑎𝑡 + 𝐵 𝑠𝑖𝑛 𝛼𝑎𝑡 𝐶 𝑐𝑜𝑠 𝛼𝑥 + 𝐷 𝑠𝑖𝑛 𝛼𝑥

The solution III is suitable for (1). Since the solution III is periodic in t and it also satisfies all
the boundary conditions.

Now applying the boundary condition (i) for solution III, we get

𝑦 0, 𝑡 = 0 𝐴 𝑐𝑜𝑠 𝛼𝑎𝑡 + 𝐵 𝑠𝑖𝑛 𝛼𝑎𝑡 𝐶 = 0

∴𝐶=0 𝑏𝑒𝑐𝑎𝑢𝑠𝑒 𝐴 cos 𝛼𝑎𝑡 + 𝐵 sin 𝛼𝑎𝑡 ≠ 0


𝐼𝑓 𝐴 𝑐𝑜𝑠 𝛼𝑎𝑡 + 𝐵 𝑠𝑖𝑛 𝛼𝑎𝑡 = 0 𝑦 𝑥, 𝑡 = 0

Now applying the boundary condition (i) and put 𝐶 = 0 for solution III, we get

𝑦 𝑙, 𝑡 = 0 𝐴 𝑐𝑜𝑠 𝛼𝑎𝑡 + 𝐵 𝑠𝑖𝑛 𝛼𝑎𝑡 𝐷 𝑠𝑖𝑛 𝛼𝑙 = 0

𝑛𝜋
𝑠𝑖𝑛 𝛼𝑙 = 0 𝛼𝑙 = 𝑛𝜋 𝛼=
𝑙
𝐴 cos 𝛼𝑎𝑡 + 𝐵 sin 𝛼𝑎𝑡 ≠ 0 𝑎𝑛𝑑 𝐷 ≠ 0
𝑆𝑖𝑛𝑐𝑒 𝐴 𝑐𝑜𝑠 𝛼𝑎𝑡 + 𝐵 𝑠𝑖𝑛 𝛼𝑎𝑡 = 0 𝑎𝑛𝑑 𝐷 = 0 𝑦 𝑥, 𝑡 = 0

𝑛𝜋
Substituting 𝛼 = 𝑎𝑛𝑑 𝐶 = 0 in solution III, we get
𝑙

𝑛𝜋𝑎𝑡 𝑛𝜋𝑎𝑡 𝑛𝜋𝑥


𝑦 𝑥, 𝑡 = 𝐴 𝑐𝑜𝑠 + 𝐵 𝑠𝑖𝑛 𝐷 𝑠𝑖𝑛 … (2)
𝑙 𝑙 𝑙

By putting 𝐴𝐷 = 𝐴𝑛 𝑎𝑛𝑑 𝐵𝐷 = 𝐵𝑛 𝑖𝑛 2 , 𝑤𝑒 𝑔𝑒𝑡

𝑛𝜋𝑎𝑡 𝑛𝜋𝑎𝑡 𝑛𝜋𝑥


𝑦 𝑥, 𝑡 = 𝐴𝑛 𝑐𝑜𝑠 + 𝐵𝑛 𝑠𝑖𝑛 𝑠𝑖𝑛 … (3)
𝑙 𝑙 𝑙

The most general form of (3) is



𝑛𝜋𝑎𝑡 𝑛𝜋𝑎𝑡 𝑛𝜋𝑥
𝑦 𝑥, 𝑡 = 𝐴𝑛 𝑐𝑜𝑠 + 𝐵𝑛 𝑠𝑖𝑛 𝑠𝑖𝑛 … (4)
𝑙 𝑙 𝑙
𝑛 =1

Now applying the boundary condition (iii) for (4), we get



𝑛𝜋𝑥
𝑦 𝑥, 0 = 0 𝐴𝑛 𝑠𝑖𝑛 =0
𝑙
𝑛 =1

∴ 𝐴𝑛 = 0

Substituting 𝐴𝑛 = 0 in (4), we get



𝑛𝜋𝑎𝑡 𝑛𝜋𝑥
𝑦 𝑥, 𝑡 = 𝐵𝑛 𝑠𝑖𝑛 𝑠𝑖𝑛 … (5)
𝑙 𝑙
𝑛 =1


𝜕𝑦 𝑛𝜋𝑎 𝑛𝜋𝑎𝑡 𝑛𝜋𝑥
= 𝐵𝑛 𝑐𝑜𝑠 𝑠𝑖𝑛 … (6)
𝜕𝑡 𝑙 𝑙 𝑙
𝑛 =1

Now applying the boundary condition (iv) for (6), we get



𝜕𝑦 𝑛𝜋𝑎 𝑛𝜋𝑥
= 𝜆𝑥(𝑙 − 𝑥) 𝐵𝑛 𝑠𝑖𝑛 = 𝜆(𝑙𝑥 − 𝑥 2 )
𝜕𝑡 (𝑥,0) 𝑙 𝑙
𝑛=1

The above equation is Fourier Sine series.


𝑙
𝑛𝜋𝑎 2 𝑛𝜋𝑥
𝐵𝑛 = 𝜆(𝑙𝑥 − 𝑥 2 ) 𝑠𝑖𝑛 𝑑𝑥
𝑙 𝑙 𝑙
0

𝑙
𝑛𝜋𝑥 𝑛𝜋𝑥 𝑛𝜋𝑥
2𝜆 −𝑐𝑜𝑠 𝑙 −𝑠𝑖𝑛 𝑐𝑜𝑠
= 𝑙𝑥 − 𝑥 2 − 𝑙 − 2𝑥 𝑙 + −2 𝑙
𝑙 𝑛𝜋 𝑛𝜋 2 𝑛𝜋 3
𝑙 𝑙 𝑙 0

𝑛𝜋𝑙 𝑛𝜋𝑙 𝑛𝜋𝑙


2𝜆 −𝑐𝑜𝑠 𝑙 −𝑠𝑖𝑛 𝑙 𝑐𝑜𝑠 𝑙
2 2
= 𝑙 −𝑙 𝑛𝜋 − 𝑙 − 2𝑙 + −2
𝑙 𝑛𝜋 2 𝑛𝜋 3
𝑙 𝑙 𝑙

2𝜆 −𝑠𝑖𝑛 0 𝑐𝑜𝑠 0
− (0) − 𝑙 − 0 + −2
𝑙 𝑛𝜋 2 𝑛𝜋 3
𝑙 𝑙

3
2𝜆 𝑐𝑜𝑠 𝑛𝜋 1 4𝜆 𝑙
= −2 − −2 = 1 − (−1)𝑛
𝑙 𝑛𝜋 3 𝑛𝜋 3
𝑙 𝑛𝜋
𝑙 𝑙

𝑛𝜋𝑎 8𝜆𝑙 2
𝐵𝑛 = 𝑛3 𝜋 3 𝑓𝑜𝑟 𝑛 𝑖𝑠 𝑜𝑑𝑑
𝑙
0 𝑓𝑜𝑟 𝑛 𝑖𝑠 𝑒𝑣𝑒𝑛

8𝜆𝑙 3
𝐵𝑛 = 𝑛4 𝜋 4 𝑎 𝑓𝑜𝑟 𝑛 𝑖𝑠 𝑜𝑑𝑑
0 𝑓𝑜𝑟 𝑛 𝑖𝑠 𝑒𝑣𝑒𝑛

Substituting the value of 𝐵𝑛 in (5), we get



8𝜆𝑙 3 1 𝑛𝜋𝑥 𝑛𝑎𝜋𝑡
𝑦 𝑥, 𝑡 = sin sin
𝑎𝜋 4 𝑛4 𝑙 𝑙
𝑛 =1,3,5

𝑙
3. A taut string of length 𝑙 has its ends 𝑥 = 0,𝑥 = 𝑙 fixed. The point where 𝑥 = is drawn
3
2 2
𝜕 𝑦 2 𝜕 𝑦. Determine
aside a small distance h, the displacement 𝑦(𝑥, 𝑡) satisfies 2 =𝛼 2
𝜕𝑡 𝜕𝑥
𝑦(𝑥, 𝑡) at any time t.

Solution:
The displacement 𝑦 𝑥, 𝑡 from one end is governed by the partial differential equation

𝜕2 𝑦 2
𝜕2𝑦
= 𝑎 … (1)
𝜕𝑡 2 𝜕𝑥 2
y 𝑇𝑕𝑒 𝑒𝑞𝑢𝑎𝑡𝑖𝑜𝑛 𝑜𝑓 𝑙𝑖𝑛𝑒 𝑂𝐴 𝑇𝑕𝑒 𝑒𝑞𝑢𝑎𝑡𝑖𝑜𝑛 𝑜𝑓 𝑙𝑖𝑛𝑒 𝐴𝐵

𝑙
𝑥 − 𝑥1 𝑦 − 𝑦1 𝑥−3 𝑦−𝑕
= 𝑙
=
𝑥2 − 𝑥1 𝑦2 − 𝑦1 𝑙− 0−𝑕
3
𝒍
𝑨 ,𝒉
𝟑
𝑥−0 𝑦−0 𝑕𝑥 − 𝑕 3𝑙
= 𝑦−𝑕 = −
2 3𝑙
𝑙
𝒉 3
−0 𝑕−0

x 3𝑕𝑥 3𝑕𝑥 −𝑕𝑙


𝑦= 𝑦=𝑕−
𝑶 𝟎, 𝟎 𝑩 𝒍, 𝟎 𝑙 2𝑙

2𝑕𝑙 −3𝑕𝑥 +𝑕𝑙 3𝑕 (𝑙−𝑥)


𝑦= =
2𝑙 2𝑙
The boundary conditions are

𝑖) 𝑦 0, 𝑡 = 0, 𝑡 > 0

𝑖𝑖) 𝑦 𝑙, 𝑡 = 0, 𝑡 > 0

𝜕𝑦
𝑖𝑖𝑖) = 0, 0 < 𝑥 < 𝑙
𝜕𝑡 (𝑥,0)

3𝑕𝑥 𝑙
𝑓𝑜𝑟 0 < 𝑥 <
𝑖𝑣) 𝑦 𝑥, 0 = 𝑙 3
3𝑕(𝑙 − 𝑥) 𝑙
𝑓𝑜𝑟 <𝑥< 𝑙
2𝑙 3

By the method of separation of variables we get three possible solutions for (1). They are

𝐼) 𝑦 𝑥, 𝑡 = 𝐴𝑡 + 𝐵 𝐶𝑥 + 𝐷

𝐼𝐼) 𝑦 𝑥, 𝑡 = 𝐴𝑒 −𝛼𝑎𝑡 + 𝐵𝑒 𝛼𝑎𝑡 𝐶𝑒 −𝛼𝑥 + 𝐷𝑒 𝛼𝑥

𝐼𝐼𝐼) 𝑦 𝑥, 𝑡 = 𝐴 𝑐𝑜𝑠 𝛼𝑎𝑡 + 𝐵 𝑠𝑖𝑛 𝛼𝑎𝑡 𝐶 𝑐𝑜𝑠 𝛼𝑥 + 𝐷 𝑠𝑖𝑛 𝛼𝑥

The solution III is suitable for (1). Since the solution III is periodic in t and it also satisfies all
the boundary conditions.

Now applying the boundary condition (i) for solution III, we get

𝑦 0, 𝑡 = 0 𝐴 𝑐𝑜𝑠 𝛼𝑎𝑡 + 𝐵 𝑠𝑖𝑛 𝛼𝑎𝑡 𝐶 = 0

∴𝐶=0 𝑏𝑒𝑐𝑎𝑢𝑠𝑒 𝐴 cos 𝛼𝑎𝑡 + 𝐵 sin 𝛼𝑎𝑡 ≠ 0


𝐼𝑓 𝐴 𝑐𝑜𝑠 𝛼𝑎𝑡 + 𝐵 𝑠𝑖𝑛 𝛼𝑎𝑡 = 0 𝑦 𝑥, 𝑡 = 0

Now applying the boundary condition (i) and put 𝐶 = 0 for solution III, we get

𝑦 𝑙, 𝑡 = 0 𝐴 𝑐𝑜𝑠 𝛼𝑎𝑡 + 𝐵 𝑠𝑖𝑛 𝛼𝑎𝑡 𝐷 𝑠𝑖𝑛 𝛼𝑙 = 0

𝑛𝜋
𝑠𝑖𝑛 𝛼𝑙 = 0 𝛼𝑙 = 𝑛𝜋 𝛼=
𝑙
𝐴 cos 𝛼𝑎𝑡 + 𝐵 sin 𝛼𝑎𝑡 ≠ 0 𝑎𝑛𝑑 𝐷 ≠ 0
𝑆𝑖𝑛𝑐𝑒 𝐴 𝑐𝑜𝑠 𝛼𝑎𝑡 + 𝐵 𝑠𝑖𝑛 𝛼𝑎𝑡 = 0 𝑎𝑛𝑑 𝐷 = 0 𝑦 𝑥, 𝑡 = 0

𝑛𝜋
Substituting 𝛼 = 𝑎𝑛𝑑 𝐶 = 0 in solution III, we get
𝑙

𝑛𝜋𝑎𝑡 𝑛𝜋𝑎𝑡 𝑛𝜋𝑥


𝑦 𝑥, 𝑡 = 𝐴 𝑐𝑜𝑠 + 𝐵 𝑠𝑖𝑛 𝐷 𝑠𝑖𝑛 … (2)
𝑙 𝑙 𝑙

By putting 𝐴𝐷 = 𝐴𝑛 𝑎𝑛𝑑 𝐵𝐷 = 𝐵𝑛 𝑖𝑛 2 , 𝑤𝑒 𝑔𝑒𝑡

𝑛𝜋𝑎𝑡 𝑛𝜋𝑎𝑡 𝑛𝜋𝑥


𝑦 𝑥, 𝑡 = 𝐴𝑛 𝑐𝑜𝑠 + 𝐵𝑛 𝑠𝑖𝑛 𝑠𝑖𝑛 … (3)
𝑙 𝑙 𝑙

The most general form of (3) is



𝑛𝜋𝑎𝑡 𝑛𝜋𝑎𝑡 𝑛𝜋𝑥
𝑦 𝑥, 𝑡 = 𝐴𝑛 𝑐𝑜𝑠 + 𝐵𝑛 𝑠𝑖𝑛 𝑠𝑖𝑛 … (4)
𝑙 𝑙 𝑙
𝑛 =1


𝜕𝑦 𝑛𝜋𝑎 𝑛𝜋𝑎𝑡 𝑛𝜋𝑎 𝑛𝜋𝑎𝑡 𝑛𝜋𝑥
= −𝐴𝑛 𝑠𝑖𝑛 + 𝐵𝑛 𝑐𝑜𝑠 𝑠𝑖𝑛 … (5)
𝜕𝑡 𝑙 𝑙 𝑙 𝑙 𝑙
𝑛=1

Now applying the boundary condition (iii) for (5), we get



𝜕𝑦 𝑛𝜋𝑎 𝑛𝜋𝑥
=0 𝐵𝑛 𝑠𝑖𝑛 =0
𝜕𝑡 (𝑥,0) 𝑙 𝑙
𝑛=1

∴ 𝐵𝑛 = 0

Substituting 𝐵𝑛 = 0 in (4), we get



𝑛𝜋𝑎𝑡 𝑛𝜋𝑥
𝑦 𝑥, 𝑡 = 𝐴𝑛 𝑐𝑜𝑠 𝑠𝑖𝑛 … (6)
𝑙 𝑙
𝑛 =1

Now applying the boundary condition (iv) for (6), we get


3𝑕𝑥 𝑙
𝑓𝑜𝑟 0 < 𝑥 <
𝑦 𝑥, 0 = 𝑙 3
3𝑕(𝑙 − 𝑥) 𝑙
𝑓𝑜𝑟 <𝑥< 𝑙
2𝑙 3


3𝑕𝑥 𝑙
𝑛𝜋𝑥 𝑓𝑜𝑟 0 < 𝑥 <
𝐴𝑛 𝑠𝑖𝑛 = 𝑙 3
𝑙 3𝑕(𝑙 − 𝑥) 𝑙
𝑛 =1 𝑓𝑜𝑟 <𝑥< 𝑙
2𝑙 3

The above equation is Fourier Sine series.


𝑙
3 𝑙
2 3𝑕𝑥 𝑛𝜋𝑥 3𝑕(𝑙 − 𝑥) 𝑛𝜋𝑥
𝐴𝑛 = 𝑠𝑖𝑛 𝑑𝑥 + 𝑠𝑖𝑛 𝑑𝑥
𝑙 𝑙 𝑙 2𝑙 𝑙
0 𝑙
3

𝑙
3 𝑙
6𝑕 𝑛𝜋𝑥 (𝑙 − 𝑥) 𝑛𝜋𝑥
= 2 𝑥 𝑠𝑖𝑛 𝑑𝑥 + 𝑠𝑖𝑛 𝑑𝑥
𝑙 𝑙 2 𝑙
0 𝑙
3

𝑙
𝑛𝜋𝑥 𝑛𝜋𝑥 3
6𝑕 −𝑐𝑜𝑠 𝑙 −𝑠𝑖𝑛 𝑙
= 2 𝑥 𝑛𝜋 −1
𝑙 𝑛𝜋 2
𝑙 𝑙 0
𝑙
𝑛𝜋𝑥 𝑛𝜋𝑥
(𝑙 − 𝑥) −𝑐𝑜𝑠 𝑙 (−1) −𝑠𝑖𝑛 𝑙
+ 𝑛𝜋 −
2 2 𝑛𝜋 2
𝑙 𝑙 𝑙
3

𝑛𝜋𝑙 𝑛𝜋𝑙
6𝑕 𝑙 −𝑐𝑜𝑠 3𝑙 −𝑠𝑖𝑛 3𝑙
= 2 𝑛𝜋 −1
𝑙 3 𝑛𝜋 2
𝑙 𝑙
𝑙 𝑛𝜋𝑙 𝑛𝜋𝑙
(𝑙 − 3) −𝑐𝑜𝑠 3𝑙 (−1) −𝑠𝑖𝑛 3𝑙
− 𝑛𝜋 −
2 2 𝑛𝜋 2
𝑙 𝑙

𝑛𝜋 𝑛𝜋 𝑛𝜋 𝑛𝜋
6𝑕 𝑙 −𝑐𝑜𝑠 3 −𝑠𝑖𝑛 3 𝑙 −𝑐𝑜𝑠 3 (−1) −𝑠𝑖𝑛 3
= 2 𝑛𝜋 −1 − 𝑛𝜋 +
𝑙 3 𝑛𝜋 2 3 2 𝑛𝜋 2
𝑙 𝑙 𝑙 𝑙
𝑛𝜋 𝑛𝜋
6𝑕 𝑠𝑖𝑛 3 1 𝑠𝑖𝑛 3 9𝑕𝑙 2 𝑛𝜋
= 2 2 + 2 = 𝑠𝑖𝑛
𝑙 𝑛𝜋 2 𝑛𝜋 2 2
𝑙 𝑛 𝜋 2 3
𝑙 𝑙
9𝑕 𝑛𝜋
𝐴𝑛 = 2 2
𝑠𝑖𝑛
𝑛 𝜋 3
Substituting the value of 𝐴𝑛 in (6), we get

9𝑕 𝑛𝜋 𝑛𝜋𝑎𝑡 𝑛𝜋𝑥
𝑦 𝑥, 𝑡 = 2 2
𝑠𝑖𝑛 𝑐𝑜𝑠 𝑠𝑖𝑛
𝑛 𝜋 3 𝑙 𝑙
𝑛 =1


9𝑕 1 𝑛𝜋 𝑛𝜋𝑎𝑡 𝑛𝜋𝑥
𝑦 𝑥, 𝑡 = 2 𝑠𝑖𝑛 𝑐𝑜𝑠 𝑠𝑖𝑛
𝜋 𝑛2 3 𝑙 𝑙
𝑛=1

4. A string is tightly stretched and its ends are fastened at two points 𝑥 = 0 and 𝑥 = 𝑙. The
point of the string is displaced transversely through a small distance ‘b’ and the string is
released from rest in that position. Find an expression for the transverse displacement of
the string at any time during the subsequent motion.

Solution:

The displacement 𝑦 𝑥, 𝑡 from one end is governed by the partial differential equation

𝜕2 𝑦 2
𝜕2𝑦
= 𝑎 … (1)
𝜕𝑡 2 𝜕𝑥 2
y 𝑇𝑕𝑒 𝑒𝑞𝑢𝑎𝑡𝑖𝑜𝑛 𝑜𝑓 𝑙𝑖𝑛𝑒 𝑂𝐴 𝑇𝑕𝑒 𝑒𝑞𝑢𝑎𝑡𝑖𝑜𝑛 𝑜𝑓 𝑙𝑖𝑛𝑒 𝐴𝐵

𝑙
𝑥 − 𝑥1 𝑦 − 𝑦1 𝑥−2 𝑦−𝑏
= 𝑙
=
𝒍 𝑥2 − 𝑥1 𝑦2 − 𝑦1 𝑙− 0−𝑏
𝑨 ,𝒃 2
𝟐
𝑥−0 𝑦−0 𝑏𝑥 − 𝑏 3𝑙
= 𝑦−𝑏 = −
𝑙
−0 𝑏−0 𝑙
𝑏 2 2
x 2𝑏𝑥 2𝑏𝑥 −𝑏𝑙
𝑦= 𝑦=𝑏−
𝑶 𝟎, 𝟎 𝑩 𝒍, 𝟎 𝑙 𝑙

𝑏𝑙 −2𝑏𝑥 +𝑏𝑙 2𝑏 (𝑙−𝑥)


𝑦= =
𝑙 𝑙

The boundary conditions are

𝑖) 𝑦 0, 𝑡 = 0, 𝑡 > 0

𝑖𝑖) 𝑦 𝑙, 𝑡 = 0, 𝑡 > 0

𝜕𝑦
𝑖𝑖𝑖) = 0, 0<𝑥<𝑙
𝜕𝑡 (𝑥,0)
2𝑏𝑥 𝑙
𝑓𝑜𝑟 0 < 𝑥 <
𝑖𝑣) 𝑦 𝑥, 0 = 𝑙 2
2𝑏(𝑙 − 𝑥) 𝑙
𝑓𝑜𝑟 <𝑥< 𝑙
𝑙 2

By the method of separation of variables we get three possible solutions for (1). They are

𝐼) 𝑦 𝑥, 𝑡 = 𝐴𝑡 + 𝐵 𝐶𝑥 + 𝐷

𝐼𝐼) 𝑦 𝑥, 𝑡 = 𝐴𝑒 −𝛼𝑎𝑡 + 𝐵𝑒 𝛼𝑎𝑡 𝐶𝑒 −𝛼𝑥 + 𝐷𝑒 𝛼𝑥

𝐼𝐼𝐼) 𝑦 𝑥, 𝑡 = 𝐴 𝑐𝑜𝑠 𝛼𝑎𝑡 + 𝐵 𝑠𝑖𝑛 𝛼𝑎𝑡 𝐶 𝑐𝑜𝑠 𝛼𝑥 + 𝐷 𝑠𝑖𝑛 𝛼𝑥

The solution III is suitable for (1). Since the solution III is periodic in t and it also satisfies all
the boundary conditions.

Now applying the boundary condition (i) for solution III, we get

𝑦 0, 𝑡 = 0 𝐴 𝑐𝑜𝑠 𝛼𝑎𝑡 + 𝐵 𝑠𝑖𝑛 𝛼𝑎𝑡 𝐶 = 0

∴𝐶=0 𝑏𝑒𝑐𝑎𝑢𝑠𝑒 𝐴 cos 𝛼𝑎𝑡 + 𝐵 sin 𝛼𝑎𝑡 ≠ 0

𝐼𝑓 𝐴 𝑐𝑜𝑠 𝛼𝑎𝑡 + 𝐵 𝑠𝑖𝑛 𝛼𝑎𝑡 = 0 𝑦 𝑥, 𝑡 = 0

Now applying the boundary condition (i) and put 𝐶 = 0 for solution III, we get

𝑦 𝑙, 𝑡 = 0 𝐴 𝑐𝑜𝑠 𝛼𝑎𝑡 + 𝐵 𝑠𝑖𝑛 𝛼𝑎𝑡 𝐷 𝑠𝑖𝑛 𝛼𝑙 = 0

𝑛𝜋
𝑠𝑖𝑛 𝛼𝑙 = 0 𝛼𝑙 = 𝑛𝜋 𝛼=
𝑙
𝐴 cos 𝛼𝑎𝑡 + 𝐵 sin 𝛼𝑎𝑡 ≠ 0 𝑎𝑛𝑑 𝐷 ≠ 0
𝑆𝑖𝑛𝑐𝑒 𝐴 𝑐𝑜𝑠 𝛼𝑎𝑡 + 𝐵 𝑠𝑖𝑛 𝛼𝑎𝑡 = 0 𝑎𝑛𝑑 𝐷 = 0 𝑦 𝑥, 𝑡 = 0

𝑛𝜋
Substituting 𝛼 = 𝑎𝑛𝑑 𝐶 = 0 in solution III, we get
𝑙

𝑛𝜋𝑎𝑡 𝑛𝜋𝑎𝑡 𝑛𝜋𝑥


𝑦 𝑥, 𝑡 = 𝐴 𝑐𝑜𝑠 + 𝐵 𝑠𝑖𝑛 𝐷 𝑠𝑖𝑛 … (2)
𝑙 𝑙 𝑙

By putting 𝐴𝐷 = 𝐴𝑛 𝑎𝑛𝑑 𝐵𝐷 = 𝐵𝑛 𝑖𝑛 2 , 𝑤𝑒 𝑔𝑒𝑡

𝑛𝜋𝑎𝑡 𝑛𝜋𝑎𝑡 𝑛𝜋𝑥


𝑦 𝑥, 𝑡 = 𝐴𝑛 𝑐𝑜𝑠 + 𝐵𝑛 𝑠𝑖𝑛 𝑠𝑖𝑛 … (3)
𝑙 𝑙 𝑙

The most general form of (3) is



𝑛𝜋𝑎𝑡 𝑛𝜋𝑎𝑡 𝑛𝜋𝑥
𝑦 𝑥, 𝑡 = 𝐴𝑛 𝑐𝑜𝑠 + 𝐵𝑛 𝑠𝑖𝑛 𝑠𝑖𝑛 … (4)
𝑙 𝑙 𝑙
𝑛 =1

𝜕𝑦 𝑛𝜋𝑎 𝑛𝜋𝑎𝑡 𝑛𝜋𝑎 𝑛𝜋𝑎𝑡 𝑛𝜋𝑥
= −𝐴𝑛 𝑠𝑖𝑛 + 𝐵𝑛 𝑐𝑜𝑠 𝑠𝑖𝑛 … (5)
𝜕𝑡 𝑙 𝑙 𝑙 𝑙 𝑙
𝑛=1

Now applying the boundary condition (iii) for (5), we get



𝜕𝑦 𝑛𝜋𝑎 𝑛𝜋𝑥
=0 𝐵𝑛 𝑠𝑖𝑛 =0
𝜕𝑡 (𝑥,0) 𝑙 𝑙
𝑛=1

∴ 𝐵𝑛 = 0

Substituting 𝐵𝑛 = 0 in (4), we get



𝑛𝜋𝑎𝑡 𝑛𝜋𝑥
𝑦 𝑥, 𝑡 = 𝐴𝑛 𝑐𝑜𝑠 𝑠𝑖𝑛 … (6)
𝑙 𝑙
𝑛 =1

Now applying the boundary condition (iv) for (6), we get

2𝑏𝑥 𝑙
𝑓𝑜𝑟 0 < 𝑥 <
𝑙 2
𝑦 𝑥, 0 =
2𝑏(𝑙 − 𝑥) 𝑙
𝑓𝑜𝑟 <𝑥< 𝑙
𝑙 2


2𝑏𝑥 𝑙
𝑛𝜋𝑥 𝑓𝑜𝑟 0 < 𝑥 <
𝑙 2
𝐴𝑛 𝑠𝑖𝑛 =
𝑙 2𝑏(𝑙 − 𝑥) 𝑙
𝑛 =1 𝑓𝑜𝑟 <𝑥< 𝑙
𝑙 2

The above equation is Fourier Sine series.


𝑙
2 𝑙
2 2𝑏𝑥 𝑛𝜋𝑥 2𝑏(𝑙 − 𝑥) 𝑛𝜋𝑥
𝐴𝑛 = 𝑠𝑖𝑛 𝑑𝑥 + 𝑠𝑖𝑛 𝑑𝑥
𝑙 𝑙 𝑙 𝑙 𝑙
0 𝑙
2

𝑙
3 𝑙
4𝑏 𝑛𝜋𝑥 𝑛𝜋𝑥
= 2 𝑥 𝑠𝑖𝑛 𝑑𝑥 + (𝑙 − 𝑥) 𝑠𝑖𝑛 𝑑𝑥
𝑙 𝑙 𝑙
0 𝑙
2

𝑙
𝑛𝜋𝑥 𝑛𝜋𝑥 2
4𝑏 −𝑐𝑜𝑠 −𝑠𝑖𝑛
= 2 𝑥 𝑙 −1 𝑙
𝑙 𝑛𝜋 𝑛𝜋 2
𝑙 𝑙 0
𝑙
𝑛𝜋𝑥 𝑛𝜋𝑥
(𝑙 − 𝑥) −𝑐𝑜𝑠 𝑙 −𝑠𝑖𝑛 𝑙
+ 𝑛𝜋 − (−1) 2
2 𝑛𝜋
𝑙 𝑙 𝑙
2
𝑛𝜋𝑙 𝑛𝜋𝑙
4𝑏 𝑙 −𝑐𝑜𝑠 2𝑙 −𝑠𝑖𝑛 2𝑙
= 2 𝑛𝜋 −1
𝑙 2 𝑛𝜋 2
𝑙 𝑙
𝑛𝜋𝑙 𝑛𝜋𝑙
𝑙 −𝑐𝑜𝑠 2𝑙 −𝑠𝑖𝑛 2𝑙
− (𝑙 − ) 𝑛𝜋 − (−1)
2 𝑛𝜋 2
𝑙 𝑙

𝑛𝜋 𝑛𝜋 𝑛𝜋 𝑛𝜋
4𝑏 𝑙 −𝑐𝑜𝑠 2 −𝑠𝑖𝑛 2 𝑙 −𝑐𝑜𝑠 2 −𝑠𝑖𝑛 2
= 2 𝑛𝜋 −1 − 𝑛𝜋 + (−1)
𝑙 2 𝑛𝜋 2 2 𝑛𝜋 2
𝑙 𝑙 𝑙 𝑙
𝑛𝜋 𝑛𝜋
4𝑏 𝑠𝑖𝑛 2 𝑠𝑖𝑛 2 8𝑏𝑙 2 𝑛𝜋
= 2 + = 𝑠𝑖𝑛
𝑙 𝑛𝜋 2 𝑛𝜋 2 𝑙 2 𝑛2 𝜋 2 2
𝑙 𝑙
8𝑏 𝑛𝜋
𝐴𝑛 = 𝑠𝑖𝑛
𝑛2 𝜋 2 2

Substituting the value of 𝐴𝑛 in (6), we get



8𝑏 𝑛𝜋 𝑛𝜋𝑎𝑡 𝑛𝜋𝑥
𝑦 𝑥, 𝑡 = 2 2
𝑠𝑖𝑛 𝑐𝑜𝑠 𝑠𝑖𝑛
𝑛 𝜋 2 𝑙 𝑙
𝑛 =1


8𝑏 1 𝑛𝜋 𝑛𝜋𝑎𝑡 𝑛𝜋𝑥
𝑦 𝑥, 𝑡 = 2 2
𝑠𝑖𝑛 𝑐𝑜𝑠 𝑠𝑖𝑛
𝜋 𝑛 2 𝑙 𝑙
𝑛 =1

5. A rod of length 20 cm has its ends A and B kept at temperature 30𝑜 𝐶 and 90𝑜 𝐶
respectively until steady state conditions prevail. If the temperature at each end is then
suddenly reduced to 0𝑜 𝐶 and maintained so, find the temperature distribution at a distance
from A at time t.

Solution:

The equation of heat flow is

𝜕𝑢 𝜕2𝑢
= 𝛼 2 2 … (1)
𝜕𝑡 𝜕𝑥

When the steady state conditions prevail the temperature function 𝑢(𝑥, 𝑡) is independent
of time t. Hence (1) becomes

𝜕2 𝑢 𝜕𝑢
=0 ∵ =0
𝜕𝑥 2 𝜕𝑡

The solution for this equation is


𝑢 𝑥 = 𝑎𝑥 + 𝑏 … (2)

When the steady state condition prevails the boundary conditions are

𝑎) 𝑢 0 = 30

𝑏) 𝑢 20 = 90

Applying boundary condition (𝑎) in (2), we get

𝑢 0 = 30 𝑏 = 30

Applying boundary condition (𝑏) and put b=30 in (2), we get

𝑢 20 = 90 𝑎20 + 30 = 90 𝑎=3

Substituting the value of 𝑎 and 𝑏 in (2), we get

𝑢 𝑥 = 3𝑥 + 30

In the steady state, the temperature function is

𝑢 𝑥 = 3𝑥 + 30

Here the steady state is changed to unsteady state. For this unsteady state the initial
temperature is given by

𝑢 𝑥, 0 = 3𝑥 + 30

The boundary conditions for the unsteady state are

𝑖) 𝑢 0, 𝑡 = 0, 𝑡 > 0

𝑖𝑖) 𝑢 20, 𝑡 = 0, 𝑡 > 0

𝑖𝑖𝑖) 𝑢 𝑥, 0 = 3𝑥 + 30, 0 < 𝑥 < 20

The solution for the equation (1) is


2𝛼2 𝑡
𝑢 𝑥, 𝑡 = 𝐴𝑒 −𝑎 𝐵 cos 𝑎𝑥 + 𝐶 sin 𝑎𝑥 … (3)

Applying boundary condition (i) for (3), we get


2𝛼2 𝑡
𝑢 0, 𝑡 = 0 𝐴𝑒 −𝑎 𝐵=0

𝐵=0 𝑆𝑖𝑛𝑐𝑒 𝐴 ≠ 0 𝑏𝑒𝑐𝑎𝑢𝑠𝑒 𝑖𝑓 𝐴 = 0 𝑢 𝑥, 𝑡 = 0

Applying boundary condition (ii) and the value of 𝐵 for (3), we get
2𝛼 2 𝑡
𝑢 20, 𝑡 = 0 𝐴𝑒 −𝑎 𝐶 sin 𝑎20 = 0
sin 𝑎20 = 0 𝑆𝑖𝑛𝑐𝑒 𝐴 ≠ 0, 𝐶 ≠ 0 𝑏𝑒𝑐𝑎𝑢𝑠𝑒 𝑖𝑓 𝐶 = 0 𝑢 𝑥, 𝑡 = 0

20 𝑎 = 𝑛𝜋
𝑛𝜋
𝑎=
20

Substituting the value of 𝐵 and 𝑎 in (3), we get

𝑛 2𝜋 2 𝛼 2 𝑡 𝑛𝜋𝑥
𝑢 𝑥, 𝑡 = 𝐴𝑒 − 400 𝐶 sin … (4)
20

By putting 𝐴𝐶 = 𝐴𝑛 in (4), we get

𝑛 2𝜋 2 𝛼 2 𝑡 𝑛𝜋𝑥
𝑢 𝑥, 𝑡 = 𝐴𝑛 𝑒 − 400 sin … (5)
20

The most general form of (5) is



𝑛 2𝜋 2 𝛼 2 𝑡 𝑛𝜋𝑥

𝑢 𝑥, 𝑡 = 𝐴𝑛 𝑒 400 sin … (6)
20
𝑛 =1

Applying boundary condition (iii) and the value of 𝐵 for (6), we get

𝑛𝜋𝑥
𝑢 𝑥, 0 = 3𝑥 + 30 𝐴𝑛 sin = 3𝑥 + 30 𝑓𝑜𝑟 0 < 𝑥 < 20
20
𝑛=1

The above equation is Fourier Sine series.


20
2 𝑛𝜋𝑥
𝐴𝑛 = 3𝑥 + 30 𝑠𝑖𝑛 𝑑𝑥
20 20
0

20
𝑛𝜋𝑥 𝑛𝜋𝑥
1 −𝑐𝑜𝑠 20 −𝑠𝑖𝑛 20
= 3𝑥 + 30 𝑛𝜋 −3
10 𝑛𝜋 2
20 20 0

𝑛𝜋20 𝑛𝜋20
1 −𝑐𝑜𝑠 20 −𝑠𝑖𝑛 20 −𝑐𝑜𝑠 0
= 3(20) + 30 𝑛𝜋 −3 − 30 𝑛𝜋
10 𝑛𝜋 2
20 20 20

1 −𝑐𝑜𝑠 𝑛𝜋 −1 30(20) 𝑛
= 90 𝑛𝜋 − 30 𝑛𝜋 = −3 −1 +1
10 10𝑛𝜋
20 20
60 𝑛
𝐴𝑛 = 1 − 3 −1
𝑛𝜋
Substituting the value of 𝐴𝑛 in (6), we get

60 𝑛 2𝜋 2 𝛼 2 𝑡 𝑛𝜋𝑥
𝑢 𝑥, 𝑡 = 1 − 3 −1 𝑛
𝑒− 400 sin
𝑛𝜋 20
𝑛=1


60 1 𝑛 2𝜋 2 𝛼 2 𝑡 𝑛𝜋𝑥
𝑢 𝑥, 𝑡 = 1 − 3 −1 𝑛
𝑒− 400 sin
𝜋 𝑛 20
𝑛 =1

6. A string is stretched between two fixed points at a distance 2𝑙 apart and the points of the
string are given initial velocities 𝑣 where
𝑐𝑥
𝑖𝑛 0 < 𝑥 < 𝑙
𝑣= 𝑐 𝑙
2𝑙 − 𝑥 𝑖𝑛 𝑙 < 𝑥 < 2𝑙
𝑙

𝑥 being the distance from an end point. Find the displacement of the string at any time.

Solution:

The displacement 𝑦 𝑥, 𝑡 from one end is governed by the partial differential equation

𝜕2 𝑦 2
𝜕2 𝑦
= 𝑎 … (1)
𝜕𝑡 2 𝜕𝑥 2
The boundary conditions are

𝑖) 𝑦 0, 𝑡 = 0, 𝑡 > 0

𝑖𝑖) 𝑦 2𝑙, 𝑡 = 0, 𝑡 > 0

𝑖𝑖𝑖) 𝑦 𝑥, 0 = 0, 0 < 𝑥 < 2𝑙


𝑐𝑥
𝜕𝑦 𝑖𝑛 0 < 𝑥 < 𝑙
𝑖𝑣) = 𝑐 𝑙
𝜕𝑡 (𝑥,0) 2𝑙 − 𝑥 𝑖𝑛 𝑙 < 𝑥 < 2𝑙
𝑙

By the method of separation of variables we get three possible solutions for (1). They are

𝐼) 𝑦 𝑥, 𝑡 = 𝐴𝑡 + 𝐵 𝐶𝑥 + 𝐷

𝐼𝐼) 𝑦 𝑥, 𝑡 = 𝐴𝑒 −𝛼𝑎𝑡 + 𝐵𝑒 𝛼𝑎𝑡 𝐶𝑒 −𝛼𝑥 + 𝐷𝑒 𝛼𝑥

𝐼𝐼𝐼) 𝑦 𝑥, 𝑡 = 𝐴 𝑐𝑜𝑠 𝛼𝑎𝑡 + 𝐵 𝑠𝑖𝑛 𝛼𝑎𝑡 𝐶 𝑐𝑜𝑠 𝛼𝑥 + 𝐷 𝑠𝑖𝑛 𝛼𝑥

The solution III is suitable for (1). Since the solution III is periodic in t and it also satisfies all
the boundary conditions.
Now applying the boundary condition (i) for solution III, we get

𝑦 0, 𝑡 = 0 𝐴 𝑐𝑜𝑠 𝛼𝑎𝑡 + 𝐵 𝑠𝑖𝑛 𝛼𝑎𝑡 𝐶 = 0

∴𝐶=0 𝑏𝑒𝑐𝑎𝑢𝑠𝑒 𝐴 cos 𝛼𝑎𝑡 + 𝐵 sin 𝛼𝑎𝑡 ≠ 0

𝐼𝑓 𝐴 𝑐𝑜𝑠 𝛼𝑎𝑡 + 𝐵 𝑠𝑖𝑛 𝛼𝑎𝑡 = 0 𝑦 𝑥, 𝑡 = 0

Now applying the boundary condition (i) and put 𝐶 = 0 for solution III, we get

𝑦 2𝑙, 𝑡 = 0 𝐴 𝑐𝑜𝑠 𝛼𝑎𝑡 + 𝐵 𝑠𝑖𝑛 𝛼𝑎𝑡 𝐷 𝑠𝑖𝑛 2𝛼𝑙 = 0

𝑛𝜋
𝑠𝑖𝑛 2𝛼𝑙 = 0 2𝛼𝑙 = 𝑛𝜋 𝛼=
2𝑙
𝐴 cos 𝛼𝑎𝑡 + 𝐵 sin 𝛼𝑎𝑡 ≠ 0 𝑎𝑛𝑑 𝐷 ≠ 0
𝑆𝑖𝑛𝑐𝑒 𝐴 𝑐𝑜𝑠 𝛼𝑎𝑡 + 𝐵 𝑠𝑖𝑛 𝛼𝑎𝑡 = 0 𝑎𝑛𝑑 𝐷 = 0 𝑦 𝑥, 𝑡 = 0

𝑛𝜋
Substituting 𝛼 = 𝑎𝑛𝑑 𝐶 = 0 in solution III, we get
2𝑙

𝑛𝜋𝑎𝑡 𝑛𝜋𝑎𝑡 𝑛𝜋𝑥


𝑦 𝑥, 𝑡 = 𝐴 𝑐𝑜𝑠 + 𝐵 𝑠𝑖𝑛 𝐷 𝑠𝑖𝑛 … (2)
2𝑙 2𝑙 2𝑙

By putting 𝐴𝐷 = 𝐴𝑛 𝑎𝑛𝑑 𝐵𝐷 = 𝐵𝑛 𝑖𝑛 2 , 𝑤𝑒 𝑔𝑒𝑡


𝑛𝜋𝑎𝑡 𝑛𝜋𝑎𝑡 𝑛𝜋𝑥
𝑦 𝑥, 𝑡 = 𝐴𝑛 𝑐𝑜𝑠 + 𝐵𝑛 𝑠𝑖𝑛 𝑠𝑖𝑛 … (3)
2𝑙 2𝑙 2𝑙

The most general form of (3) is



𝑛𝜋𝑎𝑡 𝑛𝜋𝑎𝑡 𝑛𝜋𝑥
𝑦 𝑥, 𝑡 = 𝐴𝑛 𝑐𝑜𝑠 + 𝐵𝑛 𝑠𝑖𝑛 𝑠𝑖𝑛 … (4)
2𝑙 2𝑙 2𝑙
𝑛 =1

Now applying the boundary condition (iii) for (4), we get



𝑛𝜋𝑥
𝑦 𝑥, 0 = 0 𝐴𝑛 𝑠𝑖𝑛 =0
2𝑙
𝑛 =1

∴ 𝐴𝑛 = 0

Substituting 𝐴𝑛 = 0 in (4), we get



𝑛𝜋𝑎𝑡 𝑛𝜋𝑥
𝑦 𝑥, 𝑡 = 𝐵𝑛 𝑠𝑖𝑛 𝑠𝑖𝑛 … (5)
2𝑙 2𝑙
𝑛 =1

𝜕𝑦 𝑛𝜋𝑎 𝑛𝜋𝑎𝑡 𝑛𝜋𝑥
= 𝐵𝑛 𝑐𝑜𝑠 𝑠𝑖𝑛 … (6)
𝜕𝑡 2𝑙 2𝑙 2𝑙
𝑛 =1

Now applying the boundary condition (iv) for (6), we get


𝑐𝑥
𝜕𝑦 𝑖𝑛 0 < 𝑥 < 𝑙
= 𝑐 𝑙
𝜕𝑡 (𝑥,0) 2𝑙 − 𝑥 𝑖𝑛 𝑙 < 𝑥 < 2𝑙
𝑙


𝑐𝑥
𝑛𝜋𝑎 𝑛𝜋𝑥 𝑖𝑛 0 < 𝑥 < 𝑙
𝐵𝑛 𝑠𝑖𝑛 = 𝑐 𝑙
2𝑙 2𝑙 2𝑙 − 𝑥 𝑖𝑛 𝑙 < 𝑥 < 2𝑙
𝑛 =1
𝑙

The above equation is Fourier Sine series.


𝑙 2𝑙
𝑛𝜋𝑎 2 𝑐𝑥 𝑛𝜋𝑥 𝑐 𝑛𝜋𝑥
𝐵𝑛 = 𝑠𝑖𝑛 𝑑𝑥 + 2𝑙 − 𝑥 𝑠𝑖𝑛 𝑑𝑥
2𝑙 2𝑙 𝑙 2𝑙 𝑙 2𝑙
0 𝑙

𝑙 2𝑙
𝑐 𝑛𝜋𝑥 𝑛𝜋𝑥
= 2 𝑥 𝑠𝑖𝑛 𝑑𝑥 + 2𝑙 − 𝑥 𝑠𝑖𝑛 𝑑𝑥
𝑙 2𝑙 2𝑙
0 𝑙

𝑛𝜋𝑥 𝑛𝜋𝑥 𝑙
𝑐 −𝑐𝑜𝑠 2𝑙 −𝑠𝑖𝑛 2𝑙
= 𝑥 𝑛𝜋 −1
𝑙2 𝑛𝜋 2
2𝑙 2𝑙 0
2𝑙
𝑛𝜋𝑥 𝑛𝜋𝑥
−𝑐𝑜𝑠 2𝑙 −𝑠𝑖𝑛 2𝑙
+ 2𝑙 − 𝑥 𝑛𝜋 − (−1)
𝑛𝜋 2
2𝑙 2𝑙 𝑙

𝑛𝜋𝑙 𝑛𝜋𝑙
𝑐 −𝑐𝑜𝑠 2𝑙 −𝑠𝑖𝑛 2𝑙
= 2 𝑙 𝑛𝜋 −1
𝑙 𝑛𝜋 2
2𝑙 2𝑙
𝑛𝜋𝑙 𝑛𝜋𝑙
−𝑐𝑜𝑠 2𝑙 −𝑠𝑖𝑛 2𝑙
− 2𝑙 − 𝑙 𝑛𝜋 − (−1)
𝑛𝜋 2
2𝑙 2𝑙

𝑛𝜋 𝑛𝜋 𝑛𝜋𝑙 𝑛𝜋
𝑐 −𝑐𝑜𝑠 2 −𝑠𝑖𝑛 2 −𝑐𝑜𝑠 2𝑙 −𝑠𝑖𝑛 2
= 2 𝑙 𝑛𝜋 −1 −𝑙 𝑛𝜋 + (−1)
𝑙 𝑛𝜋 2 𝑛𝜋 2
2𝑙 2𝑙 2𝑙 2𝑙
𝑛𝜋 𝑛𝜋
𝑐 𝑠𝑖𝑛 2 𝑠𝑖𝑛 2 8𝑐𝑙 2 𝑛𝜋
= 2 2 + 2 = 𝑠𝑖𝑛
𝑙 𝑛𝜋 𝑛𝜋 2 2
𝑙 𝑛 𝜋 2 2
2𝑙 2𝑙
𝑛𝜋𝑎 8𝑐 𝑛𝜋
𝐵𝑛 = 2 2 𝑠𝑖𝑛
2𝑙 𝑛 𝜋 2
16𝑐𝑙 𝑛𝜋
𝐵𝑛 = 3 3
𝑠𝑖𝑛
𝑎𝑛 𝜋 2

Substituting the value of 𝐵𝑛 in (5), we get



16𝑐𝑙 1 𝑛𝜋 𝑛𝜋𝑥 𝑛𝑎𝜋𝑡
𝑦 𝑥, 𝑡 = 3 3 𝑠𝑖𝑛 sin sin
𝑎𝑛 𝜋 𝑛3 2 2𝑙 2𝑙
𝑛=1

7. Solve the problem of heat conduction in a rod given that

𝜕 2 𝑢 𝜕𝑢
𝛼2 =
𝜕𝑥 2 𝜕𝑡
𝑖 𝑢 𝑖𝑠 𝑓𝑖𝑛𝑖𝑡𝑒 𝑎𝑠 𝑡 → ∞

𝜕𝑢
𝑖𝑖 = 0 𝑓𝑜𝑟 𝑥 = 0 𝑎𝑛𝑑 𝑥 = 𝑙, 𝑡 > 0
𝜕𝑥
𝑖𝑖𝑖 𝑢 = 𝑙𝑥 − 𝑥 2 , 𝑓𝑜𝑟 𝑡 = 0, 0 ≤ 𝑥 ≤ 𝑙.

Solution:

The equation of heat flow is

2
𝜕 2 𝑢 𝜕𝑢
𝛼 = … (1)
𝜕𝑥 2 𝜕𝑡
By the method of separation of variables we get three possible solutions. They are

𝐼) 𝑢 𝑥, 𝑡 = 𝐴 𝐵𝑥 + 𝐶
2𝛼 2 𝑡
𝐼𝐼) 𝑢 𝑥, 𝑡 = 𝐴𝑒 𝑎 𝐵𝑒 −𝑎𝑥 + 𝐶𝑒 𝑎𝑥
2𝛼2𝑡
𝐼𝐼𝐼) 𝑢 𝑥, 𝑡 = 𝐴𝑒 −𝑎 𝐵 𝑐𝑜𝑠 𝑎𝑥 + 𝐶 𝑠𝑖𝑛 𝑎𝑥

Applying the boundary condition (i) for all the possible solutions we get solution III is
suitable because it only satisfies the boundary condition (i)

Differentiating solution III partially with respect to 𝑥, we get


𝜕𝑢 2 2
= 𝐴𝑒 −𝑎 𝛼 𝑡 −𝑎𝐵 𝑠𝑖𝑛 𝑎𝑥 + 𝑎𝐶 𝑐𝑜𝑠 𝑎𝑥 … (2)
𝜕𝑥

Now applying boundary condition (ii) when 𝑥 = 0, we get

𝜕𝑢
= 0 𝑓𝑜𝑟 𝑥 = 0
𝜕𝑥
2𝛼2𝑡
𝐴𝑒 −𝑎 𝑎𝐶 𝑐𝑜𝑠 𝑎𝑥 = 0

𝐶=0 𝑆𝑖𝑛𝑐𝑒 𝐴 ≠ 0

Now applying boundary condition (ii) when 𝑥 = 𝑙 𝑎𝑛𝑑 𝑝𝑢𝑡 𝐶 = 0 𝑖𝑛 (2), we get
2𝛼2𝑡
𝐴𝑒 −𝑎 𝑎𝐵 𝑠𝑖𝑛 𝑎𝑙 = 0

𝑠𝑖𝑛 𝑎𝑙 = 0 𝑎𝑙 = 𝑛𝜋

𝑛𝜋
𝑎=
𝑙
Substituting the values of 𝑎 and 𝐶 in solution III, we get

𝑛 2𝜋 2 𝛼 2 𝑡 𝑛𝜋𝑥

𝑢 𝑥, 𝑡 = 𝐴𝑒 𝑙2 𝐵 𝑐𝑜𝑠 … (3)
𝑙

By putting 𝐴𝐵 = 𝐵𝑛

𝑛 2𝜋 2 𝛼 2 𝑡 𝑛𝜋𝑥

𝑢 𝑥, 𝑡 = 𝐵𝑛 𝑒 𝑙2 𝑐𝑜𝑠 … (4)
𝑙

The most general form of (4) is



𝑛 2𝜋 2 𝛼 2 𝑡 𝑛𝜋𝑥

𝑢 𝑥, 𝑡 = 𝐵𝑛 𝑒 𝑙2 𝑐𝑜𝑠 … (5)
𝑙
𝑛=0


𝑛 2𝜋 2𝛼 2 𝑡 𝑛𝜋𝑥

𝑢 𝑥, 𝑡 = 𝐵0 + 𝐵𝑛 𝑒 𝑙2 𝑐𝑜𝑠 … (6)
𝑙
𝑛 =1

Applying boundary condition (iii) in (6), we get

𝑢(𝑥, 0) = 𝑙𝑥 − 𝑥 2 , 0 ≤ 𝑥 ≤ 𝑙.

2
𝑛𝜋𝑥
𝑙𝑥 − 𝑥 = 𝐵0 + 𝐵𝑛 𝑐𝑜𝑠
𝑙
𝑛 =1

𝑎0
This is Half range Cosine series where 𝐵0 =
2
𝑙
2
𝑎0 = 𝑙𝑥 − 𝑥 2 𝑑𝑥
𝑙
0

2 𝑥2 𝑥3 2 𝑙3 𝑙3 𝑙2
= 𝑙 − = − =
𝑙 2 3 𝑙 2 3 3

𝑎0 𝑙 2
𝐵0 = =
2 6
𝑙
2 𝑛𝜋𝑥
𝐵𝑛 = 𝑙𝑥 − 𝑥 2 𝑐𝑜𝑠 𝑑𝑥
𝑙 𝑙
0

𝑙
𝑛𝜋𝑥 𝑛𝜋𝑥 𝑛𝜋𝑥
2 𝑠𝑖𝑛 𝑙 −𝑐𝑜𝑠 −𝑠𝑖𝑛 𝑙
= 𝑙𝑥 − 𝑥 2 − 𝑙 − 2𝑥 𝑙 + (−2)
𝑙 𝑛𝜋 𝑛𝜋 2 𝑛𝜋 3
𝑙 𝑙 𝑙 0

𝑛𝜋𝑙
2 −𝑐𝑜𝑠 𝑙 −1
= − 𝑙 − 2𝑙 + 𝑙
𝑙 𝑛𝜋 2 𝑛𝜋 2
𝑙 𝑙

2 −𝑐𝑜𝑠 𝑛𝜋 −1 2𝑙 2 𝑛
= 𝑙 + 𝑙 = − −1 −1
𝑙 𝑛𝜋𝑥 2 𝑛𝜋𝑥 2
𝑛2 𝜋 2
𝑙 𝑙

4𝑙 2
𝐵𝑛 = − 𝑖𝑓 𝑛 𝑖𝑠 𝑒𝑣𝑒𝑛
𝑛2 𝜋 2
0 𝑖𝑓 𝑛 𝑖𝑠 𝑜𝑑𝑑

Substituting the values of 𝐵0 and 𝐵𝑛 𝑖𝑛 (6), we get



𝑙2 4𝑙 2 − 𝑛 2 𝜋 22𝛼 2 𝑡 𝑛𝜋𝑥
𝑢 𝑥, 𝑡 = + − 2 2𝑒 𝑙 𝑐𝑜𝑠
6 𝑛 𝜋 𝑙
𝑛 =2,4,…


𝑙 2 4𝑙 2 1 − 𝑛 2𝜋 2 𝛼 2 𝑡 𝑛𝜋𝑥
𝑢 𝑥, 𝑡 = − 2 𝑒 𝑙2 𝑐𝑜𝑠
6 𝜋 𝑛2 𝑙
𝑛 =2,4,…

8. A tightly stretched flexible string has its ends fixed at 𝑥 = 0 and 𝑥 = 𝑙. At time 𝑡 = 0, the
string is given a shape defined by 𝑓 𝑥 = 𝑘𝑥 2 𝑙 − 𝑥 , where ′𝑘′ is a constant, and then
released from rest. Find the displacement of any point ‘𝑥’ of the string at any time 𝑡 > 0.

Solution:

The displacement 𝑦 𝑥, 𝑡 from one end is governed by the partial differential equation
𝜕2 𝑦 2
𝜕2 𝑦
= 𝑎 … (1)
𝜕𝑡 2 𝜕𝑥 2
The boundary conditions are

𝑖) 𝑦 0, 𝑡 = 0, 𝑡 > 0

𝑖𝑖) 𝑦 𝑙, 𝑡 = 0, 𝑡 > 0

𝜕𝑦
𝑖𝑖𝑖) = 0, 0<𝑥<𝑙
𝜕𝑡 (𝑥,0)

𝑖𝑣) 𝑦 𝑥, 0 = 𝑘𝑥 2 𝑙 − 𝑥 , 0<𝑥<𝑙

By the method of separation of variables we get three possible solutions for (1). They are

𝐼) 𝑦 𝑥, 𝑡 = 𝐴𝑡 + 𝐵 𝐶𝑥 + 𝐷

𝐼𝐼) 𝑦 𝑥, 𝑡 = 𝐴𝑒 −𝛼𝑎𝑡 + 𝐵𝑒 𝛼𝑎𝑡 𝐶𝑒 −𝛼𝑥 + 𝐷𝑒 𝛼𝑥

𝐼𝐼𝐼) 𝑦 𝑥, 𝑡 = 𝐴 𝑐𝑜𝑠 𝛼𝑎𝑡 + 𝐵 𝑠𝑖𝑛 𝛼𝑎𝑡 𝐶 𝑐𝑜𝑠 𝛼𝑥 + 𝐷 𝑠𝑖𝑛 𝛼𝑥

The solution III is suitable for (1). Since the solution III is periodic in t and it also satisfies all
the boundary conditions.

Now applying the boundary condition (i) for solution III, we get

𝑦 0, 𝑡 = 0 𝐴 𝑐𝑜𝑠 𝛼𝑎𝑡 + 𝐵 𝑠𝑖𝑛 𝛼𝑎𝑡 𝐶 = 0

∴𝐶=0 𝑏𝑒𝑐𝑎𝑢𝑠𝑒 𝐴 cos 𝛼𝑎𝑡 + 𝐵 sin 𝛼𝑎𝑡 ≠ 0

𝐼𝑓 𝐴 𝑐𝑜𝑠 𝛼𝑎𝑡 + 𝐵 𝑠𝑖𝑛 𝛼𝑎𝑡 = 0 𝑦 𝑥, 𝑡 = 0

Now applying the boundary condition (i) and put 𝐶 = 0 for solution III, we get

𝑦 𝑙, 𝑡 = 0 𝐴 𝑐𝑜𝑠 𝛼𝑎𝑡 + 𝐵 𝑠𝑖𝑛 𝛼𝑎𝑡 𝐷 𝑠𝑖𝑛 𝛼𝑙 = 0

𝑛𝜋
𝑠𝑖𝑛 𝛼𝑙 = 0 𝛼𝑙 = 𝑛𝜋 𝛼=
𝑙
𝐴 cos 𝛼𝑎𝑡 + 𝐵 sin 𝛼𝑎𝑡 ≠ 0 𝑎𝑛𝑑 𝐷 ≠ 0
𝑆𝑖𝑛𝑐𝑒 𝐴 𝑐𝑜𝑠 𝛼𝑎𝑡 + 𝐵 𝑠𝑖𝑛 𝛼𝑎𝑡 = 0 𝑎𝑛𝑑 𝐷 = 0 𝑦 𝑥, 𝑡 = 0

𝑛𝜋
Substituting 𝛼 = 𝑎𝑛𝑑 𝐶 = 0 in solution III, we get
𝑙

𝑛𝜋𝑎𝑡 𝑛𝜋𝑎𝑡 𝑛𝜋𝑥


𝑦 𝑥, 𝑡 = 𝐴 𝑐𝑜𝑠 + 𝐵 𝑠𝑖𝑛 𝐷 𝑠𝑖𝑛 … (2)
𝑙 𝑙 𝑙

By putting 𝐴𝐷 = 𝐴𝑛 𝑎𝑛𝑑 𝐵𝐷 = 𝐵𝑛 𝑖𝑛 2 , 𝑤𝑒 𝑔𝑒𝑡


𝑛𝜋𝑎𝑡 𝑛𝜋𝑎𝑡 𝑛𝜋𝑥
𝑦 𝑥, 𝑡 = 𝐴𝑛 𝑐𝑜𝑠 + 𝐵𝑛 𝑠𝑖𝑛 𝑠𝑖𝑛 … (3)
𝑙 𝑙 𝑙

The most general form of (3) is



𝑛𝜋𝑎𝑡 𝑛𝜋𝑎𝑡 𝑛𝜋𝑥
𝑦 𝑥, 𝑡 = 𝐴𝑛 𝑐𝑜𝑠 + 𝐵𝑛 𝑠𝑖𝑛 𝑠𝑖𝑛 … (4)
𝑙 𝑙 𝑙
𝑛 =1


𝜕𝑦 𝑛𝜋𝑎 𝑛𝜋𝑎𝑡 𝑛𝜋𝑎 𝑛𝜋𝑎𝑡 𝑛𝜋𝑥
= −𝐴𝑛 𝑠𝑖𝑛 + 𝐵𝑛 𝑐𝑜𝑠 𝑠𝑖𝑛 … (5)
𝜕𝑡 𝑙 𝑙 𝑙 𝑙 𝑙
𝑛=1

Now applying the boundary condition (iii) for (5), we get



𝜕𝑦 𝑛𝜋𝑎 𝑛𝜋𝑥
=0 𝐵𝑛 𝑠𝑖𝑛 =0
𝜕𝑡 (𝑥,0) 𝑙 𝑙
𝑛=1

∴ 𝐵𝑛 = 0

Substituting 𝐵𝑛 = 0 in (4), we get



𝑛𝜋𝑎𝑡 𝑛𝜋𝑥
𝑦 𝑥, 𝑡 = 𝐴𝑛 𝑐𝑜𝑠 𝑠𝑖𝑛 … (6)
𝑙 𝑙
𝑛 =1

Now applying the boundary condition (iv) for (6), we get



𝑛𝜋𝑥
𝑦 𝑥, 0 = 𝑘(𝑙𝑥 2 − 𝑥 3 ) 𝐴𝑛 𝑠𝑖𝑛 = 𝑘(𝑙𝑥 2 − 𝑥 3 )
𝑙
𝑛=1

The above equation is Fourier Sine series.


𝑙
2 𝑛𝜋𝑥
𝐴𝑛 = 𝑘(𝑙𝑥 2 − 𝑥 3 ) 𝑠𝑖𝑛 𝑑𝑥
𝑙 𝑙
0

𝑛𝜋𝑥 𝑛𝜋𝑥 𝑛𝜋𝑥


2𝑘 −𝑐𝑜𝑠 𝑙 −𝑠𝑖𝑛 𝑐𝑜𝑠
= 𝑙𝑥 2 − 𝑥 3 − 2𝑙𝑥 − 3𝑥 2 𝑙 + 2𝑙 − 6𝑥 𝑙
𝑙 𝑛𝜋 𝑛𝜋 2 𝑛𝜋 3
𝑙 𝑙 𝑙
𝑛𝜋𝑥 𝑙
𝑠𝑖𝑛 𝑙
− −6
𝑛𝜋 4
𝑙 0
𝑛𝜋𝑙 𝑛𝜋𝑙 𝑛𝜋𝑙
2𝑘 −𝑐𝑜𝑠 𝑙 −𝑠𝑖𝑛 𝑙 𝑐𝑜𝑠 𝑙
= 𝑙3 − 𝑙3 𝑛𝜋 − 2𝑙 2 − 3𝑙 2 + 2𝑙 − 6𝑙
𝑙 𝑛𝜋 2 𝑛𝜋 3
𝑙 𝑙 𝑙
𝑛𝜋𝑙
𝑠𝑖𝑛 𝑙 2𝑘 𝑐𝑜𝑠 0 𝑠𝑖𝑛 0
− −6 − 0 − 0 + 2𝑙 − 0 − −6
𝑛𝜋 4 𝑙 𝑛𝜋 3 𝑛𝜋 4
𝑙 𝑙 𝑙

2𝑘 𝑐𝑜𝑠 𝑛𝜋 1
= −4𝑙 − 2𝑙
𝑙 𝑛𝜋 3 𝑛𝜋 3

𝑙 𝑙

4𝑘𝑙 3 𝑛
𝐴𝑛 = − 2 −1 +1
𝑛3 𝜋 3
Substituting the value of 𝐴𝑛 in (6), we get

4𝑘𝑙 3 𝑛
𝑛𝜋𝑎𝑡 𝑛𝜋𝑥
𝑦 𝑥, 𝑡 = − 3 3 2 −1 + 1 𝑐𝑜𝑠 𝑠𝑖𝑛
𝑛 𝜋 𝑙 𝑙
𝑛 =1


4𝑘𝑙 3 1 𝑛
𝑛𝜋𝑎𝑡 𝑛𝜋𝑥
𝑦 𝑥, 𝑡 = − 3 2 −1 + 1 𝑐𝑜𝑠 𝑠𝑖𝑛
𝜋 𝑛3 𝑙 𝑙
𝑛=1

9. The ends A and B of a rod 10 cm long have the temperature 20𝑜 𝐶 and 40𝑜 𝐶 until steady
state prevails. The temperature at A is suddenly raised to 50𝑜 𝐶 and at the same time that
at B is lowered to10𝑜 𝐶. Find the temperature at the midpoint of the rod remains for all
time, regardless of the material of the rod.

Solution:

The equation of heat flow is

𝜕𝑢 𝜕2𝑢
= 𝛼 2 2 … (1)
𝜕𝑡 𝜕𝑥

When the steady state conditions prevail the temperature function 𝑢(𝑥, 𝑡) is independent
of time t. Hence (1) becomes

𝜕2 𝑢 𝜕𝑢
=0 ∵ =0
𝜕𝑥 2 𝜕𝑡

The solution for this equation is

𝑢 𝑥 = 𝑎𝑥 + 𝑏 … (2)

When the steady state condition prevails the boundary conditions are

𝑎) 𝑢 0 = 20
𝑏) 𝑢 10 = 40

Applying boundary condition (𝑎) in (2), we get

𝑢 0 = 20 𝑏 = 20

Applying boundary condition (𝑏) and put b=30 in (2), we get

𝑢 10 = 40 𝑎10 + 20 = 40 𝑎=2

Substituting the value of 𝑎 and 𝑏 in (2), we get

𝑢 𝑥 = 2𝑥 + 20

In the steady state, the temperature function is

𝑢 𝑥 = 2𝑥 + 20

Here the steady state is changed to unsteady state. For this unsteady state the initial
temperature is given by

𝑢 𝑥, 0 = 2𝑥 + 20

The boundary conditions for the unsteady state are

𝑖) 𝑢 0, 𝑡 = 50, 𝑡 > 0

𝑖𝑖) 𝑢 10, 𝑡 = 10, 𝑡 > 0

𝑖𝑖𝑖) 𝑢 𝑥, 0 = 2𝑥 + 20, 0 < 𝑥 < 10

The boundary condition (i) and (ii) are non zero, the solution for (i) is not applicable until the
boundary condition (i) and (ii) becomes zero.

For,

Let 𝑢 𝑥, 𝑡 = 𝑢𝑠 𝑥 + 𝑢𝑡 𝑥, 𝑡 … (3)

where 𝑢𝑠 𝑥 is the solution of (1) and is a function of 𝑥 alone and satisfying conditions
𝑢𝑠 0 = 50 𝑎𝑛𝑑 𝑢𝑠 10 = 10 𝑎𝑛𝑑 𝑢𝑡 𝑥, 𝑡 is a transient solution satisfying (3) which
decreases as 𝑡 increases.

𝑢𝑠 𝑥 = 𝑐𝑥 + 𝑑 … (4)

𝑢𝑠 0 = 𝑑 = 50

𝑢𝑠 10 = 10𝑐 + 𝑑 = 10 … 5

Substituting the value of 𝑑 = 50 in (5) we get


10𝑐 + 50 = 10 𝑐 = −4

Substituting the value of 𝑐 and 𝑑 in (4) we get

𝑢𝑠 𝑥 = −4𝑥 + 50 … (6)

𝑢𝑡 𝑥, 𝑡 = 𝑢 𝑥, 𝑡 − 𝑢𝑠 𝑥 … 7 𝑓𝑟𝑜𝑚 (3)

Now applying boundary conditions (i), (ii), (iii) and 𝑢𝑠 0 = 50, 𝑢𝑠 10 = 10 and (6) in (7),
we get the boundary conditions for 𝑢𝑡 𝑥, 𝑡 which is the transient solution of (1).

𝑖𝑣) 𝑢𝑡 0, 𝑡 = 𝑢 0, 𝑡 − 𝑢𝑠 0 = 50 − 50 = 0

𝑣) 𝑢𝑡 10, 𝑡 = 𝑢 10, 𝑡 − 𝑢𝑠 10 = 10 − 10 = 0

𝑣𝑖) 𝑢𝑡 𝑥, 0 = 𝑢 𝑥, 0 − 𝑢𝑠 𝑥 = 2𝑥 + 20 − −4𝑥 + 50 = 6𝑥 − 30, 0 < 𝑥 < 10.

The solution for the equation (1) for 𝑢𝑡 𝑥, 𝑡 is


2𝛼2𝑡
𝑢𝑡 𝑥, 𝑡 = 𝐴𝑒 −𝑎 𝐵 cos 𝑎𝑥 + 𝐶 sin 𝑎𝑥 … (8)

Applying boundary condition (iv) for (8), we get


2𝛼2𝑡
𝑢𝑡 0, 𝑡 = 0 𝐴𝑒 −𝑎 𝐵=0

𝐵=0 𝑆𝑖𝑛𝑐𝑒 𝐴 ≠ 0 𝑏𝑒𝑐𝑎𝑢𝑠𝑒 𝑖𝑓 𝐴 = 0 𝑢 𝑥, 𝑡 = 0

Applying boundary condition (v) and the value of 𝐵 for (8), we get
2𝛼2𝑡
𝑢𝑡 10, 𝑡 = 0 𝐴𝑒 −𝑎 𝐶 sin 𝑎20 = 0

sin 𝑎10 = 0 𝑆𝑖𝑛𝑐𝑒 𝐴 ≠ 0, 𝐶 ≠ 0 𝑏𝑒𝑐𝑎𝑢𝑠𝑒 𝑖𝑓 𝐶 = 0 𝑢 𝑥, 𝑡 = 0

10 𝑎 = 𝑛𝜋
𝑛𝜋
𝑎=
10
Substituting the value of 𝐵 and 𝑎 in (8), we get

𝑛 2𝜋 2 𝛼 2 𝑡 𝑛𝜋𝑥
𝑢𝑡 𝑥, 𝑡 = 𝐴𝑒 − 100 𝐶 sin … (9)
10

By putting 𝐴𝐶 = 𝐴𝑛 in (9), we get

𝑛 2𝜋 2 𝛼 2 𝑡 𝑛𝜋𝑥
𝑢𝑡 𝑥, 𝑡 = 𝐴𝑛 𝑒 − 100 sin … (10)
10

The most general form of (10) is



𝑛 2𝜋 2 𝛼 2 𝑡 𝑛𝜋𝑥
𝑢𝑡 𝑥, 𝑡 = 𝐴𝑛 𝑒 − 100 sin … (11)
10
𝑛 =1

Applying boundary condition (vi) and the value of 𝐵 for (11), we get

𝑛𝜋𝑥
𝑢𝑡 𝑥, 0 = 6𝑥 − 30 𝐴𝑛 sin = 6𝑥 − 30 𝑓𝑜𝑟 0 < 𝑥 < 10
10
𝑛 =1

The above equation is Fourier Sine series.


20
2 𝑛𝜋𝑥
𝐴𝑛 = 6𝑥 − 30 𝑠𝑖𝑛 𝑑𝑥
10 10
0

10
𝑛𝜋𝑥 𝑛𝜋𝑥
1 −𝑐𝑜𝑠 10 −𝑠𝑖𝑛 10
= 6𝑥 − 30 𝑛𝜋 −6
5 𝑛𝜋 2
10 10 0

𝑛𝜋10 𝑛𝜋10
1 −𝑐𝑜𝑠 10 −𝑠𝑖𝑛 10 −𝑐𝑜𝑠 0
= 6 10 − 30 𝑛𝜋 −6 − −30 𝑛𝜋
5 𝑛𝜋 2
10 10 10

1 −𝑐𝑜𝑠 𝑛𝜋 −1 30(10) 𝑛
= 30 𝑛𝜋 − −30 𝑛𝜋 = − −1 −1
5 5𝑛𝜋
10 10
120
60 𝑛 − 𝑖𝑓 𝑛 𝑖𝑠 𝑒𝑣𝑒𝑛
𝐴𝑛 = − 1 + −1 = 𝑛𝜋
𝑛𝜋 0 𝑖𝑓 𝑛 𝑖𝑠 𝑜𝑑𝑑

Substituting the value of 𝐴𝑛 in (11), we get



120 − 𝑛 2𝜋 2 𝛼 2 𝑡 𝑛𝜋𝑥
𝑢𝑡 𝑥, 𝑡 = − 𝑒 100 sin
𝑛𝜋 10
𝑛 =2,4,…


120 1 − 𝑛 2𝜋 2 𝛼 2 𝑡 𝑛𝜋𝑥
𝑢𝑡 𝑥, 𝑡 = − 𝑒 100 sin … (12)
𝜋 𝑛 10
𝑛 =2,4,…

Substituting the value of 𝑢𝑡 𝑥, 𝑡 𝑎𝑛𝑑 𝑢𝑠 𝑥 in (3), we get



120 1 − 𝑛 2𝜋 2 𝛼 2 𝑡 𝑛𝜋𝑥
𝑢 𝑥, 𝑡 = −4𝑥 + 50 − 𝑒 100 sin
𝜋 𝑛 10
𝑛=2,4,…
10. The boundary value problem governing the steady state temperature distribution in a
flat, thin, square plate is given by

𝜕2 𝑢 𝜕2 𝑢
+ = 0, 0 < 𝑥 < 𝑎, 0 < 𝑦 < 𝑎.
𝜕𝑥 2 𝜕𝑦 2
𝜋𝑥
𝑢 𝑥, 0 = 0, 𝑢 𝑥, 𝑎 = 4 𝑠𝑖𝑛3 , 0 < 𝑥 < 𝑎, 𝑢 0, 𝑦 = 0, 𝑢 𝑎, 𝑦 = 0, 0 < 𝑦 < 𝑎
𝑎

Find the steady state temperature distribution in the plate.

Solution:

The two dimensional heat flow is given by

𝜕2𝑢 𝜕2𝑢
+ = 0 … (1)
𝜕𝑥 2 𝜕𝑦 2

The boundary conditions are

𝑖) 𝑢 0, 𝑦 = 0, 0 < 𝑦 < 𝑎

𝑖𝑖) 𝑢 𝑎, 𝑦 = 0, 0 < 𝑦 < 𝑎

𝑖𝑖𝑖) 𝑢 𝑥, 0 = 0, 0 < 𝑥 < 𝑎


𝜋𝑥
𝑖𝑣) 𝑢 𝑥, 𝑎 = 4 𝑠𝑖𝑛3 ,0 < 𝑥 < 𝑎
𝑎

𝝅𝒙
𝒖 = 𝟒 𝒔𝒊𝒏𝟑
𝒂
𝒚=𝒂

𝒙=𝟎
𝒙=𝒂
𝒖=𝟎 𝒖=𝟎

𝒙
𝒚=𝟎
𝒖=𝟎

The possible solutions are

𝐼) 𝑢 𝑥, 𝑦 = 𝐴𝑥 + 𝐵 (𝐶𝑦 + 𝐷)

𝐼𝐼) 𝑢 𝑥, 𝑦 = 𝐴𝑐𝑜𝑠 𝜆𝑥 + 𝐵𝑠𝑖𝑛 𝜆𝑥 (𝐶𝑒 −𝜆𝑦 + 𝐷𝑒 𝜆𝑦 )

𝐼𝐼𝐼) 𝑢 𝑥, 𝑦 = (𝐴𝑒 −𝜆𝑥 + 𝐵𝑒 𝜆𝑥 ) 𝐶𝑐𝑜𝑠 𝜆𝑦 + 𝐷𝑠𝑖𝑛 𝜆𝑦


The suitable solution for (1) which satisfies all the given boundary conditions is

𝑢 𝑥, 𝑦 = 𝐴𝑐𝑜𝑠 𝜆𝑥 + 𝐵𝑠𝑖𝑛 𝜆𝑥 𝐶𝑒 −𝜆𝑦 + 𝐷𝑒 𝜆𝑦 … (2)

Applying boundary condition (i) in (2), we get

𝑢 0, 𝑦 = 𝐴𝑐𝑜𝑠 𝜆0 + 𝐵𝑠𝑖𝑛 𝜆0 𝐶𝑒−𝜆𝑦 + 𝐷𝑒𝜆𝑦 = 0

𝐴 𝐶𝑒 −𝜆𝑦 + 𝐷𝑒 𝜆𝑦 = 0 𝐴 = 0 ∵ 𝐶𝑒 −𝜆𝑦 + 𝐷𝑒 𝜆𝑦 ≠ 0

Applying boundary condition (ii) and substituting 𝐴 = 0 in (2), we get

𝑢 𝑎, 𝑦 = 𝐵𝑠𝑖𝑛 𝜆𝑎 𝐶𝑒−𝜆𝑦 + 𝐷𝑒𝜆𝑦 = 0

𝑠𝑖𝑛 𝜆𝑎 = 0 ∵ 𝐶𝑒 −𝜆𝑦 + 𝐷𝑒 𝜆𝑦 ≠ 0, 𝐵 ≠ 0
𝑛𝜋
𝜆𝑎 = 𝑛𝜋 𝜆=
𝑎
𝑛𝜋
𝑆𝑢𝑏𝑠𝑡𝑖𝑡𝑢𝑡𝑖𝑛𝑔 𝐴 = 0 𝑎𝑛𝑑 𝜆 = 𝑖𝑛 2 , 𝑤𝑒 𝑔𝑒𝑡
𝑎
𝑛𝜋𝑥 𝑛𝜋𝑦 𝑛𝜋𝑦
𝑢 𝑥, 𝑦 = 𝐵𝑠𝑖𝑛 𝐶𝑒 − 𝑎 + 𝐷𝑒 𝑎 … (3)
𝑎

Applying boundary condition (iii) in (3), we get


𝑛𝜋𝑥 𝑛𝜋 0 𝑛𝜋 0
𝑢 𝑥, 0 = 𝐵𝑠𝑖𝑛 𝐶𝑒 − 𝑎 + 𝐷𝑒 𝑎 =0
𝑎
𝑛𝜋𝑥 𝑛𝜋𝑥
𝐵𝑠𝑖𝑛 𝐶+𝐷 =0 𝐶 + 𝐷 = 0 ∵ 𝑠𝑖𝑛 ≠ 0, 𝐵 ≠ 0 𝐶 = −𝐷
𝑎 𝑎

𝑆𝑢𝑏𝑠𝑡𝑖𝑡𝑢𝑡𝑖𝑛𝑔 𝐶 = −𝐷 𝑖𝑛 3 , 𝑤𝑒 𝑔𝑒𝑡
𝑛𝜋𝑥 𝑛𝜋𝑦 𝑛𝜋𝑦 𝑛𝜋𝑥 𝑛𝜋𝑦 𝑛𝜋𝑦
𝑢 𝑥, 𝑦 = 𝐵𝑠𝑖𝑛 −𝐷𝑒 − 𝑎 + 𝐷𝑒 𝑎 = 𝐵𝐷𝑠𝑖𝑛 −𝑒 − 𝑎 +𝑒 𝑎
𝑎 𝑎
𝑛𝜋𝑥 𝑛𝜋𝑦
𝑢 𝑥, 𝑦 = 2𝐵𝐷𝑠𝑖𝑛 𝑠𝑖𝑛𝑕 … (4)
𝑎 𝑎
𝑃𝑢𝑡 𝐵𝑛 = 𝐵𝐷 𝑖𝑛 4 , 𝑤𝑒 𝑔𝑒𝑡
𝑛𝜋𝑥 𝑛𝜋𝑦
𝑢 𝑥, 𝑦 = 2𝐵𝑛 𝑠𝑖𝑛 𝑠𝑖𝑛𝑕 … (5)
𝑎 𝑎
The most general form of (5) is

𝑛𝜋𝑥 𝑛𝜋𝑦
𝑢 𝑥, 𝑦 = 𝐵𝑛 𝑠𝑖𝑛 𝑠𝑖𝑛𝑕 … (6)
𝑎 𝑎
𝑛 =1

Applying boundary condition (iv) in (6), we get



𝑛𝜋𝑥 𝑛𝜋𝑎 𝜋𝑥
𝑢 𝑥, 𝑎 = 𝐵𝑛 𝑠𝑖𝑛 𝑠𝑖𝑛𝑕 = 4 𝑠𝑖𝑛3
𝑎 𝑎 𝑎
𝑛=1


𝑛𝜋𝑥 𝜋𝑥
𝐵𝑛 𝑠𝑖𝑛𝑕 𝑛𝜋 𝑠𝑖𝑛 = 4 𝑠𝑖𝑛3 ,0 < 𝑥 < 𝑎
𝑎 𝑎
𝑛 =1

𝜋𝑥 2𝜋𝑥 3𝜋𝑥 𝜋𝑥 3𝜋𝑥


𝐵1 𝑠𝑖𝑛𝑕 𝜋𝑠𝑖𝑛 + 𝐵2 𝑠𝑖𝑛𝑕 2𝜋𝑠𝑖𝑛 + 𝐵3 𝑠𝑖𝑛𝑕 3𝜋 𝑠𝑖𝑛 + ⋯ = 3𝑠𝑖𝑛 − 𝑠𝑖𝑛
𝑎 𝑎 𝑎 𝑎 𝑎
𝑛𝜋𝑥
Equating coefficients of 𝑠𝑖𝑛 , 𝑛 = 1,2, … 𝑜𝑛 𝑏𝑜𝑡𝑕 𝑠𝑖𝑑𝑒𝑠 𝑜𝑓 𝑎𝑏𝑜𝑣𝑒 𝑒𝑞𝑢𝑎𝑡𝑖𝑜𝑛
𝑎
3
𝐵1 𝑠𝑖𝑛𝑕 𝜋 = 3 𝐵1 =
𝑠𝑖𝑛𝑕 𝜋
1
𝐵3 𝑠𝑖𝑛𝑕 3𝜋 = −1 𝐵3 = −
𝑠𝑖𝑛𝑕 3𝜋
𝐵𝑛 = 0, 𝑛 ≠ 1,3

Substituting the values of 𝐵1 , 𝐵3 , 𝐵𝑛 𝑖𝑛 6 , 𝑤𝑒 𝑔𝑒𝑡

3 𝜋𝑥 𝜋𝑦 1 3𝜋𝑥 3𝜋𝑦
𝑢 𝑥, 𝑦 = 𝑠𝑖𝑛 𝑠𝑖𝑛𝑕 − 𝑠𝑖𝑛 𝑠𝑖𝑛𝑕
𝑠𝑖𝑛𝑕 𝜋 𝑎 𝑎 𝑠𝑖𝑛𝑕 3𝜋 𝑎 𝑎
UNIT-V

Z-transform

Z-transform:
The two sided Z-transform 𝐹(𝑧) for a sequence 𝑓(𝑥) is defined as

𝐹 𝑧 = 𝑓 𝑥 𝑧 −𝑛
𝑛=−∞

The one sided Z-transform 𝐹(𝑧) for a sequence 𝑓(𝑥) is defined as


𝐹 𝑧 = 𝑓 𝑥 𝑧 −𝑛
𝑛=0

Z-transform of 𝟏 𝒐𝒓 𝒖(𝒏):

𝑍1 = 1. 𝑧 −𝑛
𝑛=0

1 1 1
=1+ + 2+ 3+⋯
𝑧 𝑧 𝑧
−1
1 1
= 1− 𝑖𝑓 <1
𝑧 𝑧
𝑧
= 𝑖𝑓 𝑧 > 1.
𝑧−1
Z-transform of 𝒂𝒏 𝒇 𝒙 :

𝑛
𝑍𝑎 𝑓 𝑥 = 𝑎𝑛 𝑓 𝑥 𝑧 −𝑛
𝑛=0

𝑧 −𝑛
= 𝑓 𝑥
𝑎
𝑛 =0

𝑧
=𝐹 𝑆𝑖𝑛𝑐𝑒 𝐹 𝑧 = 𝑓 𝑥 𝑧 −𝑛
𝑎
𝑛 =0
𝟏
Z-transform of 𝒏!:
∞ ∞
1 1 −𝑛 1 −1 𝑛
𝑍 = 𝑧 = 𝑧
𝑛! 𝑛! 𝑛!
𝑛 =0 𝑛 =0

𝑧 −1 𝑧 −1 2
𝑧 −1 3
−1
=1+ + + + ⋯ = 𝑒𝑧
1! 2! 3!
1
𝑍 = 𝑒1 𝑧
𝑛!
𝟏
Z-transform of 𝒏 , 𝒏 ≥ 𝟏:

1 1 −𝑛
𝑍 = 𝑧
𝑛 𝑛
𝑛=1

1 1 1
= + 2+ 3+⋯
𝑧 2𝑧 3𝑧
1 1
= − log 1 − 𝑖𝑓 <1
𝑧 𝑧
𝑧
= log 𝑖𝑓 𝑧 > 1
𝑧−1
First shifting theorem in Z-transform:

Statement:

If 𝒁 𝒇 𝒕 = 𝑭(𝒛) then prove that 𝒁 𝒆−𝒂𝒕 𝒇 𝒕 = 𝑭(𝒛𝒆𝒂𝑻 )


Proof:

−𝑎𝑡
𝑍𝑒 𝑓 𝑡 = 𝑒 −𝑎𝑛𝑇 𝑓 𝑛𝑇 𝑧 −𝑛
𝑛=0

= 𝑓 𝑛𝑇 𝑧𝑒 𝑎𝑇 −𝑛

𝑛 =0

𝑎𝑇
= 𝐹 𝑧𝑒 𝑆𝑖𝑛𝑐𝑒 𝐹 𝑧 = 𝑓 𝑛𝑇 𝑧 −𝑛
𝑛=0

Z-transform of 𝒏𝒇 𝒕 𝒊𝒇 𝒁 𝒇 𝒕 =𝑭 𝒛 :

𝐹 𝑧 =𝑍 𝑓 𝑡 = 𝑓 𝑛𝑇 𝑧 −𝑛
𝑛 =0

Differentiate both sides w.r.t. 𝑧 −1 .



𝑑𝐹(𝑧)
= −𝑛𝑓 𝑛𝑇 𝑧 −𝑛 −1
𝑑𝑧
𝑛 =0

𝑑𝐹(𝑧)
𝑧 =− 𝑛𝑓 𝑛𝑇 𝑧 −𝑛 = −𝑍 𝑛𝑓 𝑡
𝑑𝑧
𝑛=0

𝑑𝐹(𝑧)
𝑍 𝑛𝑓 𝑡 = −𝑧
𝑑𝑧
Second shifting theorem in Z-transform:

Statement:

𝐿𝑒𝑡 𝐹 𝑧 = 𝑍 𝑓(𝑛) then


𝑍 𝑓(𝑛 + 𝑘) = 𝑧 𝑘 𝐹 𝑧 − 𝑓 0 − 𝑧 −1 𝑓 1 − 𝑧 −2 𝑓 2 − ⋯ − 𝑓 𝑘 − 1 𝑧 − 𝑘−1

Proof:

𝑍 𝑓(𝑛 + 𝑘) = 𝑛 =0 𝑓(𝑛 + 𝑘) 𝑧 −𝑛 ...(1)

Multiplying and dividing by 𝑧 𝑘 in R.H.S of (1), we get


= 𝑧𝑘 𝑓(𝑛 + 𝑘) 𝑧 −(𝑛 +𝑘 )
𝑛=0

= 𝑧 𝑘 𝑓 𝑘 𝑧 − 𝑘 + 𝑓 𝑘 + 1 𝑧 − 𝑘+1 + 𝑓 𝑘 + 2 𝑧 − 𝑘+2 + ⋯

= 𝑧𝑘 𝑓(𝑛) 𝑧 −𝑛 − 𝑓 0 − 𝑓 1 𝑧 −1 − 𝑓 2 𝑧 −2 − ⋯ − 𝑓 𝑘 − 1 𝑧 − 𝑘−1
𝑛 =0

𝑆𝑖𝑛𝑐𝑒 𝑓(𝑛) 𝑧 −𝑛 = 𝑓 0 + 𝑓 1 𝑧 −1 … + 𝑓 𝑘 − 1 𝑧 − 𝑘−1 + 𝑓 𝑘 𝑧 − 𝑘 + ⋯


𝑛 =0

𝑍 𝑓(𝑛 + 𝑘) = 𝑧 𝑘 𝐹 𝑧 − 𝑓 0 − 𝑧 −1 𝑓 1 − 𝑧 −2 𝑓 2 − ⋯ − 𝑓 𝑘 − 1 𝑧 − 𝑘−1

Initial and final value theorem in Z transform:

Initial value theorem:

Statement:

If 𝑍 𝑓 𝑡 = 𝐹 𝑧 𝑡𝑕𝑒𝑛

lim 𝐹 𝑧 = 𝑓 0 = lim 𝑓 𝑡
𝑧→∞ 𝑡→0

Proof:

𝐹 𝑧 =𝑍 𝑓 𝑡 = 𝑓 𝑛𝑇 𝑧 − 𝑛
𝑛 =0

𝐹 𝑧 = 𝑓 0 + 𝑧 − 1 𝑓 1𝑇 + 𝑧 − 2 𝑓 2𝑇 + 𝑧 − 3 𝑓 3𝑇 + ⋯

Taking limit 𝑧 → ∞ on both sides, we get

lim 𝐹 𝑧 = lim 𝑓 0 + 𝑧 − 1 𝑓 1𝑇 + 𝑧 − 2 𝑓 2𝑇 + 𝑧 − 3 𝑓 3𝑇 + ⋯
𝑧→∞ 𝑧→∞

lim 𝐹 𝑧 = 𝑓(0)
𝑧→∞

Final value theorem:


Statement:

If 𝑍 𝑓 𝑡 = 𝐹 𝑧 𝑡𝑕𝑒𝑛

lim(𝑧 − 1)𝐹 𝑧 = 𝑓 ∞ = lim 𝑓 𝑡


𝑧→1 𝑡→∞

Proof:

𝑍 𝑓 𝑡 + 𝑇 − 𝑓(𝑡) = 𝑓 𝑛𝑇 + 𝑇 − 𝑓 𝑛𝑇 𝑧− 𝑛
𝑛 =0

𝑍 𝑓 𝑡 + 𝑇 } − 𝑍{𝑓(𝑡) = 𝑓 (𝑛 + 1)𝑇 − 𝑓 𝑛𝑇 𝑧− 𝑛
𝑛=0

𝑧𝐹 𝑧 − 𝑧𝑓 0 − 𝐹(𝑧) = 𝑓 (𝑛 + 1)𝑇 − 𝑓 𝑛𝑇 𝑧− 𝑛
𝑛=0

𝑆𝑖𝑛𝑐𝑒 𝑍 𝑓 𝑡 + 𝑇 = 𝑧𝐹 𝑧 − 𝑧𝑓 0

Taking limit 𝑧 → 1 on both sides, we get

lim 𝑧 − 1 𝐹 𝑧 − 𝑧𝑓(0) = lim 𝑓 (𝑛 + 1)𝑇 − 𝑓 𝑛𝑇 𝑧− 𝑛


𝑧→1 𝑧→1
𝑛 =0

lim 𝑧 − 1 𝐹 𝑧 − 𝑧𝑓(0) = lim 𝑓 (𝑛 + 1)𝑇 − 𝑓 𝑛𝑇 𝑧− 𝑛


𝑧→1 𝑧→1
𝑛 =0

lim 𝑧 − 1 𝐹 𝑧 − 𝑓(0) = 𝑓 (𝑛 + 1)𝑇 − 𝑓 𝑛𝑇


𝑧→1
𝑛=0

lim 𝑧 − 1 𝐹 𝑧 − 𝑓 0 = 𝑓 𝑇 − 𝑓 0 + 𝑓 2𝑇 − 𝑓 𝑇 + 𝑓 3𝑇 − 𝑓 2𝑇 + ⋯
𝑧→1

lim 𝑧 − 1 𝐹 𝑧 − 𝑓 0 = 𝑓 ∞ − 𝑓 0
𝑧→1

lim(𝑧 − 1)𝐹 𝑧 = 𝑓 ∞ = lim 𝑓 𝑡


𝑧→1 𝑡→∞

1. Prove that

1 𝑧
𝑍 = 𝑧 log
𝑛+1 𝑧−1

Solution:

1 1
𝑍 = 𝑧 −𝑛
𝑛+1 𝑛+1
𝑛 =0

𝑧 −1 𝑧 −2 𝑧 −3
= 1+ + + +⋯
2 3 4

𝑧 −2 𝑧 −3 𝑧 −4
= 𝑧 𝑧 −1 + + + +⋯
2 3 4

1
= −𝑧 𝑙𝑜𝑔 1 − 𝑧 −1 , <1
𝑧

𝑥2 𝑥3 𝑥4
𝑆𝑖𝑛𝑐𝑒 𝑥 + + + + ⋯ = − log 1 − 𝑥 , 𝑥 < 1
2 3 4

1 𝑧−1 1
= −𝑧 𝑙𝑜𝑔 1 − = −𝑧 𝑙𝑜𝑔 , <1
𝑧 𝑧 𝑧
1 𝑧
𝑍 = 𝑧 log , 𝑧 >1
𝑛+1 𝑧−1
𝑎 𝑏
𝑆𝑖𝑛𝑐𝑒 − log = log
𝑏 𝑎

2. Find the Z-transform of

2𝑛 + 3
𝑓 𝑛 =
𝑛 + 1 (𝑛 + 2)

Solution:

2𝑛 + 3 𝐴 𝐵
= + … (1)
𝑛 + 1 (𝑛 + 2) 𝑛+1 (𝑛 + 2)

2𝑛 + 3 = 𝐴 𝑛 + 2 + 𝐵 𝑛 + 1 … (2)

Put 𝑛 = −2 𝑖𝑛 2 , 𝑤𝑒 𝑔𝑒𝑡

2(−2) + 3 = 𝐴 −2 + 2 + 𝐵 −2 + 1

−1 = −𝐵 𝐵=1

Put 𝑛 = −1 𝑖𝑛 2 , 𝑤𝑒 𝑔𝑒𝑡

2(−1) + 3 = 𝐴 −1 + 2 + 𝐵 −1 + 1

𝐴=1

Substituting 𝐴 = 1 𝑎𝑛𝑑 𝐵 = 1 𝑖𝑛 1 , 𝑤𝑒 𝑔𝑒𝑡


2𝑛 + 3 1 1
= +
𝑛 + 1 (𝑛 + 2) 𝑛+1 (𝑛 + 2)

2𝑛 + 3 1 1
𝑍 =𝑍 +
𝑛 + 1 (𝑛 + 2) 𝑛+1 (𝑛 + 2)

1 1
=𝑍 +𝑍 𝐵𝑦 𝑙𝑖𝑛𝑒𝑎𝑟 𝑝𝑟𝑜𝑝𝑒𝑟𝑡𝑦
𝑛+1 (𝑛 + 2)
∞ ∞
1 1
= 𝑧 −𝑛 + 𝑧 −𝑛
𝑛+1 𝑛+2
𝑛 =0 𝑛=0

𝑧 −1 𝑧 −2 𝑧 −3 1 𝑧 −1 𝑧 −2 𝑧 −3
= 1+ + + +⋯ + + + + +⋯
2 3 4 2 3 4 5

𝑧 −2 𝑧 −3 𝑧 −4 𝑧 −2 𝑧 −3 𝑧 −4 𝑧 −5
= 𝑧 𝑧 −1 + + + + ⋯ + 𝑧2 + + + +⋯
2 3 4 2 3 4 5

1
= −𝑧 𝑙𝑜𝑔 1 − 𝑧 −1 + 𝑧 2 −𝑙𝑜𝑔 1 − 𝑧 −1 − 𝑧 −1 , <1
𝑧

𝑥2 𝑥3 𝑥4
𝑆𝑖𝑛𝑐𝑒 𝑥 + + + + ⋯ = − log 1 − 𝑥 , 𝑥 < 1
2 3 4

1 1
= −𝑧 𝑙𝑜𝑔 1 − + 𝑧 2 −𝑙𝑜𝑔 1 − − 𝑧 −1 , 𝑧 > 1
𝑧 𝑧
𝑧−1 𝑧−1
= −𝑧 𝑙𝑜𝑔 − 𝑧 2 𝑙𝑜𝑔 + 𝑧 −1 , 𝑧 > 1
𝑧 𝑧
2𝑛 + 3 𝑧−1
𝑍 = −𝑧 1 + (1 + 𝑧) 𝑙𝑜𝑔 , 𝑧 >1
𝑛 + 1 (𝑛 + 2) 𝑧

3. Find 𝑍 𝑛 (𝑛 − 1)𝑎𝑛 𝑢(𝑛)

Solution:

𝑍 𝑛 𝑛 − 1 𝑎𝑛 𝑢 𝑛 = 𝑍 𝑛2 𝑎𝑛 𝑢 𝑛 − 𝑛𝑎𝑛 𝑢 𝑛

= 𝑍 𝑛2 𝑎𝑛 𝑢 𝑛 − 𝑍 𝑛𝑎𝑛 𝑢 𝑛 𝐵𝑦 𝑙𝑖𝑛𝑒𝑎𝑟 𝑝𝑟𝑜𝑝𝑒𝑟𝑡𝑦

= 𝑍 𝑛2 𝑧→
𝑧 − 𝑍 𝑛 𝑧→
𝑧 𝑆𝑖𝑛𝑐𝑒 𝑍 𝑎𝑛 𝑓(𝑛) = 𝑍 𝑓(𝑛) 𝑧
𝑧→
𝑎 𝑎 𝑎

𝑧 𝑧+1 𝑧
= −
𝑧−1 3 𝑧→
𝑧 𝑧−1 2
𝑧→
𝑧
𝑎 𝑎

𝑑
𝑆𝑖𝑛𝑐𝑒 𝑍 𝑛2 = −𝑧 𝑍 𝑛
𝑑𝑧
𝑑 𝑧 𝑧 − 1 2 . 1 − 2𝑧 𝑧 − 1
= −𝑧 2
= −𝑧
𝑑𝑧 𝑧 − 1 𝑧−1 4

𝑧 − 1 − 2𝑧 −1 − 𝑧 𝑧 𝑧+1
= −𝑧 = −𝑧 =
𝑧−1 3 𝑧−1 3 𝑧−1 3

𝑧 𝑧 𝑧
= 𝑎 𝑎+1 − 𝑎
𝑧 3 𝑧 2
− 1
𝑎 𝑎−1

𝑎𝑧 𝑧 + 𝑎 𝑎𝑧 𝑎𝑧 2 + 𝑎2 𝑧 − 𝑎𝑧 𝑧 − 𝑎
= 3
− 2
=
𝑧−𝑎 𝑧−𝑎 𝑧−𝑎 3

𝑎𝑧 2 + 𝑎2 𝑧 − 𝑎𝑧 2 + 𝑎2 𝑧
=
𝑧−𝑎 3

2𝑎2 𝑧
=
𝑧−𝑎 3

4. Using convolution theorem find

−1
𝑧2
𝑍 2
𝑧+2

Solution:

−1
𝑧2 𝑧 𝑧
𝑍 2
= 𝑍 −1
𝑧+2 𝑧+2 𝑧+2
𝑧 𝑧
𝐿𝑒𝑡 𝐹 𝑧 = 𝑎𝑛𝑑 𝐺 𝑧 =
𝑧+2 𝑧+2
𝑧
𝑓(𝑛) = 𝑍 −1 𝐹(𝑧) = 𝑍 −1 = −2 𝑛
𝑧+2
𝑧
𝑔(𝑛) = 𝑍 −1 𝐺(𝑧) = 𝑍 −1 = −2 𝑛
𝑧+2
𝑛

𝑍 −1 𝐹 𝑧 . 𝐺 𝑧 = 𝑓 𝑛 − 𝑘 𝑔(𝑘)
𝑘 =0

𝑛 𝑛
𝑛 −𝑘 𝑘 𝑛
= −2 −2 = −2
𝑘=0 𝑘 =0

𝑛
𝑛 𝑛
= −2 1 = −2 1 + 1 + 1 + ⋯ 𝑛 + 1 𝑡𝑖𝑚𝑒𝑠
𝑘=0
−1
𝑧2
𝑍 2
= −2 𝑛 (𝑛 + 1)
𝑧+2

5. By the method of partial fraction find

𝑧2
𝑍 −1
𝑧 + 2 (𝑧 2 + 4)

Solution:

𝑧2
𝐿𝑒𝑡 𝐹 𝑧 =
𝑧 + 2 (𝑧 2 + 4)

𝐹 𝑧 𝑧 𝐴 𝐵𝑧 + 𝐶
= 2
= + 2
𝑧 𝑧 + 2 (𝑧 + 4) 𝑧 + 2 𝑧 + 4

𝑧 = 𝐴 𝑧 2 + 4 + 𝐵𝑧 + 𝐶 𝑧 + 2 … (1)

Put 𝑧 = −2 in (1),we get

1
−2 = 𝐴 4 + 4 𝐴= −
4

Equating coefficients of 𝑧 2 in (1) on both sides

1
𝐴+𝐵 = 0 𝐵 = −𝐴 𝐵=
4

Put 𝑧 = 0 in (1),we get

1
4𝐴 + 2𝐶 = 0 2𝐶 = −4𝐴 𝐶=
2

Substituting the values of A,B and C in (i), we get

1 1 1
𝐹(𝑧) 𝑧 − 𝑧+
= = 4 + 2 2
4
𝑧 2
𝑧 + 2 (𝑧 + 4) 𝑧 + 2 𝑧 +4

1 𝑧 1 𝑧2 1 𝑧
𝐹 𝑧 =− + +
4 𝑧 + 2 4 𝑧 + 4 2 𝑧2 + 4
2

𝑧2 1 𝑧 1 𝑧2 1 𝑧
𝑍 −1 = 𝑍 −1
− + +
𝑧 + 2 (𝑧 2 + 4) 4 𝑧 + 2 4 𝑧2 + 4 2 𝑧2 + 4

1 −1
𝑧 1 −1
𝑧2 1 𝑧
=− 𝑍 + 𝑍 + 𝑍 −1
4 𝑧+2 4 𝑧2 + 4 2 𝑧2 + 4

1 −1
𝑧 1 −1
𝑧2 1 2𝑧
=− 𝑍 + 𝑍 + 𝑍 −1
4 𝑧+2 4 𝑧2 + 22 4 𝑧2 + 22
1 1 𝑛𝜋 1 𝑛𝜋
= − (−2)𝑛 + (2)𝑛 cos + (2)𝑛 sin
4 4 2 4 2

𝑧2 𝑛𝜋 𝑎𝑧 𝑛𝜋
𝑆𝑖𝑛𝑐𝑒 𝑍 −1 = 𝑎𝑛 cos 𝑎𝑛𝑑 𝑍 −1 = 𝑎𝑛 sin
𝑧2 + 𝑎2 2 𝑧2 + 𝑎2 2

6. By the method of residues find

𝑧3
𝑍 −1
𝑧 − 1 2 (𝑧 − 2)

Solution:

𝑧3
𝐿𝑒𝑡 𝐹 𝑧 =
𝑧 − 1 2 (𝑧 − 2)

𝑇𝑜 𝑓𝑖𝑛𝑑 𝑝𝑜𝑙𝑒𝑠:
2
𝑧−1 𝑧−2 =0 𝑧 = 1, 1, 2

The poles are 𝑧 = 1 which is of order two and 𝑧 = 2 which is of order one

𝑧 𝑛−1 𝑧3 𝑧 𝑛 +2
𝑧 𝑛−1 𝐹 𝑧 = =
𝑧 − 1 2 (𝑧 − 2) 𝑧 − 1 2 (𝑧 − 2)

Residue at the pole 𝑧 = 1 of order two 𝑅1

𝑑
= lim 𝑧 − 1 2 𝑧 𝑛 −1 𝐹 𝑧
𝑧→1 𝑑𝑧

𝑑 2
𝑧 𝑛+2
= lim 𝑧−1
𝑧→1 𝑑𝑧 𝑧 − 1 2 (𝑧 − 2)

𝑑 𝑧 𝑛+2 𝑧 − 2 𝑛 + 2 𝑧 𝑛 +1 − 𝑧 𝑛+2
= lim = lim
𝑧→1 𝑑𝑧 (𝑧 − 2) 𝑧→1 (𝑧 − 2)2

1 − 2 𝑛 + 2 1𝑛 +1 − 1𝑛 +2
= = − 𝑛 + 2 − 1 = −𝑛 − 3
(1 − 2)2

Residue at the pole 𝑧 = 2 of order one 𝑅2

= lim 𝑧 − 2 𝑧 𝑛−1 𝐹 𝑧
𝑧→2

𝑧 𝑛+2
= lim 𝑧 − 2
𝑧→2 𝑧 − 1 2 (𝑧 − 2)

𝑧 𝑛+2
= lim = 2𝑛 +2
𝑧→2 𝑧−1 2
−1
𝑧3
𝑍 = 𝑠𝑢𝑚 𝑜𝑓 𝑟𝑒𝑠𝑖𝑑𝑢𝑒𝑠
𝑧 − 1 2 (𝑧 − 2)

𝑧3
𝑍 −1
= − 𝑛 − 3 + 2𝑛+2
𝑧 − 1 2 (𝑧 − 2)

7. Using convolution theorem find

−1
𝑧2
𝑍
𝑧 − 1 (𝑧 − 3)

Solution:

𝑧2 𝑧 𝑧
𝑍 −1 = 𝑍 −1
𝑧 − 1 (𝑧 − 3) 𝑧 − 1 (𝑧 − 3)
𝑧 𝑧
𝐿𝑒𝑡 𝐹 𝑧 = 𝑎𝑛𝑑 𝐺 𝑧 =
𝑧−1 𝑧−3
𝑧
𝑓(𝑛) = 𝑍 −1 𝐹(𝑧) = 𝑍 −1 = 1 𝑜𝑟 𝑢(𝑛)
𝑧−1
𝑧
𝑔(𝑛) = 𝑍 −1 𝐺(𝑧) = 𝑍 −1 = 3𝑛
𝑧−3
𝑛

𝑍 −1 𝐹 𝑧 . 𝐺 𝑧 = 𝑓 𝑛 − 𝑘 𝑔(𝑘)
𝑘 =0

= 1. 3𝑘 = 1 + 3 + 32 + 33 + ⋯ + 3𝑛
𝑘=0

3𝑛 +1 − 1 2
𝑥 𝑛+1 − 1
3 𝑛
1 − 𝑥 𝑛+1
= 𝑆𝑖𝑛𝑐𝑒 1 + 𝑥 + 𝑥 + 𝑥 + ⋯ + 𝑥 = 𝑜𝑟
3−1 𝑥−1 1−𝑥

−1
𝑧2 3𝑛 +1 − 1
𝑍 =
𝑧 − 1 (𝑧 − 3) 2

8. Find the inverse Z-transform of

𝑧(𝑧 + 1)
(𝑧 − 1)3

Solution:

𝑧(𝑧 + 1)
𝐿𝑒𝑡 𝐹 𝑧 =
(𝑧 − 1)3
𝑧 𝑛 (𝑧 + 1)
𝑧 𝑛−1 𝐹 𝑧 =
(𝑧 − 1)3

To find poles:

(𝑧 − 1)3 = 0 𝑧=1

The pole is 𝑧 = 1which is of order three

Residue at 𝑧 = 1is of order three 𝑅1


1 𝑑2 3
= lim 2 (𝑧 − 1) 𝑧 𝑛 −1 𝐹 𝑧
2 𝑧→ 1 𝑑𝑧

𝑑2 𝑛
1 3 𝑧 (𝑧 + 1)
= lim 2 (𝑧 − 1)
2 𝑧→ 1 𝑑𝑧 (𝑧 − 1)3

1 𝑑2 1 𝑑2 1 𝑑
= lim 2 𝑧𝑛 𝑧 + 1 = lim 2 𝑧𝑛+1 + 𝑧𝑛 = lim 𝑛 + 1 𝑧𝑛 + 𝑛𝑧𝑛−1
2 𝑧→ 1 𝑑𝑧 2 𝑧→ 1 𝑑𝑧 2 𝑧→ 1 𝑑𝑧

1 1 𝑛−1
= lim 𝑛 𝑛 + 1 𝑧𝑛−1 + 𝑛(𝑛 − 1)𝑧𝑛−2 = 𝑛 𝑛 + 1 1 + 𝑛(𝑛 − 1)1𝑛−2
2 𝑧→ 1 2

1
= 𝑛 𝑛 + 1 + 𝑛(𝑛 − 1) = 𝑛2
2
𝑧(𝑧 + 1)
𝑍 −1 = 𝑆𝑢𝑚 𝑜𝑓 𝑎𝑙𝑙 𝑟𝑒𝑠𝑖𝑑𝑢𝑒𝑠
(𝑧 − 1)3

𝑧(𝑧 + 1)
𝑍 −1 = 𝑛2
(𝑧 − 1)3

9. Find the inverse Z-transform of

𝑧(𝑧 + 2)
𝑧2+ 2𝑧 + 4

Solution:

𝑧(𝑧 + 2)
𝐿𝑒𝑡 𝐹 𝑧 =
𝑧2+ 2𝑧 + 4
𝑧 𝑛 (𝑧 + 2)
𝑧 𝑛 −1 𝐹 𝑧 =
𝑧 2 + 2𝑧 + 4

The poles are 𝑧 = −1 + 𝑖 3 , −1 − 𝑖 3 are of order one

Residue at the pole 𝑧 = −1 + 𝑖 3 𝑜𝑓 𝑜𝑟𝑑𝑒𝑟 𝑜𝑛𝑒 𝑅1


= lim (𝑧 − (−1 + 𝑖 3))𝑧𝑛−1 𝐹 𝑧
𝑧→ −1+𝑖 3

𝑧 𝑛 (𝑧 + 2)
= lim (𝑧 − (−1 + 𝑖 3))
𝑧→ −1+𝑖 3 (𝑧 − (−1 + 𝑖 3))(𝑧 − (−1 − 𝑖 3))

𝑧 𝑛 (𝑧 + 2) (−1 + 𝑖 3)𝑛 (−1 + 𝑖 3 + 2)


= lim =
𝑧→ −1+𝑖 3 (𝑧 − (−1 − 𝑖 3)) (−1 + 𝑖 3 − (−1 − 𝑖 3))

(−1 + 𝑖 3)𝑛 (𝑖 3 + 1) (−1 + 𝑖 3)𝑛 (𝑖 3 + 1)


= = −𝑖
2𝑖 3 2 3
𝑛
−1 + 𝑖 3 ( 3 − 𝑖)
=
2 3

Residue at the pole 𝑧 = −1 − 𝑖 3 𝑜𝑓 𝑜𝑟𝑑𝑒𝑟 𝑜𝑛𝑒 𝑅2

= lim (𝑧 − (−1 − 𝑖 3))𝑧𝑛−1 𝐹 𝑧


𝑧 → −1 − 𝑖 3

𝑧 𝑛 (𝑧 + 2)
= lim (𝑧 − (−1 − 𝑖 3))
𝑧→ −1 − 𝑖 3 (𝑧 − (−1 + 𝑖 3))(𝑧 − (−1 − 𝑖 3))
𝑛
𝑧 𝑛 (𝑧 + 2) −1 − 𝑖 3 (−1 − 𝑖 3 + 2)
= lim =
𝑧→ −1 − 𝑖 3 (𝑧 − (−1 + 𝑖 3)) (−1 − 𝑖 3 − (−1 + 𝑖 3))
𝑛 𝑛
−1 − 𝑖 3 (−𝑖 3 + 1) −1 − 𝑖 3 (−𝑖 3 + 1)
= =𝑖
−2𝑖 3 2 3
𝑛
−1 − 𝑖 3 ( 3 + 𝑖)
=
2 3
𝑧(𝑧 + 2)
𝑍 −1 = 𝑆𝑢𝑚 𝑜𝑓 𝑎𝑙𝑙 𝑟𝑒𝑠𝑖𝑑𝑢𝑒𝑠
𝑧2 + 2𝑧 + 4
𝑛 𝑛
−1
𝑧(𝑧 + 2) −1 + 𝑖 3 ( 3 − 𝑖) −1 − 𝑖 3 ( 3 + 𝑖)
𝑍 = +
𝑧 + 2𝑧 + 4
2
2 3 2 3

10. Find the inverse Z-transform of

𝑧 2 − 3𝑧
𝑧 + 2 (𝑧 − 5)

Solution:

𝑧 2 − 3𝑧
𝐿𝑒𝑡 𝐹 𝑧 =
𝑧 + 2 (𝑧 − 5)
𝑧2 − 3𝑧 𝑧𝑛 𝑧 − 3
𝑧 𝑛−1 𝐹 𝑧 = 𝑧 𝑛 −1 =
𝑧 + 2 (𝑧 − 5) 𝑧 + 2 (𝑧 − 5)

𝑇𝑜 𝑓𝑖𝑛𝑑 𝑝𝑜𝑙𝑒𝑠:

𝑧+2 𝑧−5 =0 𝑧 = −2, 5

The poles are 𝑧 = −2, 5

Residue at the pole 𝑧 = −2 of order one 𝑅1

= lim 𝑧 + 2 𝑧 𝑛 −1 𝐹 𝑧
𝑧→ −2

𝑧𝑛 𝑧 − 3
= lim 𝑧 + 2
𝑧→ −2 𝑧 + 2 (𝑧 − 5)

𝑧𝑛 𝑧 − 3 (−2)𝑛 −2 − 3 5
= lim = = (−2)𝑛
𝑧→ −2 (𝑧 − 5) (−2 − 5) 7

Residue at the pole 𝑧 = 5 of order one 𝑅2

= lim 𝑧 − 5 𝑧 𝑛 −1 𝐹 𝑧
𝑧→ 5

𝑧𝑛 𝑧 − 3
= lim 𝑧 − 5
𝑧→ 5 𝑧 + 2 (𝑧 − 5)

𝑧𝑛 𝑧 − 3 5𝑛 5 − 3 2
= lim = = 5𝑛
𝑧→ 5 𝑧+2 5+2 7

𝑧2 − 3𝑧
𝑍 −1 = 𝑆𝑢𝑚 𝑜𝑓 𝑎𝑙𝑙 𝑟𝑒𝑠𝑖𝑑𝑢𝑒𝑠
𝑧 + 2 (𝑧 − 5)

𝑧2 − 3𝑧 5 2
𝑍 −1 = (−2)𝑛 + 5𝑛
𝑧 + 2 (𝑧 − 5) 7 7

11. Find the inverse Z-transform of

𝑧(𝑧 2 − 1)
(𝑧 2 + 1)2

Solution:

𝑧(𝑧 2 − 1)
𝐿𝑒𝑡 𝐹 𝑧 =
(𝑧 2 + 1)2

𝑧 𝑛 (𝑧 2 − 1)
𝑧 𝑛−1 𝐹 𝑧 =
(𝑧 2 + 1)2

To find poles:
(𝑧 2 + 1)2 = 0 𝑧 = − 𝑖, 𝑖

The poles are 𝑧 = − 𝑖, 𝑖 of order two

𝑧 𝑛 (𝑧 2 − 1)
𝑧 𝑛−1 𝐹 𝑧 =
(𝑧 + 𝑖)2 (𝑧 − 𝑖)2

Residue at the pole 𝑧 = − 𝑖 of order two 𝑅1

𝑑
= lim (𝑧 + 𝑖)2 𝑧 𝑛 −1 𝐹 𝑧
𝑧→ − 𝑖 𝑑𝑧

𝑑 𝑧𝑛 (𝑧2 − 1)
= lim (𝑧 + 𝑖)2
𝑧→ − 𝑖 𝑑𝑧 (𝑧 + 𝑖)2 (𝑧 − 𝑖)2

𝑑 𝑧𝑛 (𝑧2 − 1) 𝑑 𝑧𝑛+2 − 𝑧𝑛
= lim = lim
𝑧→ − 𝑖 𝑑𝑧 (𝑧 − 𝑖)2 𝑧→ − 𝑖 𝑑𝑧 (𝑧 − 𝑖)2

(𝑧 − 𝑖)2 𝑛 + 2 𝑧𝑛+1 − 𝑛𝑧𝑛−1 − 𝑧𝑛+2 − 𝑧𝑛 2(𝑧 − 𝑖)


= lim
𝑧→ − 𝑖 (𝑧 − 𝑖)4

(−𝑖 − 𝑖)2 𝑛 + 2 (−𝑖)𝑛+1 − 𝑛(−𝑖)𝑛−1 − −𝑖 𝑛+2 − −𝑖 𝑛 2(−𝑖 − 𝑖)


=
(−𝑖 − 𝑖)4

(−2𝑖)2 𝑛 + 2 (−𝑖)𝑛+1 − 𝑛(−𝑖)𝑛−1 − −𝑖 𝑛+2 − −𝑖 𝑛 2(−2𝑖)


=
(−2𝑖)4

−4 𝑛 + 2 (−𝑖)𝑛+1 − 𝑛(−𝑖)𝑛−1 − −𝑖 𝑛+2 − −𝑖 𝑛 2(−2𝑖)


=
16

− 𝑛 + 2 (−𝑖)𝑛+1 − 𝑛(−𝑖)𝑛−1 − −𝑖 𝑛+3 − −𝑖 𝑛+1


=
4

− 𝑛 + 1 (−𝑖)𝑛+1 − 𝑛(−𝑖)𝑛−1 − −𝑖 𝑛+3


=
4

𝑛
𝑛 + 1 (−𝑖) − 𝑛(−𝑖)−1 + −𝑖 3
= − −𝑖
4
𝑛𝜋 𝑛𝜋 −𝑛𝑖 − 𝑖 − 𝑛𝑖 + 𝑖
= − cos − 𝑖 sin
2 2 4
𝑛𝜋 𝑛𝜋 −2𝑛𝑖 𝑛 𝑛𝜋 𝑛 𝑛𝜋
= − cos − 𝑖 sin = 𝑖 cos + sin
2 2 4 2 2 2 2
Residue at the pole 𝑧 = 𝑖 of order two 𝑅2
𝑑
= lim (𝑧 − 𝑖)2 𝑧 𝑛 −1 𝐹 𝑧
𝑧→ 𝑖 𝑑𝑧

𝑑 𝑧𝑛 (𝑧2 − 1)
= lim (𝑧 − 𝑖)2
𝑧→ 𝑖 𝑑𝑧 (𝑧 + 𝑖)2 (𝑧 − 𝑖)2

𝑑 𝑧𝑛 (𝑧2 − 1) 𝑑 𝑧𝑛+2 − 𝑧𝑛
= lim = lim
𝑧→ 𝑖 𝑑𝑧 (𝑧 + 𝑖)2 𝑧→ 𝑖 𝑑𝑧 (𝑧 + 𝑖)2

(𝑧 + 𝑖)2 𝑛 + 2 𝑧𝑛+1 − 𝑛𝑧𝑛−1 − 𝑧𝑛+2 − 𝑧𝑛 2(𝑧 + 𝑖)


= lim
𝑧→ 𝑖 (𝑧 + 𝑖)4

(𝑖 + 𝑖)2 𝑛 + 2 (𝑖)𝑛+1 − 𝑛(𝑖)𝑛−1 − 𝑖 𝑛+2 − 𝑖 𝑛 2(𝑖 + 𝑖)


=
(𝑖 + 𝑖)4

(2𝑖)2 𝑛 + 2 (𝑖)𝑛+1 − 𝑛(𝑖)𝑛−1 − 𝑖 𝑛+2 − 𝑖 𝑛 2(2𝑖)


=
(2𝑖)4

−4 𝑛 + 2 (𝑖)𝑛+1 − 𝑛(𝑖)𝑛−1 − 𝑖 𝑛+2 − 𝑖 𝑛 2(2𝑖)


=
16

− 𝑛 + 2 (𝑖)𝑛+1 − 𝑛(𝑖)𝑛−1 − 𝑖 𝑛+3 − 𝑖 𝑛+1

=
4

− 𝑛 + 1 (𝑖)𝑛+1 − 𝑛(𝑖)𝑛−1 − 𝑖 𝑛+3

=
4

𝑛
𝑛 + 1 (𝑖) − 𝑛(𝑖)−1 + 𝑖 3
=− 𝑖
4
𝑛𝜋 𝑛𝜋 𝑛𝑖 + 𝑖 + 𝑛𝑖 − 𝑖
= − cos + 𝑖 sin
2 2 4
𝑛𝜋 𝑛𝜋 2𝑛𝑖 𝑛 𝑛𝜋 𝑛 𝑛𝜋
= − cos + 𝑖 sin = −𝑖 cos + sin
2 2 4 2 2 2 2
𝑧(𝑧 2 − 1)
𝑍 −1 = 𝑆𝑢𝑚 𝑜𝑓 𝑡𝑕𝑒 𝑟𝑒𝑠𝑖𝑑𝑢𝑒𝑠
(𝑧 2 + 1)2
𝑛 𝑛𝜋 𝑛 𝑛𝜋 𝑛 𝑛𝜋 𝑛 𝑛𝜋
= 𝑖 cos + sin − 𝑖 cos + sin
2 2 2 2 2 2 2 2
𝑧(𝑧 2 − 1) 𝑛𝜋
𝑍 −1 2 2
= 𝑛 sin
(𝑧 + 1) 2
−1
𝑧3
12. 𝐹𝑖𝑛𝑑 𝑍 𝑏𝑦 𝑝𝑎𝑟𝑡𝑖𝑎𝑙 𝑓𝑟𝑎𝑐𝑡𝑖𝑜𝑛 𝑚𝑒𝑡𝑕𝑜𝑑
𝑧 − 1 2 (𝑧 − 2)

Solution:

𝑧3
𝐿𝑒𝑡 𝐹 𝑧 =
𝑧 − 1 2 (𝑧 − 2)

𝐹 𝑧 𝑧2 𝐴 𝐵 𝐶
= 2
= + 2
+ … (1)
𝑧 𝑧 − 1 (𝑧 − 2) 𝑧−1 𝑧−1 (𝑧 − 2)
2
𝐴 𝑧−1 𝑧−2 +𝐵 𝑧−2 +𝐶 𝑧−1 = 𝑧 2 … (2)

Put 𝑧 = 1 in (2), we get

𝐵 1−2 = 1 𝐵 = −1

Put 𝑧 = 2 in (2), we get


2
𝐶 2−1 = 22 𝐶=4

Put 𝑧 = 0 in (2), we get


2
𝐴 −1 −2 + 𝐵 −2 + 𝐶 −1 =0

2𝐴 − 2𝐵 + 𝐶 = 0 2𝐴 + 2 + 4 = 0 2𝐴 = −6 𝐴 = −3

Substituting the values of 𝐴, 𝐵 𝑎𝑛𝑑 𝐶 in (1), we get

𝐹 𝑧 −3 −1 4
= + 2
+
𝑧 𝑧−1 𝑧−1 (𝑧 − 2)

3𝑧 𝑧 4𝑧
𝐹 𝑧 =− − 2
+
𝑧−1 𝑧−1 (𝑧 − 2)

𝑧3 3𝑧 𝑧 4𝑧
𝑍 −1 2
= 𝑍 −1 − − 2
+
𝑧 − 1 (𝑧 − 2) 𝑧−1 𝑧−1 (𝑧 − 2)
𝑧 𝑧 𝑧
= −3𝑍 −1 − 𝑍 −1 2
+ 4 𝑍 −1
𝑧−1 𝑧−1 (𝑧 − 2)

= −3(1)𝑛 − 𝑛 + 4 (2)𝑛

−1
𝑧3
𝑍 = −3 − 𝑛 + 4 (2)𝑛
𝑧 − 1 2 (𝑧 − 2)

13. Solve the difference equation

𝑦 𝑛 + 3 − 3𝑦 𝑛 + 1 + 2𝑦 𝑛 = 0 𝑔𝑖𝑣𝑒𝑛 𝑡𝑕𝑎𝑡 𝑦 0 = 4, 𝑦 1 = 0 𝑎𝑛𝑑 𝑦 2 = 8.


Solution:

Let 𝑍{𝑦(𝑛)} = 𝐹(𝑧)

𝑦 𝑛 + 3 − 3𝑦 𝑛 + 1 + 2𝑦 𝑛 = 0 … (1)

Taking Z-transform on both sides in (1), we get

𝑍 𝑦 𝑛 + 3 − 3𝑦 𝑛 + 1 + 2𝑦 𝑛 =𝑍 0

𝑍 𝑦 𝑛+3 − 3𝑍 𝑦 𝑛 + 1 + 2𝑍 𝑦 𝑛 = 0 … (2) 𝐵𝑦 𝑙𝑖𝑛𝑒𝑎𝑟 𝑝𝑟𝑜𝑝𝑒𝑟𝑡𝑦

𝑍 𝑦 𝑛+3 = 𝑧 3 𝐹 𝑧 − 𝑧 3 𝑦 0 − 𝑧 2 𝑦 1 − 𝑧𝑦 2 … (3)

𝑍 𝑦 𝑛+1 = 𝑧𝐹 𝑧 − 𝑧𝑦 0 … (4)

Substituting (3) and (4) in (2), we get

𝑧 3 𝐹 𝑧 − 𝑧 3 𝑦 0 − 𝑧 2 𝑦 1 − 𝑧𝑦 2 − 3 𝑧𝐹 𝑧 − 𝑧𝑦 0 + 2𝐹 𝑧 = 0

𝑧 3 𝐹 𝑧 − 4𝑧 3 − 𝑧 2 (0) − 8𝑧 − 3 𝑧𝐹 𝑧 − 4𝑧 + 2𝐹 𝑧 = 0

𝑧 3 − 3𝑧 + 2 𝐹 𝑧 − 4𝑧 3 − 8𝑧 + 12𝑧 = 0

𝑧 3 − 3𝑧 + 2 𝐹 𝑧 = 4𝑧 3 − 4𝑧

4𝑧 3 − 4𝑧
𝐹 𝑧 = 𝑍{𝑦(𝑛)} =
𝑧 3 − 3𝑧 2 + 2

−1
4𝑧 3 − 4𝑧
𝑦(𝑛) = 𝑍
𝑧 3 − 3𝑧 + 2

4𝑧 3 − 4𝑧
𝐿𝑒𝑡 𝐹(𝑧) =
𝑧 3 − 3𝑧 + 2

4𝑧 3 − 4𝑧 4𝑧 𝑛 𝑧 2 − 1
𝑧 𝑛 −1 𝐹 𝑧 = 𝑧 𝑛−1 =
𝑧 3 − 3𝑧 + 2 𝑧 3 − 3𝑧 + 2

𝑇𝑜 𝑓𝑖𝑛𝑑 𝑝𝑜𝑙𝑒𝑠:

𝑧 3 − 3𝑧 + 2 = 0 𝑧 = −2,1,1

The poles are 𝑧 = −2,1

4𝑧 𝑛 𝑧 2 − 1 4𝑧 𝑛 𝑧 + 1 𝑧 − 1
𝑧 𝑛−1 𝐹 𝑧 = =
𝑧 − 1 2 (𝑧 + 2) 𝑧 − 1 2 (𝑧 + 2)

4𝑧 𝑛 𝑧 + 1
𝑧 𝑛 −1 𝐹 𝑧 =
𝑧 − 1 (𝑧 + 2)

Residue at the pole 𝑧 = 1 of order one 𝑅1


= lim (𝑧 − 1)𝑧 𝑛−1 𝐹 𝑧
𝑧→1

4𝑧 𝑛 𝑧 + 1
= lim(𝑧 − 1)
𝑧→1 𝑧 − 1 (𝑧 + 2)

4𝑧 𝑛 𝑧 + 1 4(1)𝑛 1 + 1 8
= lim = =
𝑧→1 (𝑧 + 2) (1 + 2) 3

Residue at the pole 𝑧 = −2 of order one 𝑅2

= lim 𝑧 + 2 𝑧 𝑛 −1 𝐹 𝑧
𝑧→ − 2

4𝑧 𝑛 𝑧 + 1 4(−2)𝑛 −2 + 1 4(−2)𝑛
= lim = =
𝑧→ − 2 𝑧−1 −2 − 1 3

4𝑧 3 − 4𝑧
𝑦 𝑛 = 𝑍 −1 = 𝑆𝑢𝑚 𝑜𝑓 𝑎𝑙𝑙 𝑟𝑒𝑠𝑖𝑑𝑢𝑒𝑠
𝑧 3 − 3𝑧 + 2

8 4(−2)𝑛
𝑦 𝑛 = +
3 3

14. Solve 𝑦𝑛 +2 + 𝑦𝑛 = 2 , 𝑦0 = 0 , 𝑦1 = 0

Solution:

Let 𝑍{𝑦(𝑛)} = 𝐹(𝑧)

𝑦 𝑛 + 2 + 𝑦 𝑛 = 2 … (1)

Taking Z-transform on both sides in (1), we get

𝑍 𝑦 𝑛+2 +𝑦 𝑛 =𝑍 2

2𝑧
𝑍 𝑦 𝑛+2 − 4𝑍 𝑦 𝑛 + 1 + 4𝑍 𝑦 𝑛 = … (2) 𝐵𝑦 𝑙𝑖𝑛𝑒𝑎𝑟 𝑝𝑟𝑜𝑝𝑒𝑟𝑡𝑦
𝑧−1

𝑍 𝑦 𝑛+2 = 𝑧 2 𝐹 𝑧 − 𝑧 2 𝑦 0 − 𝑧𝑦 1 … (3)

Substituting (3) in (2), we get

2𝑧
𝑧 2 𝐹 𝑧 − 𝑧 2 𝑦 0 − 𝑧𝑦 1 + 𝐹 𝑧 =
𝑧−1
2𝑧
𝑧2 𝐹 𝑧 − 𝑧2 0 − 𝑧 0 + 𝐹 𝑧 =
𝑧−1
2𝑧
𝑧2 + 1 𝐹 𝑧 =
𝑧−1
2𝑧
𝐹 𝑧 = 𝑍{𝑦(𝑛)} =
𝑧 − 1 𝑧2 + 1
2𝑧
𝑦(𝑛) = 𝑍 −1
𝑧 − 1 𝑧2 + 1

2𝑧
𝐿𝑒𝑡 𝐹(𝑧) =
𝑧 − 1 𝑧2 + 1

By Residue method:

2𝑧 2𝑧 𝑛
𝑧 𝑛−1 𝐹 𝑧 = 𝑧 𝑛 −1 =
𝑧 − 1 𝑧2 + 1 𝑧 − 1 𝑧2 + 1

𝑇𝑜 𝑓𝑖𝑛𝑑 𝑝𝑜𝑙𝑒𝑠:

𝑧 − 1 𝑧2 + 1 = 0 𝑧 = 1, 𝑖, −𝑖

The poles are 𝑧 = 1, 𝑖, −𝑖

2𝑧 𝑛 2𝑧 𝑛
𝑧 𝑛−1 𝐹 𝑧 = =
𝑧 − 1 𝑧2 + 1 𝑧−1 𝑧+𝑖 𝑧−𝑖

Residue at the pole 𝑧 = 1 of order one 𝑅1

= lim 𝑧 − 1 𝑧 𝑛−1 𝐹 𝑧
𝑧→1

2𝑧 𝑛
= lim 𝑧 − 1
𝑧→1 𝑧−1 𝑧+𝑖 𝑧−𝑖

2𝑧 𝑛 2(1)𝑛 2(1)𝑛 2(1)𝑛


= lim = = = =1
𝑧→1 𝑧+𝑖 𝑧−𝑖 1+𝑖 1−𝑖 1 − 𝑖2 1+1

Residue at the pole 𝑧 = 𝑖 of order one 𝑅2

= lim 𝑧 − 𝑖 𝑧 𝑛−1 𝐹 𝑧
𝑧→𝑖

2𝑧 𝑛
= lim 𝑧 − 𝑖
𝑧→𝑖 𝑧−1 𝑧+𝑖 𝑧−𝑖

2𝑧 𝑛 2𝑖 𝑛 2𝑖 𝑛
= lim = =
𝑧→𝑖 𝑧−1 𝑧+𝑖 𝑖−1 𝑖+𝑖 𝑖 − 1 2𝑖

𝑖 𝑛−1 𝑖 𝑛−1 −𝑖 − 1 −𝑖 𝑛 − 𝑖 𝑛 −1 1 + 𝑖 −1 1−𝑖


= = = = −𝑖 𝑛 = −𝑖 𝑛
𝑖−1 𝑖 − 1 −𝑖 − 1 2 2 2
𝑛𝜋 𝑛𝜋
𝑐𝑜𝑠 2 + 𝑖 𝑠𝑖𝑛 2 1−𝑖 1 𝑛𝜋 𝑛𝜋 𝑖 𝑛𝜋 𝑛𝜋
= − =− 𝑐𝑜𝑠 + 𝑠𝑖𝑛 + 𝑐𝑜𝑠 − 𝑠𝑖𝑛
2 2 2 2 2 2 2

Residue at the pole 𝑧 = − 𝑖 of order one 𝑅3

= lim 𝑧 + 𝑖 𝑧 𝑛 −1 𝐹 𝑧
𝑧→ − 𝑖
2𝑧 𝑛
= lim 𝑧 + 𝑖
𝑧→ − 𝑖 𝑧−1 𝑧+𝑖 𝑧−𝑖

2𝑧 𝑛 2(−𝑖)𝑛 2(−𝑖)𝑛
= lim = =
𝑧→ − 𝑖 𝑧−1 𝑧−𝑖 −𝑖−1 −𝑖−𝑖 − 𝑖 − 1 (−2𝑖)

(−𝑖)𝑛−1 (−𝑖)𝑛−1 𝑖 − 1 −(−𝑖)𝑛 − (−𝑖)𝑛−1


= = =
−𝑖−1 𝑖 − 1 −𝑖 − 1 2

1 + (−𝑖)−1 1+𝑖
= −(−𝑖)𝑛 = −(−𝑖)𝑛
2 2
𝑛𝜋 𝑛𝜋
𝑐𝑜𝑠 2 − 𝑖 𝑠𝑖𝑛 2 1+𝑖 1 𝑛𝜋 𝑛𝜋 𝑖 𝑛𝜋 𝑛𝜋
= − =− 𝑐𝑜𝑠 + 𝑠𝑖𝑛 − 𝑐𝑜𝑠 − 𝑠𝑖𝑛
2 2 2 2 2 2 2

2𝑧
𝑦 𝑛 = 𝑍 −1 = 𝑆𝑢𝑚 𝑜𝑓 𝑎𝑙𝑙 𝑟𝑒𝑠𝑖𝑑𝑢𝑒𝑠
𝑧 − 1 𝑧2 + 1

1 𝑛𝜋 𝑛𝜋 𝑖 𝑛𝜋 𝑛𝜋 1 𝑛𝜋 𝑛𝜋
𝑦 𝑛 =1− 𝑐𝑜𝑠 + 𝑠𝑖𝑛 + 𝑐𝑜𝑠 − 𝑠𝑖𝑛 − 𝑐𝑜𝑠 + 𝑠𝑖𝑛
2 2 2 2 2 2 2 2 2
𝑖 𝑛𝜋 𝑛𝜋
− 𝑐𝑜𝑠 − 𝑠𝑖𝑛
2 2 2
𝑛𝜋 𝑛𝜋
𝑦 𝑛 = 1 − 𝑐𝑜𝑠 − sin
2 2

By Partial fraction method:

𝐹(𝑧) 2
=
𝑧 𝑧 − 1 𝑧2 + 1

2 𝐴 𝐵𝑧 + 𝐶
= + 2 … (𝑖)
𝑧 − 1 𝑧2 + 1 𝑧−1 𝑧 +1

2 = 𝐴 𝑧 2 + 1 + 𝐵𝑧 + 𝐶 𝑧 − 1 … (1)

Put 𝑧 = 1 in (1),we get

2= 𝐴 1+1 𝐴=1

Equating coefficients of 𝑧 2 in (1) on both sides

𝐴+𝐵 = 0 𝐵 = −𝐴 𝐵 = −1

Put 𝑧 = 0 in (1),we get

𝐴−𝐶 = 2 𝐶 = 𝐴−2 𝐶 = −1

Substituting the values of A,B and C in (i), we get


𝐹(𝑧) 2 1 −𝑧 − 1
= 2
= + 2
𝑧 𝑧−1 𝑧 +1 𝑧−1 𝑧 +1

𝑧 𝑧2 𝑧
𝐹 𝑧 = − 2 − 2
𝑧−1 𝑧 +1 𝑧 +1

−1
2𝑧 −1
𝑧 𝑧2 𝑧
𝑦 𝑛 =𝑍 =𝑍 − 2 − 2
𝑧 − 1 𝑧2 + 1 𝑧−1 𝑧 +1 𝑧 +1

𝑧 𝑧2 𝑧
= 𝑍 −1 − 𝑍 −1 2 − 𝑍 −1 2
𝑧−1 𝑧 +1 𝑧 +1

𝑛𝜋 𝑛𝜋
𝑦 𝑛 = 1 − 𝑐𝑜𝑠 − sin
2 2

15. Solve the difference equation

𝑦 𝑛 + 2 − 4𝑦 𝑛 + 1 + 4𝑦 𝑛 = 0 𝑔𝑖𝑣𝑒𝑛 𝑡𝑕𝑎𝑡 𝑦 0 = 1, 𝑦 1 = 0.

Solution:

Let 𝑍{𝑦(𝑛)} = 𝐹(𝑧)

𝑦 𝑛 + 2 − 4𝑦 𝑛 + 1 + 4𝑦 𝑛 = 0 … (1)

Taking Z-transform on both sides in (1), we get

𝑍 𝑦 𝑛 + 2 − 4𝑦 𝑛 + 1 + 4𝑦 𝑛 =𝑍 0

𝑍 𝑦 𝑛+2 − 4𝑍 𝑦 𝑛 + 1 + 4𝑍 𝑦 𝑛 = 0 … (2) 𝐵𝑦 𝑙𝑖𝑛𝑒𝑎𝑟 𝑝𝑟𝑜𝑝𝑒𝑟𝑡𝑦

𝑍 𝑦 𝑛+2 = 𝑧 2 𝐹 𝑧 − 𝑧 2 𝑦 0 − 𝑧𝑦 1 … (3)

𝑍 𝑦 𝑛+1 = 𝑧𝐹 𝑧 − 𝑧𝑦 0 … (4)

Substituting (3) and (4) in (2), we get

𝑧 2 𝐹 𝑧 − 𝑧 2 𝑦 0 − 𝑧𝑦 1 − 4 𝑧𝐹 𝑧 − 𝑧𝑦 0 + 4𝐹 𝑧 = 0

𝑧 2 𝐹 𝑧 − 𝑧 2 − 𝑧(0) − 4 𝑧𝐹 𝑧 − 𝑧 + 4𝐹 𝑧 = 0

𝑧 2 − 4𝑧 + 4 𝐹 𝑧 − 𝑧 2 + 4𝑧 = 0

𝑧 2 − 4𝑧 + 4 𝐹 𝑧 = 𝑧 2 − 4𝑧

𝑧 2 − 4𝑧
𝐹 𝑧 = 𝑍{𝑦(𝑛)} =
𝑧 2 − 4𝑧 + 4

𝑧 2 − 4𝑧
𝑦(𝑛) = 𝑍 −1
𝑧 2 − 4𝑧 + 4

𝑧 2 − 4𝑧
𝐿𝑒𝑡 𝐹(𝑧) =
𝑧 2 − 4𝑧 + 4
𝑧 2 − 4𝑧 𝑧𝑛 𝑧 − 4
𝑧 𝑛−1 𝐹 𝑧 = 𝑧 𝑛 −1 =
𝑧 2 − 4𝑧 + 4 𝑧 2 − 4𝑧 + 4

𝑇𝑜 𝑓𝑖𝑛𝑑 𝑝𝑜𝑙𝑒𝑠:

𝑧 2 − 4𝑧 + 4 = 0 𝑧 = 2,2

The poles are 𝑧 = 2 which is of order two

𝑧𝑛 𝑧 − 4 𝑧𝑛 𝑧 − 4
𝑧 𝑛 −1 𝐹 𝑧 = =
𝑧 2 − 4𝑧 + 4 𝑧−2 2

Residue at the pole 𝑧 = 2 of order two 𝑅1

𝑑
= lim 𝑧 − 2 2 𝑧 𝑛 −1 𝐹 𝑧
𝑧→2 𝑑𝑧

𝑑 𝑧𝑛 𝑧 − 4
= lim 𝑧−2 2
𝑧→2 𝑑𝑧 𝑧−2 2

𝑑
= lim 𝑧 𝑛 𝑧 − 4 = lim 𝑧 𝑛 + 𝑛𝑧 𝑛 −1 𝑧 − 4
𝑧→2 𝑑𝑧 𝑧→2

= 2𝑛 + 𝑛2𝑛 −1 2 − 4 = 2𝑛 + 𝑛2𝑛 −1 −2

= 2𝑛 − 𝑛2𝑛 = 2𝑛 (1 − 𝑛)

𝑧 2 − 4𝑧
𝑦 𝑛 = 𝑍 −1 = 𝑆𝑢𝑚 𝑜𝑓 𝑎𝑙𝑙 𝑟𝑒𝑠𝑖𝑑𝑢𝑒𝑠
𝑧 2 − 4𝑧 + 4

𝑦 𝑛 = 2𝑛 (1 − 𝑛)

16. Solve the difference equation

𝑦 𝑛 + 3𝑦 𝑛 − 1 − 4𝑦 𝑛 − 2 = 0, 𝑛 ≥ 2, 𝑔𝑖𝑣𝑒𝑛 𝑡𝑕𝑎𝑡 𝑦 0 = 3, 𝑦 1 = −2.

Solution:

Let 𝑍{𝑦(𝑛)} = 𝐹(𝑧)

𝑦 𝑛 + 3𝑦 𝑛 − 1 − 4𝑦 𝑛 − 2 = 0, 𝑛 ≥ 2 … (1)

Replacing 𝑛 𝑏𝑦 𝑛 + 2 𝑖𝑛 (1), 𝑤𝑒 𝑔𝑒𝑡

𝑦 𝑛 + 2 + 3𝑦 𝑛 + 1 − 4𝑦 𝑛 = 0, 𝑛 ≥ 0 … (2)

Taking Z-transform on both sides in (2), we get

𝑍 𝑦 𝑛 + 2 + 3𝑦 𝑛 + 1 − 4𝑦 𝑛 =𝑍 0

𝑍 𝑦 𝑛+2 + 3𝑍 𝑦 𝑛+1 − 4𝑍 𝑦 𝑛 = 0 … (4) 𝐵𝑦 𝑙𝑖𝑛𝑒𝑎𝑟 𝑝𝑟𝑜𝑝𝑒𝑟𝑡𝑦

𝑍 𝑦 𝑛+2 = 𝑧 2 𝐹 𝑧 − 𝑧 2 𝑦 0 − 𝑧𝑦 1 … (4)
𝑍 𝑦 𝑛+1 = 𝑧𝐹 𝑧 − 𝑧𝑦 0 … (5)

Substituting (4) and (5) in (3), we get

𝑧 2 𝐹 𝑧 − 𝑧 2 𝑦 0 − 𝑧𝑦 1 + 3 𝑧𝐹 𝑧 − 𝑧𝑦 0 − 4𝐹 𝑧 = 0

𝑧 2 𝐹 𝑧 − 3𝑧 2 − 𝑧 −2 + 3 𝑧𝐹 𝑧 − 3𝑧 − 4𝐹 𝑧 = 0

𝑧 2 + 3𝑧 − 4 𝐹 𝑧 − 3𝑧 2 + 2𝑧 − 9𝑧 = 0

𝑧 2 + 3𝑧 − 4 𝐹 𝑧 = 3𝑧 2 + 7𝑧

3𝑧 2 + 7𝑧
𝐹 𝑧 = 𝑍{𝑦(𝑛)} =
𝑧 2 + 3𝑧 − 4

3𝑧 2 + 7𝑧
𝑦(𝑛) = 𝑍 −1
𝑧 2 + 3𝑧 − 4

3𝑧 2 + 7𝑧 𝑧 𝑛 3𝑧 + 7
𝑧 𝑛 −1 𝐹 𝑧 = 𝑧 𝑛−1 =
𝑧 2 + 3𝑧 − 4 𝑧 + 4 (𝑧 − 1)

𝑇𝑜 𝑓𝑖𝑛𝑑 𝑝𝑜𝑙𝑒𝑠:

𝑧+4 𝑧−1 =0 𝑧 = −4,1

The poles are 𝑧 = −4,1

Residue at the pole 𝑧 = −4 of order one 𝑅1

= lim 𝑧 + 4 𝑧 𝑛 −1 𝐹 𝑧
𝑧→ − 4

𝑧 𝑛 3𝑧 + 7
= lim 𝑧+4
𝑧→ − 4 𝑧 + 4 (𝑧 − 1)

𝑧 𝑛 3𝑧 + 7 (−4)𝑛 3(−4) + 7
= lim = = (−4)𝑛
𝑧→ − 4 (𝑧 − 1) (−4 − 1)

Residue at the pole 𝑧 = 1 of order one 𝑅2

= lim 𝑧 − 1 𝑧 𝑛 −1 𝐹 𝑧
𝑧→ 1

𝑧 𝑛 3𝑧 + 7
= lim 𝑧 − 1
𝑧→ 1 𝑧 + 4 (𝑧 − 1)

𝑧 𝑛 3𝑧 + 7 1𝑛 3 + 7
= lim = =2
𝑧→ 1 𝑧+4 1+4

3𝑧2 + 7𝑧
𝑦(𝑛) = 𝑍 −1 = 𝑠𝑢𝑚 𝑜𝑓 𝑟𝑒𝑠𝑖𝑑𝑢𝑒𝑠
𝑧2 + 3𝑧 − 4

𝑛
𝑦 𝑛 = −4 + 2
17. Solve the difference equation

𝑦 𝑛 + 2 − 7𝑦 𝑛 + 1 + 12𝑦 𝑛 = 2𝑛 𝑔𝑖𝑣𝑒𝑛 𝑡𝑕𝑎𝑡 𝑦 0 = 0, 𝑦 1 = 0.

Solution:

Let 𝑍{𝑦(𝑛)} = 𝐹(𝑧)

𝑦 𝑛 + 2 − 7𝑦 𝑛 + 1 + 12𝑦 𝑛 = 2𝑛 … (1)

Taking Z-transform on both sides in (1), we get


𝑛
𝑍 𝑦 𝑛 + 2 − 7𝑦 𝑛 + 1 + 12𝑦 𝑛 =𝑍 2

𝑧
𝑍 𝑦 𝑛+2 − 7𝑍 𝑦 𝑛+1 + 12 𝑍 𝑦 𝑛 = … (2) 𝐵𝑦 𝑙𝑖𝑛𝑒𝑎𝑟 𝑝𝑟𝑜𝑝𝑒𝑟𝑡𝑦
𝑧−2

𝑍 𝑦 𝑛+2 = 𝑧 2 𝐹 𝑧 − 𝑧 2 𝑦 0 − 𝑧𝑦 1 … (3)

𝑍 𝑦 𝑛+1 = 𝑧𝐹 𝑧 − 𝑧𝑦 0 … (4)

Substituting (3) and (4) in (2), we get


𝑧
𝑧 2 𝐹 𝑧 − 𝑧 2 𝑦 0 − 𝑧𝑦 1 − 7 𝑧𝐹 𝑧 − 𝑧𝑦 0 + 12𝐹 𝑧 =
𝑧−2
𝑧
𝑧 2 𝐹 𝑧 − 𝑧 2 0 − 𝑧 0 − 7 𝑧𝐹 𝑧 − 𝑧(0) + 12𝐹 𝑧 =
𝑧−2
𝑧
𝑧 2 − 7𝑧 + 12 𝐹 𝑧 =
𝑧−2
𝑧
𝐹 𝑧 = 𝑍{𝑦(𝑛)} =
𝑧−2 𝑧2 − 7𝑧 + 12
𝑧
𝑦(𝑛) = 𝑍 −1
𝑧−2 𝑧2 − 7𝑧 + 12

𝑧 𝑧𝑛
𝑧 𝑛 −1 𝐹 𝑧 = 𝑧 𝑛−1 =
𝑧 − 2 𝑧 2 − 7𝑧 + 12 (𝑧 − 2) 𝑧 − 3 𝑧 − 4

𝑇𝑜 𝑓𝑖𝑛𝑑 𝑝𝑜𝑙𝑒𝑠:

𝑧−2 𝑧−3 𝑧−4 = 0 𝑧 = 2, 3, 4

The poles are 𝑧 = 2, 3, 4

Residue at the pole 𝑧 = 2 of order one 𝑅1

= lim 𝑧 − 2 𝑧 𝑛 −1 𝐹 𝑧
𝑧→ 2

𝑧𝑛
= lim 𝑧 − 2
𝑧→ 2 (𝑧 − 2) 𝑧 − 3 𝑧 − 4
𝑧𝑛 2𝑛
= lim = = 2𝑛−1
𝑧→ 2 𝑧−3 𝑧−4 2−3 2−4

Residue at the pole 𝑧 = 3 of order one 𝑅2

= lim 𝑧 − 3 𝑧 𝑛 −1 𝐹 𝑧
𝑧→ 3

𝑧𝑛
= lim 𝑧 − 3
𝑧→ 3 (𝑧 − 2) 𝑧 − 3 𝑧 − 4

𝑧𝑛 3𝑛
= lim = = − 3𝑛
𝑧→ 3 (𝑧 − 2) 𝑧 − 4 (3 − 2) 3 − 4

Residue at the pole 𝑧 = 4 of order one 𝑅3

= lim 𝑧 − 4 𝑧 𝑛 −1 𝐹 𝑧
𝑧→ 4

𝑧𝑛
= lim 𝑧 − 4
𝑧→ 4 (𝑧 − 2) 𝑧 − 3 𝑧 − 4

𝑧𝑛 4𝑛 4𝑛
= lim = =
𝑧→ 4 (𝑧 − 2) 𝑧 − 3 (4 − 2) 4 − 3 2

𝑧
𝑦 𝑛 = 𝑍 −1 = 𝑠𝑢𝑚 𝑜𝑓 𝑎𝑙𝑙 𝑟𝑒𝑠𝑖𝑑𝑢𝑒𝑠
𝑧−2 𝑧2 − 7𝑧 + 12

1 𝑛
𝑦 𝑛 = 2𝑛−1 − 3𝑛 + 4
2

18. Solve 𝑦𝑛 +2 − 5𝑦𝑛+1 + 6 𝑦𝑛 = 1 , 𝑦0 = 1 , 𝑦1 = 1

Solution:

Let 𝑍{𝑦(𝑛)} = 𝐹(𝑧)

𝑦 𝑛 + 2 − 5𝑦 𝑛 + 1 + 6𝑦 𝑛 = 1 … (1)

Taking Z-transform on both sides in (1), we get

𝑍 𝑦 𝑛 + 2 − 5𝑦 𝑛 + 1 + 6𝑦 𝑛 =𝑍 1
𝑧
𝑍 𝑦 𝑛+2 − 5𝑍 𝑦 𝑛+1 +6𝑍 𝑦 𝑛 = … (2) 𝐵𝑦 𝑙𝑖𝑛𝑒𝑎𝑟 𝑝𝑟𝑜𝑝𝑒𝑟𝑡𝑦
𝑧−1

𝑍 𝑦 𝑛+2 = 𝑧 2 𝐹 𝑧 − 𝑧 2 𝑦 0 − 𝑧𝑦 1 … (3)

𝑍 𝑦 𝑛+1 = 𝑧𝐹 𝑧 − 𝑧𝑦 0 … (4)

Substituting (3) and (4) in (2), we get


𝑧
𝑧 2 𝐹 𝑧 − 𝑧 2 𝑦 0 − 𝑧𝑦 1 − 5 𝑧𝐹 𝑧 − 𝑧𝑦 0 + 6𝐹 𝑧 =
𝑧−1
𝑧
𝑧 2 𝐹 𝑧 − 𝑧 2 − 𝑧 − 5 𝑧𝐹 𝑧 − 𝑧 + 6𝐹 𝑧 =
𝑧−1
𝑧
𝑧 2 − 5𝑧 + 6 𝐹 𝑧 − 𝑧 2 − 𝑧 + 5𝑧 =
𝑧−1
𝑧
𝑧 2 − 5𝑧 + 6 𝐹 𝑧 = + 𝑧 2 − 4𝑧
𝑧−1

𝑧 + 𝑧 2 − 4𝑧 𝑧 − 1
𝑧 2 − 5𝑧 + 6 𝐹 𝑧 =
𝑧−1

𝑧 + 𝑧 2 − 4𝑧 𝑧 − 1
𝐹 𝑧 = 𝑍{𝑦(𝑛)} =
𝑧 − 1 𝑧 2 − 5𝑧 + 6

−1
𝑧 + 𝑧 2 − 4𝑧 𝑧 − 1
𝑦(𝑛) = 𝑍
𝑧 − 1 𝑧 2 − 5𝑧 + 6

𝑧 + 𝑧 2 − 4𝑧 𝑧 − 1 𝑧𝑛 1 + 𝑧 − 4 𝑧 − 1
𝑧 𝑛−1 𝐹 𝑧 = 𝑧 𝑛 −1 =
𝑧 − 1 𝑧 2 − 5𝑧 + 6 (𝑧 − 1) 𝑧 − 2 𝑧 − 3

𝑇𝑜 𝑓𝑖𝑛𝑑 𝑝𝑜𝑙𝑒𝑠:

(𝑧 − 1) 𝑧 − 2 𝑧 − 3 = 0 𝑧 = 1,2, 3

The poles are 𝑧 = 1,2, 3

Residue at the pole 𝑧 = 1 of order one 𝑅1

= lim 𝑧 − 1 𝑧 𝑛 −1 𝐹 𝑧
𝑧→ 1

𝑧𝑛 1 + 𝑧 − 4 𝑧 − 1
= lim 𝑧 − 1
𝑧→ 1 (𝑧 − 1) 𝑧 − 2 𝑧 − 3

𝑧𝑛 1 + 𝑧 − 4 𝑧 − 1 1𝑛 1 + 1 − 4 1 − 1 1
= lim = =
𝑧→ 1 𝑧−2 𝑧−3 1−2 1−3 2

Residue at the pole 𝑧 = 2 of order one 𝑅2

= lim 𝑧 − 2 𝑧 𝑛 −1 𝐹 𝑧
𝑧→ 2

𝑧𝑛 1 + 𝑧 − 4 𝑧 − 1
= lim 𝑧 − 2
𝑧→ 2 (𝑧 − 1) 𝑧 − 2 𝑧 − 3

𝑧𝑛 1 + 𝑧 − 4 𝑧 − 1 2𝑛 1 + 2 − 4 2 − 1
= lim = = 2𝑛
𝑧→ 2 (𝑧 − 1) 𝑧 − 3 (2 − 1) 2 − 3

Residue at the pole 𝑧 = 3 of order one 𝑅3

= lim 𝑧 − 3 𝑧 𝑛 −1 𝐹 𝑧
𝑧→ 3
𝑧𝑛 1 + 𝑧 − 4 𝑧 − 1
= lim 𝑧 − 3
𝑧→ 3 (𝑧 − 1) 𝑧 − 2 𝑧 − 3

𝑧𝑛 1 + 𝑧 − 4 𝑧 − 1 3𝑛 1 + 3 − 4 3 − 1 3𝑛
= lim = =−
𝑧→ 3 (𝑧 − 1) 𝑧 − 2 (3 − 1) 3 − 2 2

𝑧 + 𝑧 2 − 4𝑧 𝑧 − 1
𝑦(𝑛) = 𝑍 −1 = 𝑠𝑢𝑚 𝑜𝑓 𝑎𝑙𝑙 𝑟𝑒𝑠𝑖𝑑𝑢𝑒𝑠
𝑧 − 1 𝑧 2 − 5𝑧 + 6

1 1
𝑦 𝑛 = + 2𝑛 − 3𝑛
2 2

You might also like